You are on page 1of 177

TABLE OF CONTENTS

Introduction to NAC OSCE


General Information............................................................... ...1
Registration for NAC O SC E ................................................ ...1
Fees......................................................................................... ...1
Examination station............................................................... ...1
NAC OSCE scoring.............................................................. ...2
Sample of Therapeutic written test........................................ ...2
Sample clinical case station.................................................... ...3
Therapeutic Guidelines
Medicine
Cardiology.............................................................................. ...7
Dermatology........................................................................... .11
Endocrinology........................................................................ .14
Gastroentermogy.................................................................... .16
Hematology............................................................................ .19
Infectious Diseases................................................................. .19
Neurology............................................................................... .21
Otolaryngology...................................................................... .23
Pulmonology.......................................................................... .24
Rheumatology........................................................................ .26
Nephrology/Urology.............................................................. .29
Emergency Medicine.............................................................. .30
Counseling (smoking/alcohol).............................................. .35
Obstetrics & Gynecology
Sexually transmitted infections.............................................. .38
Urinary tract infection............................................................ .39
Vulvovaginitis......................................................................... .39
Pelvic inflammatory disease................................................... .40
Dysfunctional uterine bleeding............................................. .40
Dysmenorrhea........................................................................ .40
Endometriosis........................................................................ .40
Hormone replacement therapy.............................................. .41
Emergenqr contraception....................................................... .41
Group B Streptococcus in pregnancy.................................... .41
Pregnancy induced hypertension........................................... .41
Ectopic pregnancy.................................................................. .42
Hyperemesis gravidarum........................................................ .42
Drugs contraindicated in pregnancy..................................... . .42
Pediatrics
Acute bronchiolitis................................................................. .45
Acute otitis media.................................................................. .45
Asthma................................................................................... .45
Bacterial tracheitis.................................................................. .45
Bacterial pneumonia............................................................... .46
Croup (Laryngotracheobronchitis)........................................ .48
Epiglottitis.............................................................................. .48
Streptococcal pharyngitis (Group A streptococcus).............. .48
Whooping cough (Pertussis)................................................. .48
Bacterial meningitis................................................................ .49
Febrile seizures....................................................................... .49
Urinary tract infection............................................................ .49
Allergic reaction...................................................................... .50
Anemia................................................................................... .50
Dose of tylenol....................................................................... .50
Immunization schedule.......................................................... .50
TABLE OF CONTENTS
Psychiatry
Delerium..................................................................................................................................... 52
Mania.......................................................................................................................................... 53
Panic disorder..............................................................................................................................53
Social phobia............................................................................................................................... 54
General anxiety disorder............................................................................................................. 54
Obsessive compulsive disorder....................................................................................................55
Post traumatic stress disorder......................................................................................................55
Dementia.....................................................................................................................................55
Depression...................................................................................................................................56
Psychosis......................................................................................................................................56
Mood stabilizers..........................................................................................................................57
Medications causing sexual dysfunction..................................................................................... 58
Substance abuse...........................................................................................................................59
Clinical Examination
Abdominal ..................................................................................................................................63
Cardiovascular ............................................................................................................................ 65
Peripheral vascular ......................................................................................................................67
Respiratory examination............................................................................................................. 69
Central nervous system .............................................................................................................. 71
Upper limb neurological .............................................................................................................73
Lower limb neurological ............................................................................................................ 75
Musculo-skeletal system : Spine/Back........................................................................................ 77
Hip.............................................................................................................................................. 79
Knee.............................................................................................................................................81
Foot and ankle.............................................................................................................................83
Shoulder...................................................................................................................................... 85
Elbow.......................................................................................................................................... 87
Hand and wrist............................................................................................................................88
Breast examination...................................................................................................................... 90
Thyroid........................................................................................................................................91
Mini Mental State Examination.................................................................................................93
Clinical cases
Protocol for history taking.......................................................................................................... 99
Medicine
Atrial fibrillation....................................................................................................................... 102
Asthma...................................................................................................................................... 103
Congestive heart failure.............................................................................................................104
Cerebrovascular attack.............................................................................................................. 105
Dieoxin toxicity.........................................................................................................................106
Infectious mononucleosis (sore throat)..................................................................................... 107
Impotence..................................................................................................................................108
Meningitis................................................................................................................................. 109
Migraine (Headache)................................................................................................................ 110
Myocardial Infarction(Chest pain)............................................................................................I l l
Pneumonia.................................................................................................................................112
Post exposure prophylaxis for H IV ........................................................................................... 113
Pulmonary embolism................................................................................................................ 114
Seizure disorder.........................................................................................................................115
Temporal arteritis......................................................................................................................116
Viral hepatitis............................................................................................................................117
Obstetrics and Gynecology
Abortion....................................................................................................................................118
Antenatal visit........................................................................................................................... 119
Ectopic pregnancy..................................................................................................................... 120
TABLE OF CONTENTS
Infertility.................................................................................................................................... 121
OCP counseling.........................................................................................................................122
Pelvic inflammatory disease....................................................................................................... 123
Placenta previa........................................................................................................................... 124
Pre eclampsia..............................................................................................................................125
Pediatrics
Failure to thrive..........................................................................................................................126
Febrile seizure............................................................................................................................ 127
Measles.................................................................................................... .................................. 128
Neonatal jaundice.......................................................................................................................129
Primary nocturnal enuresis........................................................................................................ 130
Pyloric stenosis...........................................................................................................................131
Speech delay...............................................................................................................................132
Psychiatiy
Anorexia..................................................................................................................................... 133
Bulimia....................................................................................................................................... 134
Delirium..................................................................................................................................... 135
Dementia....................................................................................................................................136
Depression..................................................................................................................................137
Mania......................................................................................................................................... 138
Panic attack................................................................................................................................ 139
Schizophrenia.............................................................................................................................140
Suicide........................................................................................................................................141
Surgery
Back Pain....................................................................................................................................142
Basal cell carcinoma................................................................................................................... 143
Benign prostatic hyperplasia...................................................................................................... 144
Carpal Tunnel Syndrome...........................................................................................................145
Deep Vein Thrombosis.............................................................................................................. 146
Diabetic foot.............................................................................................................................. 147
Difficulty swallowing (Ca oesophagus ) .................................................................................... 148
Hematemesis..............................................................................................................................149
Neck swelling............................................................................................................................. 150
Pain abdomen.............................................................................................................................151
Peripheral vascular disease......................................................................................................... 152
Post operative fever.................................................................................................................... 153
Solitary lung nodule...................................................................................................................154
Thyroid mass..............................................................................................................................155
Trauma....................................................................................................................................... 156
Counseling
Breast feeding.............................................................................................................................159
Child abuse................................................................................................................................ 160
Domestic violence...................................................................................................................... 161
Hormone replacement therapy.................................................................................................. 161
Mammogram............................................................................................................................. 163
Immunization.............................................................................................................................164
Obesity....................................................................................................................................... 165
Smoking..................................................................................................................................... 167
Introduction to NAC OSCE |General Info 1

Introduction to NAC OSCE

General Information
The National Assessment Collaboration, or NAC OSCE, was established to provide a system that streamlines
the assessment of IMG medical knowledge and clinical skills throughout Canada. Many international medical
graduates (IMGs) find that the path to obtaining a medical license in Canada challenging and difficult to
navigate. Different provinces and territories have their own system for assessing IMG medical knowledge and
clinical skills.
Comprised of a number of federal and provincial assessment and educational stakeholders, the NAC OSCE
aims to streamline the evaluation process through which an IMG must navigate to obtain a license to practice
medicine in Canada. Through such a system, an IMGs path to licensure would be the same, regardless of the
jurisdiction in which he or she is being assessed. The NAC OSCE has replaced C EH PEAs Clinical
Examination 1 (CEI), which was unique to Ontario.

Registration for NAC OSCE


Registration for the NAC OSCE in Ontario starts in November, with the deadline in January the next year.
Candidates are advised to complete their registration within this time-frame. Once the deadline is over, the
candidate will not be able to register for the NAC OSCE for the entire year. The exams are scheduled for
March, June, August and September.
Visit www.mcc.ca and www.cehpea.ca for updated information.

Fees
Application Fee: $200 which is non-refundable, NAC OSCE Fee in Ontario: $1850 and Exam Date Change
Fee: $100
All fees are in (CAD) Canadian Dollars.

Examination station
The format for the National Assessment Collaboration (NAC) Objective Structured Clinical Examination
(OSCE) consists of 12 stations based on presentations of clinical scenarios. For a given administration, each
candidate rotates through the same series of stations. Each station is 10 minutes in length with two minutes
between stations.
At each station, a brief written statement introduces a clinical problem and outlines the candidate's tasks (e.g.
take a history, do a physical examination, etc.). In each station, there is at least one standardized patient and a
physician examiner. Standardized patients have been trained to consistently portray a patient problem.
Candidates should interact with standardized patients as they would with their own patients.
The physician examiner observes the patient encounter. For most stations, the candidate will be asked to
respond to a series of standardized oral questions posed by the physician examiner after seven minutes with the
standardized patient. There are no rest stations.
Orientation videos http://www.mcc.ca/en/video/QEII-Orientation/index.html
2 NAC OSCE |A Comprehensive Review

The examination includes a separate written test of candidates' therapeutic knowledge. This component lasts
45 minutes and consists of 24 short-answer questions testing the candidates' knowledge of therapeutics for
patients across the age spectrum and related to pharmacotherapy, adverse effects, disease prevention and health
promotion.

NAC OSCE scoring


The candidate's total examination score will be determined by combining the scores on the OSCE component
with the scores on the therapeutics component. The OSCE score contributes 75 per cent of the total score and
the therapeutics score contributes 25 per cent of the total score. For reporting purposes, the NAC total
examination scores are reported on a scale with a distribution ranging from 0 to 100 with a fixed passing mark
of 65.

Number of times candidates can take the examination


Starting in 2011, the NAC OSCE can be attempted once per Canadian Resident Matching Service (CaRMS)
cycle. If you pass the examination, you can register for the examination a maximum of two additional times if
your eligibility is maintained. Regardless of whether you pass or fail, you can only take the examination three
times. If you take the examination more than once, the most recent result will be the only valid result.

Sample of Therapeutic written test


Question: An otherwise healthy 65 year old woman presents with a 3 week history of aching
and morning stiffness in both shoulders with difficulty dressing. She has no temporal artery
tenderness, headache, jaw pain or visual disturbance. Her ESR (Erythrocyte sedimentation
rate) is 100 and you have made the diagnosis of POLYMYALGIA RHEUMATICA (PMP).
What would you choose as the drug of first choice for initial medical therapy? (Drug, dose,
route of administration and duration are required.)
Answer:______________________________________________________ _
Answer key the marker receives:
PREDNISONE 7.5 - 20 mg PO od for 2-4 weeks following resolution of
symptoms

Question: An otherwise healthy 55 year old male with a history of childhood chickenpox"
presents with a 2 day history of painful unilateral vesicular eruption in a restricted
dermatomal distribution. You make a diagnosis of HERPES ZOSTER (shingles).
What would you choose as the drug of first choice to promote healing and lessen the
neuropathic pain? (Drug, dose, route of administration and duration are required.)
Answer:____________________________________________ ______________
Answer key the marker receives:
VALACYCLOVIR (VALTREX ) 1000 mg PO tid X 7 days OR
FAMCICLOVIR (FAMVIR ) 500 - 750 mg PO tid X 7 days OR
ACYCLOVIR (ZORIVAX ) 800 mg PO 5X /day X 7 days)
Introduction to NAC OSCE |General Info 3

Sample Clinical Case Station


Example instruction written outside the station

David Thompson, 59 years old, presents to your office complaining of jaundice.

In the next 7 minutes, obtain a focused and relevant history.


After the 7 minutes, you will be asked to answer questions about this patient.

Example of post encounter questions

Ql.The abdominal examination of David Thompson revealed no organ enlargement, no masses and no
tenderness. What radiologic investigation would you first order to help discriminate the cause of the
jaundice?

Q2. If the investigations revealed that this patient likely had a post-hepatic obstruction, what are
the two principal diagnostic considerations?

Q3. What radiologic procedure would you consider to elucidate the level and nature of the
obstruction?
Therapeutic Guidelines
This isa
Therapeutic Guidelines |Medicine 7

Therapeutic Guidelines

Medicine

1. Cardiology
Acute Myocardial Infarction : Immediate management in ER
1. Beta blockers: Inj Metoprolol 2.5-5 mg rapid IV q2-5 min, upto MNEMONIC
15 mg over 10-15 minutes, then 15 minutes after receiving 15 mg IV. B : Beta Blockers
2. Then 50 mg PO q6h x 48 hours, then 50-100 mg PO BID. M : Morphine Sulphate
3. Inj Morphine Sulfate IV 2-5 mg every 5-30 min prn 0:0xygen
N : Nitroglycerin
(If pain not relieved with 3 Sublingual Nitroglycerins) A : Aspirin
4. Oxygen by nasal cannula at 4 liters per minute
5. Sublingual Nitroglycerin 0.3-0.6 mg q5min up to 3 times.
6. Non-enteric coated Aspirin 325 mg PO.
7. Cardiology Consultation

Post MI drugs

Drugs Benefits Side effects CofitraliidiiiilWi


ACE Inhibitors 1 mortality Hypotension/dizziness Bilateral renal artery
Ramipril - 10mg hs Prevents ventricular remodelling Hyperkalemia stenosis
Lisinopril - lOmgod 1 proteinuria Angioedema Hx of angioedema
Enalapril - 20mg od Renal insufficiency Pregnancy
Captopril - 50 mg tid Cough, taste changes
ARB 1 mortality Angioedema
Valsartan - 160mgbid 1 proteinuria Cough, taste changes
Candesartan - 32 mg od
Beta Blocker 1 mortality Decreases BP&HR Severe/poorly
Metoprolol - lOOmg bid 1 sudden death, reinfarction & Dizziness, fatigue controlled asthma
Atenolol - lOOmg od arrhythmias Sexual dysfunction 2nd/3rd degree heart
Carvedilol - 25mg bid Cardioselective : preferred for May mask hypoglycemia block
Propranolol - 60-80 tid mild asthma and diabetes Increase risk of HR<50, SBP<90
cardiogenic shock Cocaine use

Statins 1 mortality in post Ml patients Gl upset, muscle aches, Active liver disease,
Atorvastatin - 10mgod with high cholesterol myopathy, rhabdomyo- alcoholics, pregnancy
Simvastatin - 20-40mg od lysis, Impotence

Anti-platelets 1 vascular events Gl upset, Recent/active


ASA - 80-162mg od hypersensitivity bleeding
Clopidogrel - 75mgod Gl bleed Gl intolerance or ASA
Warfarin - 1-10mgod allergy
8 NAC OSCE |A Comprehensive Review

Atrial Fibrillation
1. To control rate:
Inj Metoprolol 5 mg bolus IV, followed by infusion at 0.05 mg/kg/min, increasing as needed
to 0.2 mg/kg/min.
Inj Diltiazem 20 mg bolus. Maintenance infusion of 5-15 mg/hr.
Inj Verapamil 5-10 mg IV over 2-3 min, repeated once after 30 mins.
Tab Amiodarone (in case of heart failure):
Loading dose: 800 - 1600 mg PO in divided doses until response; till max 1000
mg/day divided bid-tid.
Maintenance: 200 mg PO od.
2. To prevent thromboembolism: Assess with CHADS 2 score
No risk: Tab Aspirin 81-325 mg PO od.
1 moderate risk: Tab Aspirin 81-325 mg PO od or Tab Warfarin 2-15 mg PO od to maintain
INR 2-3.
>1 moderate risk or very high risk: Tab Warfarin 2-15 mg PO od to maintain INR 2-3.
3. To control rhythm:
Tab Flecainide 300-400 mg PO bolus dose, maintenance: 50-150 mg PO bid. (First choice)
Tab Sotalol 80-160 mg PO bid. ( Second choice)
Tab Amiodarone (in case of heart failure):
Loading dose: 800 - 1600 mg PO in divided doses until response; till max 1000
mg/day divided bid-tid.
Maintenance: 200 mg PO od.
Electrical Cardioversion: 100-360 joules.

Congestive Cardiac Failure : Immediate management in the ER O ff TREATMENT MNEMONIC


L: Lasix
Oxygen by nasal cannula at 4 liters per minute. M : Morphine Sulphate
N : Nitroglycerin
Inj Furosemide (Lasix) 10 mg IV stat.
0:0xygen
Inj Morphine sulfate IV 2-5 mg every 5-30 min prn. P : Positive airway pressure
Sublingual Nitroglycerin 0.3-0.6 mg q5min up to 3 times. P : Position > 45 degrees
Position of patient > 45 degrees. D : Dopamine (indicated in
cardiogenic shock and
hypotension)_________
Non pharmacological management of Heart Failure
Exercise : Regular physical activity
Salt restriction : symptomatic HF - 2-3g salt/day (Vfc tsp/day) no added salt in diet.
HF with fluid retention : l-2g salt/day (V4 tsp/day)
Fluid intake : 1.5/2L per day in patients with fluid retention or HF not controlled by diuretics.
Daily weight measurement.
Education.
Aggressive risk reduction (BP, glucose, lipids).
Lifestyle modifications, influenza vaccination.
Therapeutic Guidelines |Medicine 9

Dyslipidemia

1. HMG CoA Inhibitors:


Atorvastatin : Tab Lipitor 10-80 mg qhs
Rosuvastatin : Tab Crestor 10-40mg qhs
S/E: Gl symptoms, rash, pruritus, increased liver enzymes, myositis.
C/I: active liver disease, muscle disease, pregnancy.
2. Fibrates: increased TG (triglycerides)
Fenofibrate : Tab Lipidil 67-200 mg/d
3. Bile acid sequestrants : increased LDL
Tab Colestipol 5-30g/day
4. Cholesterol absorption inhibitors:
Tab Ezetimibe lOmg /day.

Lipid Risk LDL Total cholesterol/HDL


HIGH (lOyrCAO >20%) Target LDL-<2.0 Target <4

MODERATE (10yr CAD > 10-19%) Treat if LDL- >3.5 Treat if >5

LOW (10yr CAD <10%) Treat if LDL - >5 Treat if >6

High Risk : All with CAD, CVD, most diabetes cases & chronic renal disease.

Hypertension

Non pharmacological treatment :

Smoking cessation: smoking aggravates hypertension and remains the major contributor to
cardiovascular disease in people under 65 years.
Weight reduction : Maintain BMI<27, particularly in patients with glucose intolerance
Alcohol restriction.
Sodium restriction <150mmol/day.

Blood pressure risk factors Consider treatment if BP BP target


No risk factors >160/100 <140/90
isolated systolic hypertension SBP>160 SBP<140
Moderate-High risk patient >140/90 <140/90
Diabetes or Renal disease >130/80 <130/80
10 NAC OSCE |A Comprehensive Review

Commonly used anti-hypertensive drugs :

Drug Indication Side Effect

Diuretics Uncomplicated HTN, Diabetes Rash, allergic rxn, pancreatitis, sexual


Hydrochlorothiazide HCT - 12.5-25mgod with normal albuminuria, LVH dysfunction. HCT contraindicated in
Spironolactone - 25-50mg od and isolated systolic HTN gout.

Beta Blockers Stable angina, Ml, LVH, Fatigue, insomnia, 1 HR, impotence,
Metoprolol - 50mg bid or 100mg SR od uncomplicated HTN <60 years, dizziness. C/I - asthma/COPD, 2"<,/3,,,
Propranolol - 80mg bid degree heart block, uncompensated HF
Atenolol - 50-100mgod severe PAD

ACE Inhibitors Heart failure, diabetes, post Ml, Cough, loss of taste, rash, angioedema,
Ramipril - 10mghs uncomplicated HTN, LVH, prior renal failure, IBP
Lisinopril - 10mgod CVA/TIA, renal disease, all C/I - b/l renal artery stenosis, Hx of
Enalapril - 10-20mgod
coronary artery disease pts. angioedema, pregnancy
Captopril - 25-50 mg bid
Angiotensin II Receptor Blockers Diabetes, uncomplicated HTN, Fatigue, headache, rash, angioedema,
Losartan - 25-50mg od isolated systolic HTN, LVH, i BP, t K+, pancreatitis.
Valsartan - 80-160mgod patients unable to tolerate ACEI. C/I - b/l renal artery stenosis, Hx of
Candesartan - 8-16mgod angioedema, pregnancy

Calcium Channel Blockers Uncomplicated HTN, LVH, Angina, Dizziness, headache, rash, edema,
Amlodipine - 2.5-10mgod Isolated systolic HTN, diabetes gingival hypertrophy, worsen HF
Nefidipine- 10mgtid without nephropathy C/I - hypotension, recent Ml with
Verapamil - 40-80mg tid pulmonary edema, sick sinus
Diltiazem - 30-60mgtid syndrome, 2nd/3rd AV block

Methyldopa - 125mg bid to 500mgqid First-line for hypertension in Sedation, dry mouth, hepatotoxic,
pregnancy lupus like symptoms.

Infective Endocarditis Prophylaxis

Inj Ampicillin 2g IV q4h x 4weeks.


Inj Gentamicin 1mg/kg IV q8h x 4 weeks.
Prophylaxis: Dental/respiratory/esophageal procedure: Tab Amoxicillin 2g PO 30-60 min prior;
Tab Clindamycin 600mg PO, if allergic to penicillin.

Rheumatic Heart Disease (EHD)

Tab Erythromycin 500 mg tid PO x lOdays.


Tab Penicillin VK 500 mg PO bid x 10 days.
Therapeutic Guidelines |Medicine 11

2. Dermatology
Acne

Mild : <20 comedones (whiteheads/blackheads) or <15 inflammatory papules, or a lesion count <30
Moderate : 15-50 papules and pustules with comedones, cysts are rare, lesion count ranges from 30-125
Severe : Primarily nodules and cysts,also present are comedones, papules and pustules, scarring is present,
lesion count >125

T Benzoyl Peroxide (Antibacterial/Keratolytlc) Indication: 1st line S/E : contact dermatitis,


0
Dose : apply to entire affected area qhs or bid medication for mild- dryness, erythema, burning
P
i moderate acne. & pruritis
c
A Tretinoin (Retinoid) 1st line treatment for mild- S/E :erythema, dryness,
L Dose : qhs, apply 30-45 minutes after wash moderate comedones acne. burning, photosensitivity.

Oral antibiotics Indicated for moderate- S/E : Gl upset, nausea,


Tetracycline - initial 500mg bid then 250-500mg od severe acne. vomiting, candidiasis.
Doxycydine - 100mgod Acne on chest, back & C/I : liver disease
Erythromycin - initial 500mg bid then 250-500mg od shoulders
S Combined Oral Contraceptive Females with moderate- C/I : Smoking, migraine with
Y
5 Diane 35/Yasmin/Alesse : od x 21 days, 7 days severe acne + seborrhea + aura, seizures
T off/cycle hirsutism, late onset acne
E
M Isotretinoin Severe nodulocystic acne, Teratogenicity : ocular
1 Accutane : 0.5-1 mg/kg/day x 16-20 weeks acne with scarring, failure to effects - conjunctivitis,
c
"Important : Tests for pregnancy 30 days prior to respond to other treatments 1 night vision, premature
starting Accutane, before each refill. Patient has to epiphyseal closure, t LFTs,
sign an informed consent. pseudomotor cerebri,
mucocutaneous effects,
myalgias. Photosensitivity.

Bums

Initial assessment of ABCs , consider the need for early intubation if airway is compromised.
Humidified 0 2 if any suspicion for inhalational injury.
Oxygen 100% if known carbon monoxide exposure of fire in an enclosed space. (Half life of
hemoglobin will drop from 330 to 90 mins).
Establish IV access.
Fluid resuscitation : Parkland formula 4mL/kg/%BSA burn, Vi over 8 hours and rest over 16 hours
Nasogastric tube drainage for ileus.
Bladder catheterization to monitor urinary output, minimum lmL/kg/hr.
Tetanus prophylaxis : 0.5 mL tetanus toxoid IM in previously immunized and 250 units TIG IM if
unimmunized.
12 NAC OSCE |A Comprehensive Review

Psoriasis

Topical Preparations :

1. Topical Corticosteroids :
High Potency Topical Steroids (Usually indicated)
Very high potency: e.g. Clobetasol (Temovate)
High potency: e.g. Fluocinonide (Lidex)
Low Potency Topical Steroids (Alclometasone dipropionate) usually indicated in
Face
Genitals
Maintenance Therapy
2. Vitamin D based topicals :
Calcipotriene (Dovonex)
Used in combination with Topical Corticosteroids
3. Retinoid based topicals :
Tazarotene (Tazorac)
More irritating than Calcipotriene
4. Immunosuppressant based topicals :
Tacrolimus 0.1% or Pimecrolimus 0.1% creams
Effective in facial and intertriginous Psoriasis
5. Adjunctive agents in combination with above :
Topical Salicylic Acid (Keratolytic Agent)
6. Poorly tolerated topicals (use Calcipotriene instead) :
Historically used with UVB light exposure
Anthralin (Anthra-Derm)
Coal Tar (e.g. Zetar)

Ultraviolet light

Risk of non-Melanoma skin cancer


Protocols
Ultraviolet B exposure alone
Ultraviolet A exposure with psoralen (PUVA)
Increased risk of non-Melanoma skin cancer

Systemic agents (most are for higher risk)

Immunosuppressants
Etretinate
Cyclosporine
Methotrexate (unclear efficacy)
Therapeutic Guidelines |Medicine 13

Biological agents
Tumor necrosis factor (TNF) receptor blockers
Etanercept (Enbrel)
Infliximab (Remicade)
Other mechanisms
Alefacept (Amevive)
Efalizumab (Raptiva)
Thiazolidinedione (Avandia, Actos) - experimental
Appears effective in Psoriasis even in non-diabetics
Only small trials support to date

Cellulitis

Cause : P Hemolytic Streptococcus , Staphylococcus


Treatment : Tab Cloxacillin 500mg PO qid x 10-14 days
If patient is allergic to penicillin : Tab Cephalexin 500mg PO qid 10-14 days OR
Tab Clindamycin 300mg PO qid x 10-14 days

Pediculosis

Permethrin 1% - wash hair with regular shampoo, then apply permethrin and leave for 10 mins then
rinse
Pyrethrins with piperonyl butoxide
Lindane 1% C/I in neonates, young children and pregnant women, causes neurotoxicity
Wash all clothes and linen in hot water, then machine dry.

Scabies

Permethrin 5% - massage into all skin areas, from the top of the head to the soles of the feet, leave for
8-14 hours then wash off.
Crotamiton 10%
Scabene (aerosol spray)
Lindane : used only if allergic to permethrin.
Treat family and contacts.
Wash all clothes and linen in hot water, then machine dry.

Tinea Cruris/Pedis (Jock itch/Athlete's foot)

Clotrimazole 1% cream apply bid


Ketoconazole 2% cream apply bid
14 NAC OSCE |A Comprehensive Review

3. Endocrinology
Diabetes Mellitus

Blood glucose target

A1C q3-6 months Target <7 Normal range<6

Fasting plasma glucose Target 4-7mmol/L Normal range 4-6mmol/L

Post prandial blood glucose 2h Target 5-10mmoi/L Normal range 5-8mmol/L

Approach to management of diabetes mellitus

1. Lifestyle modification & patient education


2. Oral hypoglycemic monotherapy :
Biguanides (Metformin) - 250-500mg PO bid-tid (if obese or overweight)
Sulfonylureas (Glyburide) - 80mg PO bid
Thiazolidinedione (Pioglitazone) - 15mg PO od; Rosiglitazone - 4mg PO od
Alpha glucosidase inhibitors (Acarbose) - 50mg PO tid
3. Oral combination therapy (2 agents often needed; after 3 years 50%, after 9 years 75%)
4. Insulin therapy +/- oral hypoglycemics

Diabetic Ketoacidosis Management

Fluid replacement
Initial : Give 1 liter NS bolus over first 45 minutes, repeat fluid bolus until shock corrected.
Next : Replace first 50% volume deficit in first 8 hours, use Normal Saline or Lactated
Ringers. Replace remaining 50% deficit over next 16 hours, use D5 1/2 NS at 150-250 ml per
hour.
Insulin (Hypokalemia must be corrected prior to Insulin)
Initial
i. Give IV bolus of 0.15 units/kg
ii. Start 0.1 units/kg/hour Insulin Drip
Maintenance
i. Anticipate Serum Glucose drop of 50-70 mg/dl/hour
If inadequate drop, then increase drip
a) Increase Insulin Infusion rate by 50-100%
b) Continue at increased rate until adequate
ii. When Serum Glucose <200-250 mg/dl
a) Keep Serum Glucose at 150 to 200 mg/dl
b) Decrease rate by 50% (to 0.05 units/kg) or
c) Discontinue Insulin Drip and start SC dosing
Therapeutic Guidelines |Medicine 15

Potassium
Do not administer Insulin until potassium >3.3
Give KC1 40 mEq/hour IV until corrects
Serum Potassium 3.3 to 5.0 mEq/L
i. Standard replacement: 20-30 mEq per liter
Serum Potassium >5.0 mEq/L
i. Do not administer any potassium
ii. Monitor every 2 hours until <5.0
Bicarbonate
Indications
i. ABG pH < 6.9 to 7.0 after initial hour of hydration
ii. Other contributing factors
Shock or Coma
Severe Hyperkalemia

Hyperthyroidism

Tab Propylthiouracil(PTU) 100 mg PO tid, to max 150 mg 6-8 hours.


Tab Methimazole 10-30 mg PO od.
Medications associated with Hyperthyroidism:
Excess Thyroid hormone intake
Dietary Iodine
Amiodarone

Hypothyroidism

Tab L-Thyroxine 0.05-0.2 mg/day


Medications associated with Hypothyroidism:
i. Inorganic iodine
ii. Iodide
iii. Amiodarone
iv. Lithium

Hyperprolactinemia

Tab Bromocriptine 1.25-2.5 mg PO od, increase by 2.5 mg/day q3-7days to max 15 mg/day.
Tab Cabergoline 0.25 mg PO twice weekly, may increase by 0.25 mg q4weeks up to max lmg twice
weekly.
16 NAC OSCE |A Comprehensive Review

Medications causing hyperprolactinemia


a) Benzodiazepines
b) Buspirone
c) MAOI
d) SSRI
e) TCA
f) Valproic acid
g) Methyldopa
h) Verapamil
i) Atenolol
j) Danazol
k) Estrogen
1) Depo-Provera
m) OCPs
n) Metoclopromide
o) Amphetamines
P) Cannabis

Impotence

Tab Sildenafil 25-lOOmg per dose, to take half an hour to 4 hours prior to intercourse.
S/E: flushing, headache, indigestion
C/I: don't take with Nitrates.

4. Gastroenterology

Appendicitis

Perioperative for 24hrs


Inj Ampicillin l-2g IV q4-6h.
Inj Flagyl 500mg IV bid.
Inj Gentamicin 3-5mg/kg/day q8h (monitor creatinine levels).
NPO

Acute Gastroenteritis
Acute Gastroenteritis Causes
(Watery diarrhea)
Tab Flagyl 500 mg PO bid x 5 days.
E. Coll (Traveler's diarrhea)
Tab Ciprofloxacin 500 mg PO bid x 3 days. CMV
Tab Norfloxacin 400 mg PO bid x 3 days. Cryptosporidium
Oral rehydration solution. Giardia lamblia
Therapeutic Guidelines |Medicine 17

Acute Pancreatitis

NPO
Inj Flagyl 400 mg IV q8h
Inj Meperidine 75-lOOmg IV q2-3h
I VF
NG tube
Replace calcium

Crohns Disease

1. Mild to moderate:
Tab Mesalamine 800 mg PO tid. Maintenance dose 3.2 - 4g per day.
Tab Sulfasalazine 250 mg per day and increase up to 2 g per day. Maintenance dose is 500-
1000 mg PO qid with food.
2. Moderate to severe:
Tab Prednisone 40 mg PO qid x 8-12 weeks and taper gradually.
Tab Azathioprine 2-2.5 mg/kg/day. Used for maintenance while tapering corticosteroids.

Diverticulitis

Inj Flagyl 500mg IV bid.


Inj Ciprofloxacin 500mg IV bid.

Helicobacter Pylori

1. HP-PAC (7 blister pack) 7-14 days


Tab Lansoprazole 30mg PO bid +
Tab Clarithromycin 500mg PO bid +
Tab Amoxicillin lg bid
2. 2ndLINE Quadruple : 14 days
Tab Lansoprazole 30mg PO bid
Tab Flagyl 500mg PO bid
Tab Tetracycline 500mg bid
Tab Bismuth 525mg PO qid
18 NAC OSCE |A Comprehensive Review

Hepatitis B post exposure prophylaxis

1. Known HBsAg Positive Source:


i. Unvaccinated exposed patient:
Hepatitis B Immunoglobulin (HBIG) 0.06 ml/kg and
Hepatitis B Vaccine 0,land 6 months.
ii. Exposed patient with known response to vaccine:
No treatment.
iii. Exposed patient with known failed response to vaccine:
Patient has not yet completed second 3-dose series:
Hepatitis B Immunoglobulin (HBIG) 0.06 ml/kg and
Hepatitis B Vaccine (complete second 3-dose series)
Patient has completed two prior 3-dose series:
Hepatitis B Immunoglobulin (HBIG) 0.06 ml/kg
Second Hepatitis B Immunoglobulin dose.
iv. Exposed patient with unknown response to vaccine:
Test for Antibody to HBsAg
Adequate Antibody (HBsAg Positive): No treatment
Inadequate Antibody (HBsAg Negative)
Hepatitis B Immunoglobulin (HBIG) 0.06 ml/kg and
Hepatitis B Vaccine booster dose:
2. Known HBsAg Negative Source:
i. Administer Hepatitis B Vaccine Series if unvaccinated
ii. No treatment otherwise needed.
3. Unknown HBsAg Source Status:
i. Unvaccinated exposed patient
Hepatitis B Vaccine Series
ii. Exposed patient with known response to vaccine
No treatment
iii. Exposed patient with known failed response to vaccine
Treat source as HBsAg positive if high risk
iv. Exposed patient with unknown response to vaccine
Test for Antibody to HBsAg
Adequate Antibody (HBsAg Positive): No treatment
Inadequate Antibody (HBsAg Negative)
Hepatitis B Vaccine initial and booster dose
Recheck titer in 1 to 2 months
4. Infant with HBsAg Positive Mother:
i. Hepatitis B Immunoglobulin (HBIG) 0.5 ml within 12 hours of birth.
ii. Hepatitis B vaccine: Dose 1 within 12 hours of birth, Dose 2 at age 1 months, Dose 3 at age
6 months.
iii. Repeat HBsAg and HbsAb at 9 months &c 15 months.
Therapeutic Guidelines |Medicine 19

Peptic ulcer disease

Tab Omeprazole 20mg PO od.


Tab Ranitidine 150 mg PO bid.

Ulcerative Colitis

Tab Sulfasalazine 250 mg per day and increase up to 2 g per day. Maintenance dose is 500-1000 mg
PO qid with food.
Tab Mesalamine 800 mg PO tid. Maintenance dose 3.2 - 4g per day.
Rectal suppositories preferred for proctitis.

Acute Cholecystitis (Perioperative)

Inj Cefazolin 0.5-1.5mg IV q6h


NPO
I VF
NG Tube

5. Hematology

Anemia

Iron Deficiency Anemia : Tab Ferrous fumarate(Palafer) 300 mg PO qd OR


Tab Ferrous Sulfate 325 mg PO qd
Megaloblastic Anemia : Tab Ferrous Fumarate 300mg PO qd + Tab Folic acid l-5mg PO qd +
Inj B12 1000 meg q monthly or 1000 - 2000 microgram PO.

6. Infectious Diseases

Prophylaxis for opportunistic infections in HIV patients

Pneumocystis carinii: CD4 count< 200 cells/mm3 or oral candidiasis.


- Tab TMP/SMZ DS PO OD till CD4 counts rises.
Toxoplasma gondii: IgG antibody positive and CD4 count < 100 cells/mm3
- Tab TMP/SMZ DS PO OD till CD4 counts rises.
Mycobacterium tuberculosis: Mantoux > 5 mm in immunocompromised or contact with active TB.
- Tab Isoniazid 300 mg PO OD x 9 months along with
- Tab Pyridoxine 50 mg PO OD.
Mycobacterium avium complex: CD4 counts < 50 cells/mm3.
- Tab Azithromycin 1200 mg PO once a week.
- Tab Clarithromycin 500 mg PO once a week.
Varicella zoster virus: Recent exposure to chicken pox or shingles.
- Varicella zoster immune globulin within < 96 hours of exposure.
20 NAC OSCE |A Comprehensive Review

HIV post exposure prophylaxis

Start within hours of exposure (under 24 to 48 hours).


Triple Therapy for 4 weeks:
1. First two medications: AZT and 3TC (or Combivir)
i. Tab Zidovudine (AZT) 300 mg PO bid and
ii. Tab Lamivudine (3TC) 150 mg PO bid.
2. Third medication (choose one):
i. Tab Indinavir (IDV) 800 mg PO tid or
ii. Tab Nelfinavir (Viracept) 750 mg PO tid or
iii. Tab Efavirenz 600 mg PO qhs.
Obtain baseline labs to monitor for adverse reaction:
1. Pregnancy Test
2. Complete Blood Count with differential and platelets
3. Urinalysis
4. Renal Function Tests
5. Liver Function Tests

Malaria

1. Treatment for active infection:


i. Tab Chloroquine 1 g PO stat, then 500 mg PO 6-8 hours later, then 500 mg PO at 24 hours
& 48 hours after initial dose.
ii. Tab Mefloquine 1250 mg stat dose.
iii. Tab Primaquine 15 mg base PO od x 14 days.
2. Chemoprophylaxis:
i. Tab Chloroquine 500 mg PO once a week.
ii. Tab Mefloquine 250 mg PO once a week.

Pulmonary tuberculosis

1. Initiation Phase: Tab Rifampin 600 mg + Tab Isoniazid 300 mg + Tab Pyrazinamide 2 g for
2 months.
2. Continuation Phase: Tab Isoniazid 300 mg +Tab Rifampin 600 mg for 4 months.
3. Add Tab Pyridoxine (Vit B6) 50 mg PO OD.

Rabies

Post exposure prophylaxis:


Wash wound with soap and water.
Human Rabies Immunoglobulin 20 IU/kg IM stat and half dose into the wound.
Rabies vaccine 1 ml IM on days 0,3,7,14,28.
Inform Public Health.
Capture animal & observe x 10 days, then examine brain for negri bodies.
Therapeutic Guidelines |Medicine 21

Tetanus Prophylaxis : Based upon Tetanus immunization status -

Clean, minor wounds All other wounds


History of tetanus Immunization
Td or Tdap* 0.5ml Tig** 250U Td or Tdap* Tig
Uncertain or < 3 doses of an immunization Yes No Yes Yes
> 3 doses received in an immunization Not No No No
* Adult-type combined tetanus and diphtheria toxoids or a combined preparation of diphtheria, tetanus and acellular
pertussis. If the patient is < 7 years old, a tetanus toxoid-containing vaccine is given as part of the routine childhood
immunization. ** Tetanus immune globulin, given at a separate site from Td (or Tdap)
t Yes, if > 10 years since last booster.
Yes, if > 5 years since last booster. More frequent boosters not required and can be associated with increased adverse
events. The bivalent toxoid, Td, is not considered to be significantly more reactogenic than T alone and is recommended
for use in this circumstance. The patient should be informed that Td (or Tdap) has been given.

7. Neurology

Seizures

1. Acute Management:
Inj Diazepam 5-10mg IV q2-3mins till seizure stops.
Inj Phenytoin 20mg/kg IV at 50mg per min. IP: P~450 Interactions
Inj Phnobarbital 20mg/kg IV at 50-75mg/min H: Hirsutism
E: Enlarged gums
If all fails then rapid sequence intubation. N: Nystagmus
2. Primary Generalized & Partial seizures: Y; Yellow-browning of skin
T: Teratogenicity
Tab Phenytoin: Loading 300mg PO q4h x 3 doses, 0: Osteomalacia
then 300mg PO qhs. I: Interference with folic acid
absorption {hence anemia)
Tab Valproate: Loading 15mg/kg/day, increments by N: Neuropathies: vertigo,
5-10mg/kg/day qweekly, till seizures are controlled. ataxia, headache
Tab Carbamazepine: Start 100-200mg PO od-bid,
increments by 200mg/per q2d, if needed till max
800mg-1200mg per day.
3. Absence Seizures:
Tab Ethosuximide 500mg PO daily in divided doses, increments by 250mg/day q4-7d pm
till max 1500mg per day.

Meningitis

Investigations : CT then LP, CSF analysis, blood C&S, neurology consult


Empirical adult antibiotics : 3rdgeneration cephalosporins + vancomycin + ampicillin
Inj Ceftriaxone 2g IV ql2h
Inj Dexamethasone lOmg q6h IV x 4 days for pneumococcal meningitis
Meningococcal: give contacts Tab Rifampin 600mg PO ql2h x 4 doses
22 NAC OSCE |A Comprehensive Review

CSF Findings :

Normal Bacterial Viral Fungai/TB


Pressure (cmMsO) 5-20 >30 Normal or mildly increased

Appearance Normal Turbid Clear Fibrin web

Protein {g/U 0.18-0.45 >1 <1 0.1-0.5

Glucose (mmol/l) 25-3.5 <2.2 Normal 1.6-2.5

Gram stain Normal 60-90% Positive Normal

Glucose - CSF^enim 0.6 <0.4 >0.6 <0.4


Ratio
WCC <3 >500 <1000 100-500

Other 90% PMN Monocytes Monocytes


10% have >90% PMN
30% have >50% PMN

Cluster headache

Tab Triptan and Tab Prednisone at the beginning of the cycle and prophylactic treatment with
Tab Lithium(300-600mg daily initially then monitor serum levels)
Dihydroergotamine nasal spray 4mg per 1 ml. One spray each nostril and repeat ql5mins.

Migraine

1. Mild - Moderate -NSAIDS


Tab Ibuprofen 200mg tid
Tab Aspirin 600mg PO q4h
2. Moderate - Severe -TRIPTANS
Tab Sumatriptan 25mg PO & repeat q 2hrs prn
Tab Metoclopramide lOmg PO stat
3. Prophylaxis:
Tab Propranolol 60mg PO daily
Tab Amitriptyline 10-25mg PO qhs.

Tension headache

Tab Tylenol 500mg PO 4-6hrs prn.


Therapeutic Guidelines |Medicine 23

Myasthenia Gravis

1. Anticholinesterase (Cholinergic)
Tab Mestinon (Neostigmine and Pyridostigmine): 60-120 mg q3-4h.
2. Immunosuppressive therapy
Tab Prednisone: Start at 20 mg qd, increase gradually by 5 mg every 3 days to 60mg.
Continue for 3 months or until clinical improvement stops or declines. Taper gradually to
every other day
Tab Azathioprine (Imuran) 2 mg/kg/day. Effective when given with Prednisone. Effect not
seen for 6 months or more. Monitor CBC and LFTs.
3. Plasmapheresis (Plasma Exchange) and IV Ig: Indicated for emergent worsening/crisis.
Response rate: 70%.

Parkinsons disease

Tab Carbidopa/Levodopa 25/100 mg PO bid-qid, increase as needed to max 200/2000mg/day.


Tab Bromocriptine 1.25 mg PO bid.
Tab Pergolide 0.05 mg PO od, titrate q2-3 days to the desired effect. Maintenance dose is 3-6 mg/day
in divided doses.
Tab Premipexole 0.125 mg PO tid, increase to 1.5 - 4.5 mg/day in divided doses.
Tab Ropinirole 0.25 mg PO tid, increase weekly to max dose 24 g/day.
Tab Amantadine 100 mg PO od to max 100 mg PO qid.
Tab Selegiline 5 mg PO bid.
Tab Benztropine 0.5-6 mg/day PO in divided doses.
Tab Entacapone 200 mg given concurrently with Carbidopa/Levodopa.

8. Otolaryngology

Acute Sinusitis

Tab Amoxicillin 500mg tid PO x 10 days.


Decongestant: Tab Sudafed 60mg PO q6h
Nasal saline.

Acute Pharyngitis

Group A 6 Hemolytic Strep: Tab Penicillin V 300mg PO tid x lOdays


P e n ic illin /lctuk : l a b E r y d i r o m y c i a 5 0 0 r n u rid x 1 0 days
24 NAC OSCE |A Comprehensive Review

9. Pulmonology

Asthma

1. Intermittent Asthma: Short acting beta-agonist - Salbutamol (Ventolin) Inhaler 1-2 puffs q4-6h pm.
2. Mild Intermittent Asthma:
Long acting beta agonist - Salmeterol Inhaler 1-2 puffs bid.
Inhaled steroids:
i. Fluticasone (Flovent) 2-4 puffs bid.
ii. Budesonide (Pulmicort) 2 puffs bid.
iii. Beclomethasone (Vanceril) 1-4 puffs (40|ig) bid or 1-2 puffs (80pg) bid.
3. Moderate Persistent Asthma:
Inhaled steroids:
i. Fluticasone (Flovent) 2-4 puffs bid.
ii. Budesonide (Pulmicort) 2 puffs bid.
iii. Beclomethasone (Vanceril) 1-4 puffs (40pg) bid or 1-2 puffs (80|ig) bid.
Long acting beta agonist - Salmeterol Inhaler 1-2 puffs bid.
Leukotriene Receptor Antagonist:
Tab Montelukast 10 mg PO qhs.
Tab Zileuton 600 mg PO qid.
4. Severe Persistent Asthma:
High dose Inhaled steroids.
Long acting beta agonist.
Leukotriene Receptor Antagonist.
Systemic Steroids:
i. Tab Prednisone 2 mg/kg/day PO (max 60 mg/day).
ii. Inj Methylprednisolone (Depo-medrol) 2mg/kg IV, then 0.5 mg/kg q6h x 5days.

Acute exacerbation of COPD

Admit with nasal 0 2.Keep saturation between 88-92%. If silent chest/GCS < 8 or decreased LOC
then intubate.
Elevated bed > 45 degrees.
I VF.
MDI : 8 puffs of Ventolin (Salbutamol) alternate with 8 puffs of Atrovent (Ipratropium) back to back
every 20 mins 3 times.
Nebulizer : 2cc Ventolin + lcc Atrovent in 3cc NS q20 mins x 3 times.
Inj Hydrocortisone 125mg IV stat, if severe.
Inj Ceftriaxone 1-2 g IV q24h along with
Inj Piperacillin-Tazobactam 3.375 g IV q6h.
Inj Methylprednisolone 2mg/kg IV, then 0.5 mg/kg q6h x 5 days.
Therapeutic Guidelines |Medicine 25

Community Acquired Pneumonia

1. Outpatient management:
Tab Doxycycline 100 mg PO bid x 7-10 days.
Tab Erythromycin 250 - 500 mg bid x 7-10 days.
Tab Azithromycin 500 mg PO od x 5 days.
Tab Levofloxacin 500 mg PO od x 7-10 days.
Tab Augmentin 500 mg/ 125 mg PO q8h x 5days.
2. Inpatient management:
Inj Ceftriaxone 1-2 g IV bid along with
Inj Levofloxacin 500 mg IV od x 7-10 days.
Inj Azithromycin 50 mg IV over 1 hour od x 1-2 days.

Pulmonary Embolism

1. Investigations
V/Qscan, spiral CT or D-dimer (if unlikely Wells' score < 4)
CBC, INR, PTT, BUN, creatinine, ALT, AST.
2. Management: Initiation
Start Warfarin (Coumadin) concurrent with Heparin.
Contraindicated in pregnancy. (If contraindicated may put IVC filter)
Start Tab Warfarin at 5 mg PO daily on Day 1-2 and Heparin 5000 U IV bolus followed by
continuous infusion 20 U/kg/hour, titrate to INR 2-3 then stop heparin within 24 hours.
Check INR in 3-5 days.
Therapeutic INR: 2.0 to 3.0 IU.
Oxygen, and if pain give morphine or NSAID.
3. Management: Duration of Anticoagulation
Very low risk: 6-12 weeks
Symptomatic isolated calf vein thrombosis.
Low risk patient: 3-6 months
Reversible thromboembolism risk (transient risk such as post-operative PE).
Upper extremity Deep Vein Thrombosis.
Moderate risk patient: 6-12 months
First idiopathic DVT or PE.
High risk patient: 12 months or lifetime Anticoagulation
Recurrent DVT or PE or Thrombophilia.
26 NAC OSCE |A Comprehensive Review

10. Rheumatology

Osteoporosis

Tab Calcium (I500mg/day) and Tab Vitamin D (800 IU/day) intake in diet or as supplements.
Bisphosphonates: Alendronate, Risedronate or Raloxifene.
Hormone Replacement Therapy
Calcitonin
Recombinant Parathyroid Hormone
Lifestyle modifications: Weight bearing exercises, smoking and alcohol cessation.

Osteoarthritis

Tab Tylenol 500 mg PO tid.


Tab Ibuprofen 200-600 mg PO tid.
Tab Naproxen 125-500 mg PO bid.
Tab Celecoxib 200 mg PO od.
Other treatment:
Tab Acetaminophen +Tab Codeine.
Intra-articular corticosteroid injection.
Intra-articular hyaluron injection.
Topical NSAIDs.
Capsaicin cream.
Glucosamine sulfate.

Rheumatoid Arthritis

1. First Choice:
Tab Naproxen 500 mg PO bid.
Tab Ibuprofen 300-800 mg PO qid.
Tab Indomethacin 25-50 mg PO bid or tid.
2. Analgesics: Tab Acetaminophen 500 mg PO tid prn.
3. Corticosteroids: given intra-articular
i. Small Joints:
Inj Hydrocortisone 8-20 mg.
Inj Methylprednisolone 2-5 mg.
Inj Betamethasone 0.8 - 1.0 mg.
ii. Large Joints:
Inj Hydrocortisone 40 100 mg.
Inj Methylprednisolone 1 0 -2 5 mg.
Inj Betamethasone 2 - 4 mg.
Therapeutic Guidelines |Medicine 27

4. Disease Modifying Antirheumatic Drugs (DMARDs): Start within 3 months of diagnosis to reduce
disease progression.
i. Mild disease:
Tab Hydroxychloroquine 200 mg PO bid.
Tab Sulfasalazine 500 mg PO bid to tid.
ii. Moderate disease:
Tab Methotrexate 10-15 mg PO once weekly, then increase to 20 mg PO once
weekly.
Combination therapy:
Methotrexate + Sulfasalazine + Hydroxychloroquine.
Methotrexate + Cyclosporine.
Methotrexate + Etanercept (biological DMARD).
iii. Biological DMARDs: used in persistent disease:
Etanercept SC.
Infliximab IV.
Anakinra SC.
Adalimumab SC.
Abatacept IV.
Rituximab IV.
NOTE:
If Corticosteroids are used for> 3 months, do baseline DEXA and start bisphosphonate therapy.
S/E of Corticosteroids: Osteoporosis, cataracts, glaucoma, peptic ulcer disease, avascular necrosis,
hypertension, increased infection rate, hypokalemia, hyperglycemia, hyperlipidemia.
C/I to Corticosteroids: Active infection, hypertension, diabetes mellitus, gastric ulcer, osteoporosis.

Gout

1. Acute Gout:
i. NSAIDs: Tab Indomethacin 25-50 mg PO tid x 10-14 days.
ii. Tab Naproxen 500 mg PO bid x 4-10 days.
iii. Tab Colchicine 0.6 mg PO qlh till pain relief (max 4-6 doses), then bid x 3-5 days.
iv. Systemic Steroids: (rule out Septic Arthritis)
Inj Methylprednisolone 40 mg IV single dose
Inj Depo-Medrol 80-120 mg IM single dose.
Oral: Tab Prednisone 40 mg PO od x 5days, then gradually taper the dose.
v. Intra- Articular Corticosteroid: used in large single joints & refractory cases.
Inj Betamethasone 7 mg or Inj ACTH 40-80IU.
2. Recurrent Gout: Treat for 3-6 months.
i. Over producers: Tab Allopurinol 100-300 mg/day PO.
ii. Under-excreters: Tab Probenecid 250 mg PO bid (max:1500 mg bid) or Tab Sulfapyrizine 50
mg PO bid (max: 1000 mg bid).
iii. Concurrently start with Tab Colchicine 0.6 mg PO bid x 3-6 months.
28 NAC OSCE |A Comprehensive Review

Temporal arteritis

Start high dose Tab Prednisone 60 mg PO od until symptoms subside and ESR normal
Then 40 mg PO od for 4-6 weeks
Then taper to 5-10 mg PO od for 2 years (relapses occur in 50% if treatment is terminated before 2
years). Treatment does not alter biopsy results if the sample is taken within 2 weeks.
Monitor ESR regularly.
If visual symptoms are present, or develop during treatment, the patient is admitted and given
Inj Prednisolone 1000 mg IV ql2h for 5 days.

Polymyalgia Rheumatica Management

1. General measures
Consider concurrent Temporal Arteritis (See above)
NSAIDs
2. Prednisone (key to management)
See Corticosteroid Associated Osteoporosis
Efficacy: 90% response
Dramatic improvement in first 48 hours
If no response to steroids - reconsider diagnosis
Reconsider diagnosis
Consider Methotrexate
Polymyalgia alone
Dose: 15-20 mg PO qd
Polymyalgia with Temporal Arteritis
Dose: 40-60 mg PO qd
Symptoms and signs remit within 1 month
Decrease dose by 10% each week after improvement
Course
Initial: Maintain starting dose for 1 month
First steroid taper (depends on clinical response)
Taper by 2.5 mg per month down to 10 mg/day then
Taper 1 mg per 4-6 weeks down to 5 to 7.5 mg/day
Final steroid taper
Indicated when symptom free for 6-12 months
Do not taper until sedimentation rate normalizes
Taper by 1 mg every 6-8 weeks until done
Anticipate 2-6 year course of steroids
Relapse common in first 18 months of steroid use
Patients off steroids at 2 years: 25%
Therapeutic Guidelines |Medicine 29

Fibromyalgia

1. ANTIDEPRESSANTS : Benefits
Assists with local pain, stiffness and sleep
Does not affect Tender Points
2. Tricyclic Antidepressants
Amitriptyline (Elavil)
i. First week: 10 mg PO qhs
ii. Next three weeks: 25 mg PO qhs
iii. Later: 50 mg PO qhs
Nortriptyline (Pamelor)
3. Novel Antidepressants
Venlafaxine (Effexor)
Duloxetine (Cymbalta)
4. Selective Serotonin Reuptake Inhibitors (SSRI)
Combination: Fluoxetine and Amitriptyline

Septic Arthritis

Gonococcal: Inj Ceftriaxone lg IV q24h x 2-4 days, then switch to Tab Ciprofloxacin 500 mg PO
bid x 7 days.
Non-Gonococcal: Inj Naficillin 2g IV q4h x 2 weeks, then switch to Tab Ciprofloxacin 500 mg PO
bid x 2-4weeks.

11. Urology/Nephrology

Urinary tract infection (UTI)

1. Acute uncomplicated UTI: outpatient


Tab Bactrim DS PO bid x 3 days.
Tab Nitrofurantoin (Macrobid) 100 mg PO bid x 5 days.
2. Drug resistant UTI: outpatient
Tab Ciprofloxacin 500 mg bid x 3 days.
Tab Norfloxacin 400 mg PO bid x 3 days.
Tab Ofloxacin 200 mg PO bid x 3 days.
3. Acute complicated UTI: inpatient
Inj Ampicillin 1-2 g IV q4-6h and Inj Gentamicin 2mg/kg IV loading dose followed by 1.7
mg /kg q8h IV OD
Inj Ciprofloxacin 400 mg IV bid.
Switch to oral antibiotics upon improvement for a total course of 14-21 days.
30 NAC OSCE |A Comprehensive Review

Acute Pyelonephritis

1. Outpatient management: For acute uncomplicated cases


Tab Ciprofloxacin 500 mg PO bid x 10 days.
Tab Gatifloxacin 400 mg PO daily x 10 days.
Tab Moxifloxacin 400 mg PO daily x 10 days.
Tab Levofloxacin 250 mg PO daily x 10 days.
Tab Augmentin bid x 14 days.
Tab Bactrim bid x 14 days.
2. Inpatient management: IV for 48-72 hours, then switch to oral agents. Total duration of treatment for
14 days.
Inj Ceftriaxone (Rocephin) 1-2 grams IV q24 hours.
Inj Cefotaxime (Claforan) 1 gram IV ql2 hours.
Inj Ampicillin 2 g IV q6h with Inj Gentamicin 2mg/kg IV loading dose , then 1.7mg/kg
q8h.
Inj Piperacillin 3.375g IV q6h.

12. Emergency Medicine/Poisoning

Acetaminophen Intoxication

Toxic level dose is more than 7.5g


Investigations : Monitor drug level stat and then q4h (Acetaminophen nomogram), LFT, INR, PTT,
BUN, Creatinine, ABG, Glucose
Rx : Charcoal/Gastric lavage as per presentation
N-acetyl cysteine 140mg/kg PO, then 70mg/kg q4h for 18 doses

Alcohol withdrawal

Treatment : Inj Diazepam 10-20mg IV


Inj Thiamine lOOmg IM then 50-100mg/day
Fluid resuscitation with D5W l-2mL/kg IV

Allergic Reaction
1. Severe: Inj Epinephrine 0.3-0.5 mg SC/IM stat
2. Mild: Tab Benadryl 25-50mg PO q6h x 3d
3. Tab Prednisone 60mg PO od x 3d

Anaphylaxis
Epinephrine autoinjector (EpiPen) if available
Epinephrine IV or ETT : 1ml of 1:10,000 in adults
Inj Diphenhydramine (Benadryl) 50mg IV or IM q4-6 h
Inj Methylprednisone 50-lOOmg IV according to severity
If wheezing or spasm present : Salbutamol via nebulizer.
Therapeutic Guidelines |Medicine 31

Arrhythmias

Arrhythmias due to 2nddegree and 3rddegree heart block :


Inj Atropine 0.5mg IV while waiting for transcutaneous pacing.
Transcutaneous pacing first (give Inj Midazolam 2mg for sedation)
Admit for transvenous pacing
Unstable patients (hypotensive systolic BP < 90, chest pain, SOB, altered mental status or
unconscious) : CARDIOVERT!
Stable patient :
Atrial fibrillation : either chemical cardioversion (Amiodarone) or electrical (Synchronized DC
cardioversion)
Ventricular tachycardia : DC cardioversion or Inj Lidocaine/Amiodarone 150mg IV over 10 mins.
Ventricular fibrillation : Always defibrillate! Synchronized cardioversion not useful because there is no
QRS complex to synchronize with.
PSVT : Valsalva or carotid massage (after checking for bruit), Inj Adenosine 6mg rapid IV push.
If no response then Metaprolol, Diltiazem.

ASA Intoxication

Investigations : Drug levels, electrolytes, ABG, BUN, Creatinine


Rx : Gastric lavage/Charcoal
Alkalinize urine with D5W, KC1 and NaHC03
Aim : urine pH > 7.5

Diabetic ketoacidosis

Estimated daily basal glucose requirement is 0.5U/kg


Investigations : Blood glucose, electrolytes, ABG, serum ketones, osmolar gap, anion gap, BUN,
creatinine. Look of the cause : Urinalysis, blood C&S, chest x-ray, ECG.
Monitor : Urine output, extra-cellular fluid volume, electrolytes, ABG, creatinine,
capillary blood glucose and level of consciousness every 1-2 hours.
Management : Rehydration : NS lL/h in first 2 hours followed by 0.45% NS 500cc/h then switch
to maintain blood glucose 13.9-16.6mmol/L to avoid rapid decrease of osmolality.
K+ replacement : As acidosis is corrected, hypokalemia may develop.
If K+ is 3.3-5.0 mmol/L, add KC120-30 mEq/L to keep it within this range.
Correct acidosis : If pH < 7.0/hypotension/coma then give 3 amp NaHCOs
(150mEq/L)
Reduce blood glucose : Start Insulin therapy with 0.15U/kg bolus and maintain
0.1U/kg/h until acidosis and blood glucose resolve.
Treat underlying precipitant.
32 NAC OSCE |A Comprehensive Review

Digoxin Intoxication

Investigations : Plasma digoxin/digitoxin levels, ECG, electrolytes, BUN, Cr ( levels > 2.6 indicate
intoxication)
Rx : Treat arrhythmias (common with digoxin intoxication; vfib, vtach, conduction blocks)
Gastric lavage / Charcoal (lg/kg) for ingestion
NaHC03 or glucose and insulin
Ventricular tachycardia : Digibind 10-20 vials if dose unknown
Chronic toxicity : then Digibind 3-6 vials IV over 30 mins.
Follow ECG, K+ Mg+, Digoxin levels every 6 hours.

Hypertensive emergency

Systolic BP > 180mmHg and Diastolic BP > 120mmHg (with signs of acute organ damage)
Investigations : CBC, electrolytes, BUN, Creatinine, ABG, Urinalysis, CXR, ECG, BP in all four
limbs, Fundoscopy, Cardiology consult.
1st Line : Inj Sodium nitroprusside 0.3 mcg/kg/min IV OR Inj Labetalol 20mg IV bolus q 10 mins.
Aortic dissection : Sodium nitroprusside + Beta blocker (esmolol)
Catecholamine excess : Inj Phentolamine 5-l5mg IV q 5-15 mins
Ml/Pulmonary edema : Inj Nitroglycerin 5-20mcg/min IV, increase by 5mcg/min every 5 min till
symptoms improve.

Hypoglycemia

Investigations : Baseline blood glucose, insulin and C-peptide, check glucose ql5 mins
until > 5mmol/L
Rx : If patient can eat/drink : give I5g carbohydrate if BG < 4mmol/L (15g glucose tabs or Z
A caps
of juice or 3 spoons of sugar in water.)
NPO : give 25g carbohydrate if BG < 4mmol/L ( D50W 50cc IV push 1 amp OR
D10W 500cc IV OR glucagon l-2mg IM/SC )

Methanol/Ethylene glycol intoxication

Investigations : CBC, electrolytes, glucose, methanol level.


Rx : Ethanol 10mg/kg over 30 mins OR Inj Fomepizole 15mg/kg IV over 30 mins.
Therapeutic Guidelines |Medicine 33

Opioid Intoxication
Mental status effects include euphoria, sedation, decreased anxiety, a sense of tranquility and
indifference to pain produced by mild-to-moderate intoxication. Severe intoxication can lead to
delirium and coma.
Physiological effects include the following:
Respiratory depression (may occur while the patient maintains consciousness)
Alterations in temperature regulations
Hypovolemia (true as well as relative), leading to hypotension
Miosis
Soft tissue infection
Increase sphincter tone (can lead to urinary retention)
Treatment
IV glucose : 50% Dextrose 50ml
Inj Nalaxone 0.4mg upto 2mg IV for reversal of opioid intoxication.
Inj Thiamine lOOmg IM stat & OD x 3days
0 2 , intubation &, mechanical ventillation

Shock (Cardiogenic/Neurogenic)
Dopamine : l-3mcg/kg/min is the renal dose; 4-10mcg/kg/min is the inotropic dose
Dobutamine : 2.5-5mcg/kg/min

Sprain (Ankle) RICE

Rest
Ice : using bag of ice, apply during the day for 5-20 mins every 2 hours.
Compression : Tensor bandage or special supports.
Elevation : Elevate the ankle as much as possible.
Analgesics as needed.
Crutches if too painful to bear weight.

Stroke

Investigations : CBC, electrolytes, BUN, glucose, creatinine, INR/PTT, lipids, ECG, carotid doppler
if suspecting TIA, ABG, Non contrast urgent CT scan.
Treatments : NPO, Foley catheter, DVT prophylaxis, Neurology consult
Rule out contraindications for thrombolytic treatment.
Urgent neurology consult.
Thrombolysis : rTPA within 3 hours of symptoms
Anti-coagulation : Low dose Heparin 5000 U bid, start Warfarin within 3 days,
monitor INR/PTT
If unable to thrombolyse or anti-coagulate then : Tab ASA 50-325mg od or
Tab Clopidogrel 75mg od
BP control : decrease slowly, IV Labetalol (First line treatment)
Bed rest, analgesics, mild sedation and laxatives, avoid hyperglycemia.
34 NAC OSCE |A Comprehensive Review

TCA Intoxication

Patients who present to the ED following psychotropic drug overdose with GCS < 8 should undergo
intubation at the earliest opportunity to prevent hypoventilation and aspiration pneumonia.
Investigations : Drug levels, ECG, ABG, electrolytes, LFTs, RFTs.
Rx : Activated charcoal lgm/kg via NG
Diazepam for seizures
Wide QRS/Seizures : NaHCOa ( 1-2 mEq/kg bolus dose and then 100-150 mEq in
1L D5/0.45% NaCl infused 100-200 ml/h IV)

Upper Gl Bleed

Stabilize patient with IVF, cross & type, 2 large bore IV cannulas.
Investigations : CBC, platelets, INR, BUN, creatinine, PTT, electrolytes, LFTs
Management : NG tube, NPO, blood transfusion if needed, upper Gl endoscopy
Inj Octreotide 50mcg loading and 50mcg per hour (for varices) SC/IV
Inj Pantoprazole 50mg IV stat and 50mg q8h (gastric ulcer)

Lower Gl Bleed

Stabilize patient with IVF, cross & type, 2 large bore IV cannulas.
Investigations : CBC, platelets, INR/PTT, BUN, creatinine, electrolytes.
Management : NG tube, NPO, blood transfusion if needed, sigmoidoscopy, colonoscopy, angiogram
(for angiodysplasia)

Warfarin Intoxication

Treatment according to INR levels


INR < 5 : Stop warfarin, observation, serial INR/PTT
INR 5-9 : If no risk factors for bleeding, hold warfarin x 1-2 days & reduce maintenance dose.
OR Vitamin K 1-2 mg PO, if patient at increased risk or FFP for active bleeding.
INR 9-20 : Stop warfarin, Vitamin K 2-4 mg PO, serial INR/PTT then additional Vitamin K if
needed or FFP for active bleeding.
INR > 20 : FFP 10-15ml/kg, Inj Vitamin K lOmg IV over 10 min, increase dose of Vitamin K (q4h)
if needed.
Therapeutic Guidelines |Medicine 35

13. Counselling

Smoking cessation

1. Nicotine gums: 2mg if < 25 cig/day, 4mg if > 25cig/day


1 piece ql-2h for l-3mths
2. Nicotine patch:
21 mg per day for 4 weeks
14mg per day for 2 weeks
7mg per day for 2 weeks
3. Nicotine inhaler: 6-16 cartridges per day upto 12 weeks
4. Bupropion(Zyban):
150mg qAM x 3days, then 150mg bid for 7- 12 weeks
Maintenance 150mg bid for upto 6 months.
General
Stop smoking during second week of medication
Stop Bupropion if unable to quit by 7 weeks
Minimum of 8 hours between doses
More is not better
Swallow pills whole (not crushed, divided or chewed).

Alcohol cessation

Protocol: Alcohol Dependence


CAGE Questionnaire

Lab markers C : Have you ever felt the need


Serum Gamma glutamyl transferase or to CUT down on your
drinking?
Carbohydrate deficient Transferrin
A : Have you ever felt
ANNOYED at criticism of
1.Initial Management your drinking?
Tab Thiamine 100 mg PO qd & : Have you ever felt GUILTY
about your drinking?
Tab Folate 1 mg PO qd
E : Have you ever had a drink
Multivitamin qd first thing in the morning
Treat Hypomagnesemia if present (EYE OPENER}?
Seizure precautions
2.Long-Term Abstinence Programs
Alcoholics Anonymous
Detoxification centers
Halfway House
36 NAC OSCE |A Comprehensive Review

3.Adjunctive Medications for abstinence


1.First line (consider Naltrexone with Campral)
Tab Naltrexone
Blocks Opioid receptors
Decreases pleasure from Alcohol
Dosing: 50 mg orally daily
Effective in short-term, but not in long-term
Tab Campral (Acamprosate)
Balances GABA and glutamate neurotransmitters
Reduces anxiety from abstinence
Dosing: 2 tabs PO tid
2.Second line agents to consider
Selective Serotonin Reuptake Inhibitors (SSRI)
Consider especially if comorbid depression
Prozac often used, but other SSRIs effective
Topiramate (Topamax)
Decreases Alcohol use severity and binge drinking
Improves well being, quality of life in Alcoholics
3.Agents to avoid
Antabuse
Taken 250 to 500 mg orally daily
Not recommended due to risk and uncertain benefit

Delirium Tremens

General Protocol (Requires ICU observation)


Tab Diazepam (Valium)
Dose: 10-25 mg PO qlh prn while awake
Endpoint: until adequate sedation
Inj Lorazepam (Ativan)
Dose: 1-2 mg IV qlh prn while awake for 3-5 days
Endpoint: until adequate sedation
Librium (Chlordiazepoxide)
Dose: 50 to 100 mg PO/IM/IV q4h (max: 300 mg/day)
Endpoint: until adequate sedation
NOTES
38 NAC OSCE |A Comprehensive Review

Obstetrics & Gynecology


1. SexuallyTransmitted Infection
a. Chlamydia:
Tab Azithromycin lg PO stat or Tab Doxycycline lOOmg PO bid x 7 days
If pregnant: Tab Erythromycin SOOmg PO tid x 7 days.
Treat partner, Reportable disease.

b. Gonorrhea:
Inj Ceftriaxone 125mg IM stat +Tab Doxycycline lOOmg bid x 7 days.
If pregnant : Inj Spectinomycin 2g IM stat
Treat partner, Reportable disease.

c. Syphilis:
Primary, Secondary, Latent Syphilis (duration less 1 year ):
Inj Benzathine Penicillin G 2.4 MU IM for 1 dose
Treat partner, Reportable disease.
If allergic to Penicillin: Tab Doxycycline 100 mg PO bid for 14 days.
Late latent, Cardiovascular (duration over 1 year)
Inj Benzathine Penicillin G 2.4 MU IM once a week for 3 weeks
If Penicillin allergic : Tab Tetracycline 500 mg PO qid for 4 weeks or
Tab Doxycycline 100 mg PO bid for 4 weeks
Neurosyphilis : Inj Aqueous Penicillin G 3-4 MU IM every 4 hours for 10-14 days.

d. Genital herpes:
First episode: Tab Acyclovir 400mg PO tid x 10 days or
Tab Famciclovir 250 mg tid x 10 days or
Tab Valacyclovir 1 g bid x 10 days
Recurrent: Tab Acyclovir 400mg PO tid x 5 days or
Tab Famciclovir 120 mg bid x 5 days or
Tab Valacyclovir 500 mg bid x 5 days
Suppression: if more than 6 episodes per year
Tab Acyclovir 400mg PO bid x 12 months
Severe episode: Inj Acyclovir 5-10 mg/kg q8h x 5-7 days

e. Genital warts (HPV):


Local treatment with LIQUID NITROGEN repeat every 1-2 weeks
Podophyllotoxin 0.5% gel bid x 3days,then 4 days off - to be repeated for 4
weeks.
Prophylaxis for HPV (for Cervical CA & warts) - Inj Gardasil IM 0,2 and 6 months.
Therapeutic Guidelines |Obstetrics & Gynecology 39

GENERAL INSTRUCTIONS for all sexually transmitted infections:


Treat all partners
Avoid sexual intercourse till treatment completion.
Barrier contraception/ educate about safe sex practices.
Rescreening in 3 months.

SIDE EFFECTS:
DOXYCYCLINE: Drug induced PHOTOSENSITIVITY, use sun screen
ACYCLOVIR: headache, Gl upset, impaired renal function, tremors, agitation, lethargy,
confusion, coma

2. Urinary Tract Infection

Uncomplicated:
Tab Bactrim DS PO bid x 3 days or
Tab Nitrofurantoin lOOmg PO qid x 5days. (with food)
In pregnancy: Treat asymptomatic UTI
Tab Amoxicillin 250mg PO tid or
Tab Macrobid lOOmg PO bid x 10 days.
Pyelonephritis: Acute Uncomplicated:
Tab Ciprofloxacin 500mg PO bid x 10 days or
Tab Augmentin 625mg PO bid x 14 days.

Inpatient: Inj Ceftriaxone lg IV bid for 48 hours then switch to oral drugs +
Inj Gentamicin 50mg IV q8h for 24 hours.

3. Vulvovaginitis
a. Candidiasis:
Tab Miconazole 200mg PV qhs x 3 days or
Tab Nystatin (100,00 unit) vaginal tab PV qhs x 14 days or
Tab Fluconazole I50mg PO stat dose.
Prophylaxis: 4 or more infection per year - Tab Fluconazole 150mg PO every
3days for 3 doses.
Maintenance: Tab Fluconazole 150mg PO each week. Monitor liver enzymes every 1-2
months.
b. Bacterial vaginosis:
Tab Flagyl 500mg PO bid x 7days.(with food)
c. Trichomonas vaginalis:
Tab Flagyl 2g PO for 1 dose, or
Tab Flagyl 500mg PO bid x 7days.(with food), treat partner.
d. Atrophic vaginitis:
Topical Estrogen cream 0.5 to 2g daily to be applied locally.
40 NAC OSCE |A Comprehensive Review

4. Pelvic Inflammatory Disease (PID)

a. Outpatient: Inj Ceftriaxone 250mg IM stat dose + Tab Doxycycline lOOmg PO bid x 14days.
b. Inpatient: Inj Cefoxitin 2g IV q6h + Inj Doxycycline lOOmg IV ql2h.
Continue IV for 48 hrs & then tab Doxycycline lOOmg PO bid x 14 days.
Reportable disease, treat partners, rescreening after 4-6 weeks incase of documented
infection.

5. Dysfunctional Uterine Bleeding (DUB)

a. Mild DUB:
NSAIDs - Tab Mefenamic acid 500mg PO tid x 5 days,
Anitfibrinolytics - Tranexamic acid 500mg PO tid x 5 days, Combined OCPS
Mirena / Provera
Tab Progestin one tab OD in first 10-14days.
b. Severe DUB:
Inj Premarin 25mg IV q4h +Tab Gravol 50mg PO q4h.
With Tab Ovral PO tid till bleeding stops (24hrs),THEN bid for 2 days, THEN od for
3days.
Continue conventional OCPs if pregnancy not desired.

6.Dysmenorrhea

Tab Ibuprofen 400mg PO qid from 1st day of menstrual cycle.


Oral Contraceptive Pills.
Important to rule out secondary causes of dysmenorrhea.

7. Endometriosis

a) NSAIDs : Tab Ibuprofen 400 mg PO qid till symptoms last.


b) Oral Contraceptive pills.
c) Tab Provera 10-20 mg PO OD.
d) Tab Danazol 600-800 mg PO OD for 6 months.
e) GnRH Agonist: Inj Leuprolide 3.75 mg IM once a month for 6 months.
Inj Goserelin 3.6 mg SC every 28 days for 6 months.
Use GnRH Agonist along with Estrogen/Progesterone add back therapy. (To reduce the side
effects of bone loss.)
Therapeutic Guidelines |Obstetrics & Gynecology 41

8. Hormone Replacement Therapy (HRT)

a)Only Estrogen - Tab Premarin 0.625mg PO OD ( only estrogen)


b)Cyclic Dose - Tab Premarin 0.625mg PO OD and Tab Provera S-10mg PO OD from days 1-14.
c)Standard dose - Tab Prempro (premarin 0.625mg and provera 2.5mg) combination pill PO OD.
d)Pulsatile - Tab Premarin 0.625mg PO OD and Low dose Tab Provera 1.5 mg PO OD.
Given as 3 days on and 3 days off.
e) Transdermal : Estradiol transdermal patch twice daily and Tab Provera 2.5 mg PO OD.

9. Emergency contraception

OTC no prescription needed. Take within 72 hours of unprotected intercourse.


Tab Ovral 2 tabs PO ql2h x 2 doses (has Levonorgestrel 0.5mg/dose + estrogen O.lmg/dose) +
Tab Benadryl lOmg 1 hr before dose (emesis induced by Estrogen).
Plan B ( Tab Levonorgestrel 0.75mg/tab) one tab ql2hrs x 2 doses.

10. Group B Streptococcus (GBS) in pregnancy

Inj Penicillin G 5 MU IV then 2.5 MU IV q4h till delivery.


Penicillin allergic: Inj Cefazolin 2 g IV then 1 g q8h or
Inj Clindamycin 900 mg IV q8h or
Inj Erythromycin 500 mg IV q6h.

11. Pregnancy Induced Hypertension (PIH)

a. Initial: To maintain DBP<100


Inj Labetalol 20mg IV bolus every 10-20 mins prn. (C/I asthma,CHF)
Tab Nifedipine XL lOmg PO very 20-30 mins prn.
Inj Hydralazine 5mg or lOmg IM every 20 mins pm, then 5 or lOmg every 3 hrs pm.
(S/E: fetal tachycardia, maternal headache, palpitations)
b. Maintenance:
Tab Methyldopa 250-500mg PO bid - qid.
Tab Metoprolol 25-lOOmg PO bid.
Tab Labetalol 100-400mg PO qid.
c. Anticonvulsant therapy:
Inj Magnesium sulfate 4g IV bolus over 20 min, followed by maintenance of 2-4 g/hour.
Monitor signs of Magnesium toxicity - depressed deep tendon reflexes, decreased respiratory
reflex, anuric, hypotonic, CNS or cardiac depression.
Antagonist to Magnesium sulfate: Calcium gluconate(10%) 10 ml IV over 2 minutes.
d. Avoid these antihypertensives:
ACE(-) & ARBs - neonatal renal failure, teratogenic, IUGR.
Atenolol - IUGR
Thiazide diuretics - maternal fluid depletion.
42 NAC OSCE |A Comprehensive Review

12. Ectopic Pregnancy

a) Inj Methotrexate 50mg/m2BSA IM stat dose.


b) Repeat beta hCG levels weekly till <1.
c) Contraception till beta hcg returns to 5mIU/ml or less.
d) Do CBC, LFTs.

13. Hyperemesis Gravidarum

Tab Diclectin (10 mg Doxylamine with 10 mg Pyridoxin) started as 1 tab qAM + 1 tab qPM +
2 tabs qhs. Maximum 8 tabs per day.

14. Drugs contraindicated in pregnancy

Chloramphenicol: Gray baby syndrome


Erythromycin: Maternal liver damage (used only if allergic to penicillin).
Fluoroquinolones: Cartilage damage.
Metronidazole: Anti-metabolite, high risk in 1st trimester and breast feeding. Can lead to miscarriage.
Sulfa drugs: Miscarriage in 1st trimester and kernicterus in 3rdtrimester.
Tetracyclines: Staining of teeth in children.
ACE inhibitors: IUGR, oligohydraminos, fetal renal defects.
Anticonvulsants:
i) Phenytoin: Fetal hydantoin syndrome - IUGR, facial dysmorphogenesis, cardiovascular
defects, congenital anomalies of hand & foot, umbilical hernia and congenital
anomalies.
ii) Valproic acid: Lumbosacral spina bifida with meningomyelocele or meningocele, often
accompanied by midfacial hypoplasia, deficient orbital ridge, prominent forehead,
congenital heart disease, and decreased postnatal growth.
iii) Carbamazepine: Unique facial appearance and underdevelopment of the fingers, toes, and
nails; developmental delay.
iv) Phnobarbital: Cleft palate/lip, congenital heart disease, intra-cranial hemorrhage.
DES: Vaginal adenosis, adenocarcinoma, uterine malformations in female fetuses.
Lithium: Ebsteins cardiac anomaly, goiter, hyponatremia.
Misoprostol: Congenital facial paralysis with or without limb defects (Mobius syndrome) and Neural
tube defects.
Retinoids: Deformities of the cranium, ears, face, limbs, and liver, hydrocephalus, microcephaly,
heart defects, cognitive defects, craniofacial alteration, cleft palate, neural tube defects,
cardiovascular malformations and kidney alterations.
Therapeutic Guidelines |Obstetrics & Gynecology 43

Warfarin: High risk of spontaneous abortion, stillbirths, IUGR.


Fetal Warfarin Syndrome: Deformities of the axial and appendicular skeleton, hypoplastic
nose, optic atrophy, mental retardation, brachydactyly, scoliosis, mental retardation, intra
cranial hemorrhage.
Alcohol: High incidence of abortion and still births, IUGR.
Fetal Alcohol Syndrome: Decreased muscle tone and coordination, cognitive impairment,
ASD/VSD, narrow small eyes with large epicanthal folds, small head, small mid-face,
indistinct philtrum, thin upper lip.
Cigarette smoking: IUGR, placental abruptio/ previa, spontaneous abortion.
Cocaine: IUGR, microcephaly, prematurity, mental retardation.
NOTES
Therapeutic Guidelines |Pediatrics 45

Pediatrics
1. Acute Bronchiolitis

a. Mild distress: oral/IV hydration, antipyretics for fever, humidified 0 2, VENTOLIN 0.03cc in 3ml NS
by face mask q20min and then qlhr.
b. Moderate to severe distress: all the above + Ribavirin in high risk groups like congenital lung disease,
congenital heart disease, bronchopulmonary dysplasia, immunodeficient patients.
c. Antibiotics, ipratropium, systemic corticosteroids have no use.

2. Acute Otitis Media (AOM)


a) First line:
Tab Amoxicillin 80-90mg/kg/day PO divided q8h for lOd.
If allergic - Tab Azithromycin 10mg/kg/day OD for 3 days. To be given if child > 6months
old.
b) Second line:
Tab Augmentin 90mg/kg/day divided ql2h for 10 days or
Tab Cefuroxime 30mg/kg/day divided bid for 10 days.
Avoid FLUOROQUINOLONES under 16 years age.

3. Asthma

9 Classification (NIH recommendations)


a) Intermittent Asthma- Occasional exacerbations (Less than twice per week).
b) Mild Persistent Asthma- Frequent exacerbations (>twice weekly, but not daily).
c) Moderate Persistent Asthma- Daily symptoms with daily Beta Agonist use
d) Severe Persistent Asthma- Continuous Symptoms and frequent exacerbations.
Acute Management
i. 0 2 (to maintain 0 2 saturation > 90%).
ii. Fluids, if dehydrated.
iii. P2 Agonist : Salbutamol (Ventolin)- 0.03 cc/kg in 3cc NS every 20 minutes for 3 doses then
0.15-0.3 mg/kg (not to exceed 10 mg) every 1-4 hours as needed or 0.5 mg/kg/hour by
continuous nebulization.
iv. If Severe - Ipratropium bromide (Atrovent) lcc added to each of first 3 salbutamol masks.
v. Steroids: Inj Prednisone 2mg/kg in ER, then lmg/kg PO OD x 4d.

4. Bacterial Tracheitis
Airway management, keep child calm.
Humidified 0 2
Nebulized racemic epinephrine(l:1000 solution) in 3ml NS, 1-3 doses, ql-2h.
Inj Ceftriaxone 75-100mg/kg/day q24hrs + Inj Vancomycin 40mg/kg/day in divided doses every
6-8h.
46 NAC OSCE |A Comprehensive Review

5. Bacterial Pneumonia

Newborn (under 3 weeks old)


1. Admit all newborns with Pneumonia.
2. Antibiotic regimen (Use 2-3 antibiotics combined)
a) Antibiotic 1: Ampicillin
i. Age <7 days
Weight <2 kg: 50-100 mg/kg divided ql2h.
Weight >2 kg: 75-150 mg/kg divided q8h.
ii. Age >7 days
Weight <1.2 kg: 50-100 mg/kg divided ql2h
Weight 1.2-2 kg: 75-150 mg/kg divided q8h.
Weight >2 kg: 100-200 mg/kg divided q6h.
b) Antibiotic 2: Gentamicin (dosing below if >37 weeks old)
Age <7 days: 2.5 mg/kg repeated ql2h.
Age >7 days: 2.5 mg/kg repeated q8h.
c) Antibiotic 3: Cefotaxime (optional)
Age <7 days: 100 mg/kg divided ql2h.
Age >7 days: 150 mg/kg divided q8h.
3. Organisms requiring additional antibiotic coverage
i. Methicillin Resistant Staphylococcus Aureus (MRSA)-Vancomycin
a) Age < 7 days :
Weight < 1.2 kg : 15 mg/kg IV OD.
Weight 1.2 - 2 kg : 10-15 mg/kg IV ql2-18h.
Weight > 2 kg 10-15 mg/kg IV q8-12h.
b) Age > 7days , weight > 2 kg : 45-60 mg/kg/day in divided IV q8h.
ii. Chlamydia trachomatis-Erythromycin 30-50 mg/kg/d PO divided q8h.

Management: Age 3 weeks to 3 months


1. Outpatient (if afebrile without respiratory distress)
i. Azithromycin 10 mg/kg day 1,5 mg/kg days 2-5 PO.
ii. Erythromycin 30-40 mg/kg/day PO divided q6h xlOdays.
2. Inpatient (if febrile or hypoxic)
i. Inj Erythromycin 40 mg/kg/day IV divided q6h and
ii. One of the following antibiotics if febrile:
Inj Cefotaxime 200 mg/kg/day IV divided q8h.
Inj Cefuroxime 150 mg/kg/day IV divided q8h.
3. Critically ill
i. Inj Cefotaxime as above and Inj Cloxacillin or
ii. Inj Cefuroxime alone as above
Therapeutic Guidelines |Pediatrics 47

Management: Age 3 months to 5 years


1. Outpatient (if afebrile without respiratory distress)
a) Consider initial parenteral antibiotic at diagnosis:
Inj Ceftriaxone 50 mg/kg/day up to 1 gram IM xl dose.
Start oral antibiotics concurrently as below.
b) First-line oral agents:
Amoxicillin 90 mg/kg/day PO divided q8h x7-10d.
c) Alternative oral agents:
Amoxicillin-Clavulanic Acid (Augmentin).
Erythromycin.
Clarithromycin.
Azithromycin.
2. Inpatient (if febrile or hypoxic):
a) Inj Cefotaxime 150 mg/kg/day IV divided q6h.
b) Inj Cefuroxime 150 mg/kg/day IV divided q8h.
c) If confirmed Pneumococcal Pneumonia:
Inj Ampicillin alone 200 mg/kg/day IV divided q8h.
3. Critically ill:
a) Option 1
Inj Cefotaxime 150 mg/kg/day IV divided q6h and
Inj Erythromycin 40 mg/kg/day IV divided q6h.
b) Option 2
Inj Cefuroxime 150 mg/kg/day IV divided q8h and
Inj Cloxacillin 150-200 mg/kg/day IV divided q6h.

Management: Age 5 to 18 years


1. Outpatient:
a) First-line oral agents:
i. Erythromycin 40 mg/kg/day PO divided q6h x 7-10d.
ii. Clarithromycin 15 mg/kg/day PO divided ql2h x 7-10d.
iii. Azithromycin
Day 1:10 mg/kg day 1 PO (maximum 500 mg).
Days 2-5: 5 mg/kg/day PO (maximum 250 mg).
b) Pneumococcal Pneumonia confirmed:
i. Amoxicillin 90 mg/kg/day PO divided q8h x 7-10d.
2. Inpatient:
a) First line and in critical illness:
i. Inj Cefuroxime 150 mg/kg/day IV divided q8h and
ii. Inj Erythromycin 40 mg/kg/day IV divided q6h.
b) Pneumococcal Pneumonia confirmed:
i. Inj Ampicillin 200 mg/kg/day IV divided q8h.
48 NAC OSCE |A Comprehensive Review

6. Croup (Laryngotracheobronchitis)

a) Humidified 0 2
b) Nebulized racemic epinephrine(l:1000 solution) in 3ml NS, 1-3 doses, ql-2h :
Child < 6mths: 0.25ml
Child > 6mths: 0.5ml
Adolescent: 0.75ml
c) Dexamethasone 0.6mg/kg IM/IV/PO, max dose lOmg, given as a single dose.

7. Epiglottitis

Suspect epiglottitis if child has fever, ill looking, dyspnea, dysphonia, loss of voice, stridor, sudden in
onset.
Investigations : Pharyngeal swab and culture
Blood culture
Lateral X-ray neck (Thumbprint sign)
ABG, CBC
Endoscopy in ER
Treatment : Intubation
IV fluid
IV Cefuroxime

8. Streptococcal Pharyngitis (Group A Streptococcus)

Mclsaac Criteria - no cough, tender anterior cervical lymph nodes, erythematous tonsils with exudate,
fever > 38C, age 3-14 years.
a) If 1 symptom only - no culture or antibiotics needed.
b) If > 1 symptom, culture positive - treat with antibiotics:
Penicillin V 40 mg/kg/day PO divided bid x 10 days.
Erythromycin 40mg/kg/day PO divided tid x 10 days.
Acetaminophen for fever or pain.
c) Invasive GAS: needs admission -
Inj Clindamycin 40 mg/kg divided into 3-4 doses and
Inj Penicillin 250 000 - 400 000 U/kg/day divided into 6 doses x 10 days.

9. Whooping Cough (Pertussis)

a) Erythromycin 40-50mg/kg/day PO divided qid x lOd.


b) Azithromycin lOmg/kg/PO OD dayl, 5mg/kg PO OD day2 to day5. (preferred)
c) Isolate for 5 days of treatment.
d) Erythromycin to all the household members.
Therapeutic Guidelines |Pediatrics 49

10. Bacterial Meningitis (Reportable disease)

a) Inj Dexamethasone 0.6 mg/kg/day IV in 4 divided doses. Start within 1 hour of 1st antibiotic
dose..
b) Ampicillin:
i. Age< 1 month - SO mg/kg IV q8-12h.
ii. Age>l month - 50 mg/kg IV q6h.
c) Cefotaxime:
i. Age < 1 month - 50 mg/kg IV q8-12h.
ii. Age>l month - 200 mg/kg/day IV divided q6-8h.
d) Ceftriaxone:
i. Age< 1 month - 50-75 mg/kg IV divided ql2-24h.
ii. Age> 1 month - 100 mg/kg/d IV divided ql2h.
e) Gentamycin: 2-2.5 mg/kg IV q8h.
f) Vancomycin: 15 mg/kg q6h IV x 7-14 days.
g) Prophylaxis for contacts:
i. H. \nfluenzae : Rifampin 20 mg/kg/day up to 4 days.
ii. N. Meningitides :
Rifampin
Children: 10 mg/kg PO ql2h x 2 days (max 600 mg).
Adults: 600 mg PO ql2h x 2 days.
Ciprofloxacin (adults) 500mg PO for one dose.
Ceftriaxone :
Age<l5 years: 125 mg IM for one dose.
Age> 15 years: 250 mg IM for one dose.

11. Febrile Seizures

a) In ER : Inj Diazepam 0.2 - 0.5mg/kg IV ql5mins till seizure stops.


b) Home : Diazepam rectal suppository.
c) Investigate & treat the cause of fever.

12. U rinary^

1. Oral Treatment- for 7-14 days.


i. Cefixime (Suprax) 8 mg/kg PO divided bid or
ii. Cefpodoxime (Vantin) 10 mg/kg PO divided bid or
iii. Cefprozil (Cefzil) 30 mg/kg PO divided bid or
iv. Cephalexin (Keflex) 50-100 mg/kg PO divided qid.
2. IV antibiotics
i. Inj Cefotaxime 50-150mg/kg/day divided q4-6h or
ii. Inj Ceftriaxone 50-75mg/kg/day divided ql2-24h.
3. UTI Prophylaxis
i. Bactrim (2mgTMP/10mg SMZ per kg) at bedtime qhs.
50 NAC OSCE |A Comprehensive Review

13. Allergic reaction

a) General Measures:
ABC management.
i. Oxygen.
ii. IVF.
iv. Nebulised beta-agonist (Albuterol).
b) Anaphylaxis with airway compromise: Epinephrine (1:1000 solution) 0.01ml/kg SC/IM(upto 0.3ml)
c) Urticaria, Pruritus or Flushing: Inj Diphenhydramine 25-50mg IM/IV every 6hrs prn. Orally same
dose q6h x 3days.
d) Prevention:
i. Medical alert bracelet.
ii. Strict avoidance of allergen.
iii. EpiPen.
iv. Allergy testing and desensitization therapy.

14. Anemia in children

6mg/kg/day elemental iron bid-tid.


Investigate the cause of anemia.

15. Dose ofTylenol in children

a) 40-60 mg/kg/day PO divided q6hr prn (not to exceed 5 doses/24 hours).


b) Neonates: 10-15 mg/kg PO q6-8hr prn.

16. Immunization Schedule for Infants and Children in Canada

I: Routine likinttinteation Sch ed u le for infants and C h ild re n


gB at 0Taf*~ Pneu-
vaccination IPV Hit mmm m C*7 Men~C Tdap HPV inf
8ifth
2 months m m im
4 months m m m
6 m n 8~23
Mmmm and W S
- m
12 WRS JU K
m m* 1-2
! mmmthm m ,, M i\
4~S j j W M TSi - m !

%Z ^ t| | f|
NOTES
52 NAC OSCE |A Comprehensive Review

Psychiatry

1. Agitation & Aggression


Agitation can be defined as excessive verbal and/or motor behavior. It can readily escalate to aggression, which
can be either verbal (vicious cursing and threats) or physical (toward objects or people).

Goals of treatment (Nonpharmacological treatment)

Create a safe environment for treatment


Decrease stimulation
Permit patient to ventilate his or her feelings, but this may need to be cut short if the degree of
agitation is escalating and there is clear danger to self and others.
Behavioral approaches include never turning your back to an agitated patient, talking softly rather than
shouting, and inquiring about what specific needs the patient may have
Prevent further episodes of agitation or aggression

Delirium

Delirium or acute confusional state is a common and severe neuropsychiatric syndrome with core
features of acute onset and fluctuating course, attentional deficits and generalized severe
disorganization of behavior. Treatment of delirium requires treatment of the underlying causes.
Antipsychotics are first-line treatment. Haloperidol is the most effective medication for decreasing
agitation in delirious patients. First generation antipsychotic Loxapine and second generation
(atypical) antipsychotics such as Olanzapine,Risperidone and Quetiapine can also be used.
Benzodiazepines should be reserved for cases of alcohol withdrawal.

First-generation Antipsychotics (FGA)


Haloperidol : 0.5-2.5 mg PO/IM bid ANnpsYCHoncss/e
FGA>SGA
Extrapyramidal S/E - Acute
Loxapine : 12.5-50 mg/day PO dystonia, Parkinsonism,
akathasia, Tardive dyskinesia.
Second generation (atypical) antipsychotics (SGA)
Neuroleptic Malignant
Olanzapine : 5-10 mg/day PO, 2.5-10 mg IM (repeat 2h and 6h prn Syndrome (NMS)
to max of 30 mg/24 h) Sedation
Risperidone : 0.5-2 mg/day PO Weight gain, diabetes and
dyslipidemia
Quetiapine : 25-100 mg/day PO Hyperprolactinemia and
Benzodiazepines sexual dysfunction.
Lorazepam : 0.5-1 mg PO/IM q6-8h
Oxazepam : 10-15 mg PO tid
Therapeutic Guidelines |Psychiatry 53

Mania

Mania is a state of abnormally elevated or irritable mood, arousal, and/ or energy levels. Treatment of
mania involves both acute control of severe agitation by a mood stabilizer and long term mood
stabilizers. Initially atypical antipsychotics such as Risperidone, Olanzapine or Quetiapine are effective.
First-generation Antipsychotics
Haloperidol : 5-10 mg/day PO/IM
Atypical antipsychotics
Risperidone : 2-3 mg/day PO
Olanzapine : 5-20 mg/day PO, 2.5-10 mg IM (repeat 2h and 6h prn to max of 30 mg/24 h)
Quetiapine : start with 100 mg/day PO; increase by 100 mg/day as needed to 300-600 mg/day
divided BID

2. Anxiety Disorders

Anxiety disorders are a group of conditions with exaggerated anxiousness and worry about a number of
concerns persists for an extended period of time.

Goals of treatment (Nonpharmacologic treatment)

Stress reduction and relaxation techniques such as meditation and low impact yoga is often helpful.
Cognitive behavioral therapy (CBT)
Reduction of consumption of caffeine and other stimulants.
Minimize use of alcohol

Panic disorder

Panic attack or panic disorder involves sudden anxiety that occurs without warning. Symptoms can
include chest pain, heart palpitations, sweating, shortness of breath, feeling of unreality, trembling,
dizziness, nausea, hot flashes or chills, a feeling of losing control, or a fear of dying. Panic attacks are
extremely common - 10% to 20% of the population experience a panic attack at some point in their
life. Some people start to avoid situations that might trigger a panic attack; this is called panic attack
with agoraphobia. Panic disorder refers to recurring feelings of terror and fear, which come on
unpredictably without any clear trigger.
SSRIs and SNRIs are the first choice in the treatment of panic disorders. Selective serotonin reuptake
inhibitors (SSRIs) like Citalopram, Escitalopram, Fluoxetine, Paroxetine and Sertraline are all
effective in reducing panic attacks. Serotonin norepinephrine reuptake inhibitor (SNRIs) eg.
Venlafaxine is also used in panic disorder.
There is a delay in the onset of response to these drugs which may be accompanied by initial agitation.
Combining SSRI or SNRI with a brief course of low dose benzodiazepine can increase adherence to
medication and produce rapid response.
Other medication include Tricyclic antidepressants (TCAs) eg. Imipramine, Desipramine and
Clomipramine and Monoamine oxidase inhibitors (MAOIs) eg. Phenelzine, Tranylcypromine.
54 NAC OSCE |A Comprehensive Review

Selective serotonin reuptake inhibitors (SSRIs)


Paroxetine : 20-60 mg/day PO smmnmsmmom
Confusion, disorientation,
Fluoxetine : 20-80 mg/day PO agitation, fever, diaphoresis,
Sertraline : 50-200 mg/day PO sinus tachycardia, HTN,
mydriasis, tachypnea,
Citalopram : 20-60 mg/day PO myoclonic jerks, hyper-
Escitalopram : 10-20 mg/day PO reflexia, muscle rigidity,
tremor, ataxia, incoordination.
Serotonin norepinephrine reuptake inhibitor (SNRIs) TREATMENT: Benzodiazepines
Venlafaxine : 37.5-22.5 mg/day (1 line), Serotonin
Antagonists :Cyproheptadine,
Tricyclic antidepressants (TCAs) Methysergide, Propranolol*
Clomipramine : 75-225 mg/day
Desipramine : 75-300 mg/day
Imipramine : 75-300 mg/day
Benzodiazepines
Alprazolam : 0.25 mg tid-qid, up to 1 mg PO qid
Clonazepam : 0.25-0.5 mg PO bid
Monoamine oxidase inhibitors (MAOIs)
Phenelzine : 45-90 mg/day
Tranylcypromine : 20-60 mg/day

Social Phobia

Social anxiety, also known as social phobia, involves excessive anxiety in social situations where people
fear being embarrassed or made fun of. Situations that can trigger social anxiety include small group
discussions, dating, going to a party, and playing sports. Common symptoms of social anxiety include
blushing, sweating, and dry mouth. People with social phobia often avoid social situations that cause
anxiety.
SSRI and SNRI are mainstay drugs for the treatment for social phobia. Escitalopram, Fluvoxamine,
Paroxetine, Sertraline and Venlafaxine may be used for milder cases.
Simple stage fright or fear of public speaking may respond to low dose Propranolol lOmg taken 30
minutes before the event.

General Anxiety Disorder

Generalized anxiety disorder (GAD) is associated with continual excessive anxiety and worry about a
number of things (e.g., work, money, children, and health). There is no specific source of fear.
Symptoms can include muscle tension, trembling, shortness of breath, fast heartbeat, dizziness, dry
mouth, nausea, sleeping problems, and poor concentration. CBT is the most effective psychosocial
treatment but often takes 20 or more sessions to be effective.
SSRIs and SNRIs have become established as first line treatments for GAD. Bupropion and
Pregabalin are further choices. Low dose benzodiazepines can be used but dependence is a problem.
Buspirone has a low abuse potential and is less sedating than benzodiazepines.
Buspirone : 5mg bid-tid, up to 60 mg/day
Therapeutic Guidelines |Psychiatry 55

Pregabalin : Initial 150 mg/day in 2-3 divided doses, may be increased to 150 mg bid after 1 week if
necessary
Bupropion (Wellbutrin, Zyban): Use : Smoking cessation, second line Antidepressant.
Antidepressant: Start 100 mg bid x 4 days 100 m g tid.

Obsessive-compulsive disorder (OCD)

OCD involves recurring unpleasant thoughts (obsessions) and/or repetitive behaviours (compulsions).
The thoughts may be connected to the repetitive behaviours. For example, people who fear getting an
infection may constantly wash their hands. At times, however, theres no connection at all between the
thoughts and the behaviours.
CBT is the psychotherapy of choice. SSRIs : Fluoxetine, Fluvoxamine, Paroxetine and Sertraline, in
the usual antidepressant dosing range are the drugs of choice in Canada. It may take 6-8 weeks for
symptoms to improve. Second line drugs include Clomipramine, Venlafaxine, Citalopram and
Mirtazapine.

Post-traumatic stress disorder (PTSD)

PTSD is associated with extreme anxiety that appears after a traumatic experience. Symptoms usually
start within 3 months of the traumatic event but may take years to start. PTSD can be associated with
sleep problems, nightmares, irritability, and anger. Feelings of guilt and unworthiness are common with
PTSD. Traumatic experiences that can trigger PTSD include wars, plane crashes, natural disasters
(e.g., hurricane, earthquake), and violent crimes (e.g., rape, abuse).
SSRI and SNRI antidepressants have been shown to be effective in reducing the symptoms of PTSD.
Fluoxetine, Paroxetine, Sertraline and Venlafaxine are first line options.

3. Dementia

Dementia is a serious loss of cognitive ability in a previously unimpaired


DUMBI*
person, beyond what might be expected from normal aging. Dementia is Dressing
not a single disease, but rather a non-specific illness syndrome in which Eating
affected areas of cognition may be memory, attention, language, and Ambulation
problem solving. It is normally required to be present for at least 6 months Toilet
Hygiene
to be diagnosed.
The most common causes of dementia are Alzheimers disease and vascular Shopping
dementia. It affects about 1% of people aged 60-64 years and as many as Housekeeping
30-50% of people older than 85 years. Accounting
Benzodiazepines must be used cautiously in the elderly patients due to Food Making
Transportation
increase risk of falls and impaired cognition.
56 NAC OSCE |A Comprehensive Review

4. Depression

Nonpharmacological treatment
Cognitive behavioral and interpersonal psychotherapy are as effective as antidepressants in mild to
moderate depression.

Pharmacological treatment
Take medication daily, antidepressant must be taken for 2 to 4 weeks for effect to be noticeable.
Medication must be taken even if patient is feeling better.
SSRI
Paroxetine (Paxil): Start 20 mg qhs, increase lOmg every 2wks, max 60mg per day.
Fluoxetine (Prozac): Start 20mg PO qd, avoid increasing more often than monthly, max 80mg PO per
day
Sertraline (Zoloft): Start 50mg PO qAM, increase 50mg every 2 weeks, max 200mg per day
Fluvoxamine (Luvox): Start 25mg PO qhs x 3 days -> 50mg PO qhs x 7 days -> titrate 150-250 mg
daily divided doses bid.
Citalopram (Celexa): Start 20mg PO qd, max 60mg.
Escitalopram (Lexapro): Start lOmg PO qd
Monoamine Oxidase Inhibitor (MAO inhibitor): Use in Atypical depression, Refractory depression.
Isocarboxazid (Marplan) - 10 mg PO bid, max 60 mg per day.
Phenelzine (Nardil) - 15 mg PO tid, max 90 mg per day.
Tranylcypromine (Parnate) - 10-40 mg per day in divided doses, max 60 mg per day.
Complication: Hypertensive crisis, Serotonin syndrome. Interaction with tyramine containing foods to
be avoided strictly.
Serotonin Norepinephrine Reuptake Inhibitors(SNRI) not used these days
Tricyclic Antidepressants: Amitriptyline 25 mg qhs, Nortriptyline (Pamelor)
S/E: Anti-cholinergic - dry mouth, constipation, blurred vision, Anti-histaminergic - sedation, weight
gain; Serotonergic - sexual dysfunction; Orthostatic hypotension; Sinus tachycardia, SVT, Ventricular
tachycardia, Prolonged QT interval, heart block; Withdrawal symptoms.
Other : Venlafaxine (Effexor) 37.5 mg PO od.

6. Psychosis
In acutely psychotic individuals, short-acting parenteral antipsychotics either alone or in combination
with a parenteral benzodiazepine may be recommended. Liquid formulations of atypical antipsychotics may be
used as an alternative to intramuscular injections, Risperidone and Olanzapine are examples.

Atypical antipsychotics :
Clozapine - 12.5 mg PO qd or bid, titrate slowly upwards in increments o f25-50 mg/day
Target dose : 300 - 450 mg/day, max 900 mg/day.
S/E: Agranulocytosis, Diabetes mellitus, hypertriglyceridemia.
NOT 1st LINE Anti-psychotic. Order weekly blood counts for 1 month and then q2 weeks.
Therapeutic Guidelines |Psychiatry 57

Olanzapine (Zyprexa) - Start 5-10 mg PO qd, increase in 5 mg


increments weekly as tolerated, max 20 mg/day.
S/E: Mild sedation, insomnia, dizziness, early AST & ALT Life* threatening neurological
elevation, resdessness, weight gain, increased risk of diabetes disorder presents with muscle
mellitus and hyperlipidemia.
Quetiapine (Seroquel) - Start 25 mg PO bid - tid, increase in 25- Elevated creatine
phosphokinase
50 mg/day increments, target 300-400 mg/day divided doses bid- Rx : dantrolene sodium
tid, max 750 mg/day.
S/E: Headache, sedation, dizziness, constipation.
Risperidone (Risperdal) - Start 1 mg PO bid, slow titration 2-4 mg
PO daily or divided doses bid, max 16 mg/day.
S/E: Insomnia, agitation, EPS, headache, anxiety, hyperprolactenemia, postural hypotension,
constipation, dizziness, weight gain.

Typical antipsychotics:
Haloperidol (Haldol)- 5-10 mg PO, IM, IV. May repeat q30-60mins, max 300 mg per day.
Fluphenazine (prolixin) - 2.5 mg PO bid, max 40 mg per day.

S/E - EXTRA PYRAMIDAL SIDE EFFECTS:


Akinesia - treat with Benztropine 2mg PO /IM/IV OD
Dystonie reaction - treat with Lorazepam or benztropine.
Dyskinesia
Akathesia - treat with Lorazepam, Propranolol or Diphenhydramine.
Perioral tremor
Neuroleptic malignant Syndrome - Muscle rigidity, tremor, delirium, high fever, diaphoresis,
hypertension. Discontinue drug. Give symptomatic treatment and supportive care. Treatment
with Dantrolene or bromocriptine.
Tardive dyskinesia - Blinking, lip smacking, sucking, chewing, grimaces, choreoathetoid movements,
tonic contractions of neck /back.
Treatment - Clozapine.

7. Mood stabilizers : Used in Bipolar disorder.


Lithium bicarbonate - Start 300 mg PO bid, then increase to 900 - 1800 mg per day divided doses.
Serum levels - 0.6 -1 .2 mEq/1, monitor RFTs,TFTs.
S/E: tremor, polydipsia, hypothyroidism, weight gain, nausea/vomiting, diarrhea, ataxia, slurred speech,
lack of coordination.
Treatment of Lithium toxicity:
Discontinue lithium.
Check serum lithium levels, BUN, electrolytes.
IV fluids - Normal saline.
Hemodialysis in case Li > 2 mmol/L, coma, shock, severe dehydration, deterioration,
unresponsive to treatment.
58 NAC OSCE |A Comprehensive Review

Divalproex (Epival) 300-1600 mg/day PO bid.


Monitor LFTs weekly x 1 month, then monthly.
S/E: Liver dysfunction, nausea/vomiting, diarrhea, ataxia, drowsiness, tremor, sedation, hair loss, weight
gain thrombocytopenia, neural tube defects in early pregnancy.
Carbamazepine (Tegretol) 750- 3000 mg/day PO tid.
Monitor weekly CBC due to risk of agranulocytosis.
S/E: Nausea/vomiting, hepatic toxicity, ataxia, drowsiness, confusion, nystagmus, diplopia, transient
leukopenia, agranulocytosis, drug reaction (SJS), neural tube defect in early pregnancy.

8. Medications causing sexual dysfunction


1. Antiandrogen Medications
Spironolactone
Ketoconazole
Finasteride

2. Antihypertensives
Centrally acting sympatholytics (e.g. Clonidine)
Peripherally acting sympatholytics (e.g. Guanadrel)
Beta Blockers
Thiazide Diuretics

3. Antidepressant Medications
Selective Serotonin Reuptake Inhibitors (SSRI)
Tricyclic Antidepressants
MAO inhibitors

4. Sedative-Hypnotic Medications
Barbiturates
Benzodiazepines

5. Drug Abuse
Alcohol Abuse
Heroin abuse
Marijuana abuse
Methadone
Tobacco abuse

6. Other Medications
Anticholinergic Medications
Antipsychotic Medications
H2 Receptor Blockers
Therapeutic Guidelines |Psychiatry 59

9. Substance abuse

Alcohol withdrawal:
Tab Diazepam 20 mg PO ql-2h prn .
Observe for 1-2 hours and re-assess.
Inj Thiamine 100 mg IM then 100 mg PO OD x 3 days.
Maintain hydration.
If oral Diazepam not well tolerated then switch to Inj Diazepam 2-5 mg IV/min - maximum
10-20 mg qlh, or S/L Lorazepam.
If severe liver dysfunction ,severe asthma, respiratory failure or age> 65 years present -
Lorazepam PO/SL/IM 1-4 mg q l-2h.
Hallucination present - Haloperidol 2-5 mg IM/PO ql-4h - max 5 doses/day along with
Diazepam 20 mg x 3 doses as seizure prophylaxis.
Wernickes syndrome:Thiamine 100 mg PO OD x 1-2 weeks.
Korsakoffs syndrome: Thiamine 100 mg PO bid/tid x 3-12 months.

Opioid Intoxication:
ABCs
IV Glucose
Inj Naloxone (Narcan) 0.4 mg - 2mg IV.
Intubation and mechanical ventilation may be required for decreased level consciousness.
Cocaine Overdose:
ABCs
Inj Diazepam 2-5 mg IV/min - maximum 10-20 mg qlh ( to control seizures).
Propranolol or labetalol to treat hypertension and arrhythmia.
Hallucinogens: LSD, mescaline, psilocybin, MDMA.
Symptomatic treatment and supportive care.
Decreased stimulation.
Benzodiazepines or antipsychotics might be required.
Phencyclidine:
Room with minimal stimulation.
Inj Diazepam IV for muscle spasms or seizures.
Haloperidol to suppress psychotic behavior.
NOTES
Clinical Exam ination
Clinical Examination 63

Abdominal Examination

1. Steps before beginning examination

Introduce yourself : I am Dr._______ , your attending physician and Ill be examining you today. At
any point of the examination you feel uncomfortable, please let me know and I'll stop the examination
right there.
Wash/Sanitize hands
Explain to the patient what you are about to do and gain informed consent.
Look for medical equipment/therapies (e.g. drains, colostomy/ileostomy bags).
Verbalize the steps of the examination and your findings.
Use proper draping techniques.

2. Inspection

General inspection of the patient : Is patient comfortable at rest? Do they appear to be tachypnoeic?
Examine the patients hands for presence of koilonhychia (iron deficiency), leukonychia
(hypoalbuminemia), clubbing (IBD, coeliac disease, cirrhosis), palmar erythema, tar staining or
Dupuytrens contracture.
Ask the patient to hold their hands out in front of them looking for a any tremor and then get them
to extend their wrists up towards the ceiling keeping the fingers extended and look for flapping
(asterixis in hepatic encephalopathy).
Examine the face, check the conjunctiva for pallor. Also check the sclera for jaundice. Look at the
buccal mucosa for any obvious ulcers which could be a sign of Crohns disease, B12 or iron deficiency.
Also look at the tongue. If it is red and fat it could be another sign of anaemia, as could angular
stomatitis. Check state of dentition - pigmentation of oral mucosa (Peutz-Jeghers syndrome),
telangectasia, candidiasis.
Examine the neck for an enlarged left supraclavicular lymph node. A palpable enlarged supraclavicular
(Virchows) node is known as Troisiers Sign, may be a sign of malignancy. Virchows node drains the
thoracic duct and receives lymphatic drainage from the entire abdomen as well as the left thorax.
Therefore, enlargement of this node may suggest metastatic deposits from a malignancy in any of
these areas.
Examine the chest, in particular look for gynaecomastia in men and the presence of 5 or more spider
naevi. These are both stigma of liver pathology.
Inspect the abdomen and comment on any obvious abnormalities such as scars, masses and pulsations.
Also note if there is any abdominal distension/ascites. Look for distended veins, striae, Cullens/Grey-
Turners signs (pancreatitis), Sister Mary Joseph's nodule (widespread abdominal cancer)
64 NAC OSCE |A Comprehensive Review

3. Auscultation

Listen with the diaphragm next to the umbilicus for up to 30 seconds.


Listen for bowel sounds - absent (e.g. Ileus, peritonitis), tinkling (bowel obstruction)

4. Palpation

Palpation of the abdomen should be performed in a


systematic way using the 9 named segments of the
1
/l\
HYPO* /
CHONORWl/
: ' HYPO
;
EPKWWmW
I
CHOMORIAL \
abdomen: right and left hypochondrium, right and left
flank, right and left iliac fossa, the umbilical area, the LUMBAfi
hypochondrium and the suprapubic region. UMBi(JCAL
If a patient has pain in one particular area you should
start as far from that area as possible. The tender area t ILIAC
should be examined last as they may start guarding OASTRJUM

making the examination very difficult.


Initial examination should be superficial using one
hand.Once you have examined all 9 areas superficially,
you should examine deeper. This is performed with two hands, one on top of the other.
Feel for organomegaly, particularly of the liver, spleen and kidneys. Palpation for the liver and spleen is
similar, both starting in the right iliac fossa. For the liver, press upwards towards the right
hypochondrium. You should try to time the palpation with the patient's breathing as this presses
down on the liver. If the liver is distended, its distance from the costal margin should be noted.
Palpating for the spleen is as for the liver but in the direction of the left hypochondrium. The edge of
the spleen which may be felt if distended is more nodular than the liver.
To feel for the kidneys you should place one hand under the patient in the flank region and the other
hand on top. You should then try to ballot the kidney between the two hands.

5. Percussion

Percussion over the abdomen is usually resonant, over a distended liver it will be dull. Percussion can also
be used to check for 'shifting dullness' - a sign of ascites. With the patient lying flat, start percussing from
the midline away from you. If the percussion note changes, hold you finger in that position and ask the
patient to roll towards you. Again percuss over this area and if the note has changed then it suggests
presence of fluid such as in ascites. It is also appropriate at this time to check for pedal edema.

6. You should mention to the examiner at this point that you would like to finish the examination with an
examination of the hernial orifices, the external genitalia and also a rectal examination.
Clinical Examination 65

Cardiovascular Examination

1. Steps before beginning examination

Introduce yourself : I am Dr.________, your attending physician and I'll be examining you today. At
any point of the examination you feel uncomfortable, please let me know and I'll stop the examination
right there.
Wash/Sanitize hands
Explain to the patient what you are about to do and gain informed consent.
Look for medical equipment/therapies (e.g. GTN spray, ECG pads, oxygen)
Verbalize the steps of the examination and your findings.

2. Inspection

Start by observing the patient from the end of the bed. You should note whether the patient looks
comfortable. Are they cyanosed or flushed?
Respiratory rate, rhythm and effort of breathing.
Chest shape, chest movements with respration (symmetrical/assymetrical), skin (scars/nevi)
Inspect the nails for clubbing, splinter hemorrhages (infective endocarditis), koilonychia (iron
deficiency anemia).
Inspect fingers for capillary refill time, peripheral cyanosis, osier's nodes (infective endocarditis) and
nicotine staining.
Inspect palms for palmar erythema,Janeway lesions and xanthomas.
Take the radial pulse, assess the rate and rhythm.At this point you should also check for a collapsing
pulse - a sign of aortic incompetence. Locate the radial pulse and place your palm over it, then raise
the arm above the patients head. A collapsing pulse will present as a knocking on your palm.

At this point you should say to the examiner that you would like to take the blood pressure. They will
usually tell you not to and give you the value.

Inspect the sclera for any signs of jaundice, anaemia and corneal arcus. You should also look for any
evidence of xanthelasma.
Whilst looking at the face, check for any malar facies, look in the mouth for any signs of anaemia such
as glossitis, check the colour of the tongue for any cyanosis, and around the mouth for any angular
stomatitis - another sign of anaemia.
Assess jugular venous pressure (JVP), ask patient to turn their head to look away from you. Look
across the neck between the two heads of sternocleidomastoid for a pulsation then measure the JVP.
Examine the chest, or praecordium for any obvious pulsations, abnormalities or scars, remembering to
check the axillae as well.
66 NAC OSCE |A Comprehensive Review

3. Palpation

Palpate praecordium trying to locate the apex beat and describe its location anatomically. The normal
location is in the 5th intercostals space in the mid-clavicular line.
Palpate for any heaves or thrills. A thrill is a palpable murmur whereas a heave is a sign of left
ventricular hypertrophy. Feel for these all over the praecordium.

4. Auscultation

Mitral valve - where the apex beat was felt.


Tricuspid valve - on the left edge of the sternum in the 4th
intercostal space.
Pulmonary valve - on the left edge of the sternum in the
2nd intercostal space.
Aortic valve - on the right edge of the sternum in the 2nd
intercostal space.

How many heart sounds are heard? Are the heart sounds
normal in character? Any abnormal heart sounds? If you hear any abnormal sounds you should describe
them by when they occur and the type of sound they are producing. Are there any murmurs? Can you hear
any rub? Feeling the radial pulse at the same time can give good indication as to when the sound occurs -
the pulse occurs at systole. Furthermore, if you suspect a murmur, check if it radiates. Mitral murmurs
typically radiate to the left axilla whereas aortic murmurs are heard over the left carotid artery.

To further check for mitral stenosis you can lay the patient on their left side, ask them to breathe in,
then out and hold it out and listen over the apex and axilla with the bell of the stethoscope.
Aortic incompetence can be assessed in a similar way but ask the patient to sit forward, repeat the
breathe in, out and hold exercise and listen over the aortic area with the diaphragm.

5. With patient sitting up percuss back for pleural effusion (cardiac failure)

6. Finally assess for any pedal & sacral oedema.

7. Finish by thanking the patient and ensuring they are comfortable and well covered.
Clinical Examination 67

Peripheral Vascular Examination

1. Steps before beginning examination

Introduce yourself : I am Dr._______ , your attending physician and Ill be examining you today. At
any point of the examination you feel uncomfortable, please let me know and I'll stop the examination
right there.
Wash/Sanitize hands
Explain to the patient what you are about to do and gain informed consent.
Look for medical equipment/therapies (e.g. GTN spray, ECG pads, oxygen)
Verbalize the steps of the examination and your findings.

2. Inspection

General observation of the patient, arms from the finger tips to the shoulder and legs from the groin
and buttocks to the toes. Comment on the general appearance of the arms and legs, size, swelling,
symmetry, skin color, hair, scars, pigmentation including any obvious muscle wasting. Note colour and
texture of nails.
Any signs of gangrene or pre-gangrene such as missing toes or blackening of the extremities.
The presence of any ulcers - ensure you check all around the feet including behind the ankle. These
may be venous or arterial - one defining factor is that venous ulcers tend to be painless whereas
arterial are painful.
Any skin changes such as pallor, change in colour (eg purple/black from haemostasis or brown from
haemosiderin deposition), varicose eczema or sites of previous ulcers, atrophic changes and hair loss.
Presence of any varicose veins - often seen best with the patient standing.

3. Palpation

Assess the skin temperature. Starting distally, feel with the back of your hand and compare each limb
to the other noting any difference.
Check capillary return by compressing the nail bed and then releasing it. Normal colour should return
within 2 seconds. If this is abnormal, perform Buergers Test. This involves raising the patients feet
to 45. In the presence of poor arterial supply, pallor rapidly develops. Following this, place the feet
over the side of the bed, cyanosis may then develop.
Any varicosities which you noted in the observation should now be palpated. If these are hard to the
touch, or painful when touched, it may suggest thrombophlebitis.
Palpate peripheral pulses. These are:
Carotid - only palpate one carotid at a time
Radial - use the pad of three fingers
Brachial - may use thumb to palpate
Femoral - feel over the medial aspect of the inguinal ligament.
68 NAC OSCE |A Comprehensive Review

Popliteal - ask the patient to flex their knee to roughly 60 keeping their foot on the bed,
place both hands on the front of the knee and place your fingers in the popliteal space.
Posterior tibial - felt posterior to the medial malleolus of the tibia.
Dorsalis pedis -feel on the dorsum of the foot, lateral to the extensor tendon of the great toe.
You should compare these on both sides and comment on their strength.

Check for radio-femoral delay. Palpate both the radial and femoral pulses on one side of the body. The
pulsation should occur at the same time. Any delay may suggest coarctation of the aorta.

4. Auscultation : listen for femoral and abdominal aortic bruits

5. Special Tests

Allen Test : Ask the patient to make a tight fist and elevate the hand. Occlude the radial and ulnar
arteries with firm pressure. The hand is then opened. It should appear blanched (pallor can be
observed at the finger nails). Release either the Ulnar or radial artery pressure and the color should
return in 7 seconds. If the palm does not redden immediately, this suggests arterial insufficiency.

Straight Leg Raise and Refill Test (Buerger's Test) : Raise the leg 45 to 60 for 30 seconds until
pallor of the feet develops and observe empty veins. Sit the patient upright and observe the feet. In
normal patients, the feet quickly turn pink (within 10-15 seconds). If, pallor persists for more than 10-
15s or there is development of a dusky cyanosis (rubor), this suggests of arterial insufficiency.

Test for incompetent Saphenous Vein : Ask the patient to stand and note the dilated varicose veins.
Compress the vein proximally with one hand and place the other hand 10-15 cm distally. Briskly
compress and decompress the distal site. Normally, the hand at the proximal site should feel no
impulse, however with varicose veins a transmitted pulse may be felt.

Trendelenburg Maneuver (Retrograde filling) : Ask the patient to lie down. Elevate the leg, and
empty the veins by massaging distal to proximal. Using a tourniquet, occlude the superficial veins in
the upper thigh. Ask the patient to stand. If the tourniquet prevents the veins from re-filling rapidly,
the site of the incompetent valve must be above this level i.e. at the sapheno-femoral junction. If the
veins re-fill, the communication must be lower down.
Observing the same protocol, proceed down the leg until the tourniquet controls re-filling. As
necessary, test:
above the knee - to assess the mid-thigh perforator
below the knee - to assess competence between the short saphenous vein and popliteal vein
If re-filling cannot be controlled, the communication is probably by one or more distal perforating
veins.
Clinical Examination 69

Respiratory Examination

1. Steps before beginning examination

Introduce yourself : I am Dr.________, your attending physician and I'll be examining you today. At
any point of the examination you feel uncomfortable, please let me know and I'll stop the examination
right there.
Wash/Sanitize hands
Explain to the patient what you are about to do and gain informed consent.
Look for medical equipment/therapies (e.g. inhalers, oxygen).
Verbalize the steps of the examination and your findings.

2. Inspection

General look of the patient. Check whether they are comfortable at rest, is patient tachypnoeic? Are
they using accessory muscles? Are there any obvious abnormalities of the chest? Check for any clues
around the bed such as inhalers, oxygen masks or cigarettes.
Inspect the hands, hot, pink peripheries may be a sign of carbon dioxide retention. Look for any signs
of clubbing, cyanosis, hypertrophic pulmonary osteoarthropathy, dupytren's contacture and nicotine
staining. Assess for carbon dioxide retention flap/salbutamol tremor.
Take the patients pulse. After you have taken the pulse it is advisable to keep your hands in the same
position and subtly count the patients respiration rate.
Inspect the face, ask the patient to stick out their tongue and note its colour - checking for cyanosis.
- Horner's sydrome (Pancoast tumour), plethora (polycythemia).
Look for any use of accessory muscles such as the sternocleidomastoid muscle. Also palpate for the
left supraclavicular node (Virchow's Node) as an enlarged node (Troisier's Sign) may suggest
metastatic lung cancer.
Examine the chest and back. Observe the chest for any deformities (barrel chest, kyphoscoliosis,
pectus excavatum, pectus carinatum), symmetry of expansion, dilated veins, intercostal recession.

3. Palpation

Palpate the chest. Feel between the heads of the two clavicles for the trachea, see if it is deviated.
Feel for chest expansion. Place your hands firmly on the chest wall with your thumbs meeting in the
midline. Ask the patient to take a deep breath in and note the distance your thumbs move apart.
Normally this should be at least 5 centimetres. Measure this at the top and bottom of the lungs as well
as on the back.
70 NAC OSCE |A Comprehensive Review

4. Percussion

Percussion should be performed on both sides, comparing similar areas on both sides. Start by tapping
on the clavicle which gives an indication of the resonance in the apex. Then percuss normally for the
entire lung fields. Hyper-resonance may suggest a collapsed lung where as hypo-resonance or dullness
suggests consolidation such as in infection or a tumour. Be sure to perform this on the back as well.

5. Vocal Fremitus

Check for tactile vocal fremitus. Place the medial edge of your hand on the chest and ask the patient to say
99\ Do this with your hand in the upper, middle and lower areas of both lungs.

6. Auscultation

Do this in all areas of both lungs and on front and back comparing the sides to each other. Listen for
any reduced breath sounds, or added sounds such as crackles, wheezes or rhonchi.

Tracheal Percussion TactileVocal Breath Sounds OtherSounds


Fremitus
Effusion Away Dull Decreased Decreased Bronchial sounds
+/- egophony at
edge

ComoMatton Central Dull Increased Bronchial Occasional crackles

Pneumothorax Away Hyper-resonant Decreased Absent Nil

Atelectasis Towards lesion Dull Increased Decreased Nil

Fibrosis Central Resonant (normal) Normal Decreased if Late inspiratory


severe crackles

7. Finish by examining the lymph nodes in the head and neck. Start under the chin with the submental
nodes, move along to the submandibular then to the back of the head at the occipital nodes. Next
palpate the pre and post auricular nodes. Move down the cervical chain and onto the supraclavicular
nodes.
Clinical Examination 71

Central Nervous System Examination

1. Steps before beginning examination

Introduce yourself : I am Dr._______ , your attending physician and Ill be examining you today. At
any point of the examination you feel uncomfortable, please let me know and I'll stop the examination
right there.
Wash/Sanitize hands
Explain to the patient what you are about to do and gain informed consent.
Look for medical equipment/therapies (e.g. walking aids).
Verbalize the steps of the examination and your findings.

2. Cranial Nerve Examination

1) The Olfactory nerve (CN I) is simply tested by offering something familiar for the patient to smell
and identify - for example coffee or vinegar.
2) The Optic nerve (CN II) is tested in five ways:
The acuity is easily tested with Snellen charts. This should be assessed both with the patient
wearing any glasses or contact lenses they usually wear and without them.
Colour vision is tested using Ishara plates, these identify patients who are colour blind.
Visual fields are tested by asking the patient to look direcdy at you and wiggling one of your
fingers in each of the four quadrants. Ask the patient to identify which finger is moving.
Visual inattention can be tested by moving both fingers at the same time and checking the
patient identifies this.
Visual reflexes comprise direct and concentric reflexes. Place one hand vertically along the
nose to block any light from entering the eye not being tested. Shine a pen torch into one eye
and check that the pupils on both sides constrict. This should be tested on both sides.
Finally fundoscopy should be performed on both eyes.
3) Eye movements: Oculomotor nerve (III), Trochlear nerve (IV) and Abducent nerve (VI) are
involved in movements of the eye. Asking the patient to keep their head perfecdy still direcdy in front
of you, you should draw two large joining Hs in front of them using your finger and ask them to
follow your finger with their eyes. It is important the patient does not move their head. Always ask if
the patient experiences any double vision and if so when is it worse. Also look for ptosis and assess
saccadic eye movements.
4) The Trigeminal nerve (CN V) is involved in sensory supply to the face and motor supply to the
muscles of mastication. Initially test the sensory branches by lightly touching the face with a piece of
cotton wool and then with a blunt pin in three places on each side - around the jawline, on the cheek
and on the forehead. The corneal reflex should also be examined as the sensory supply to the cornea is
from this nerve. This is done by lighdy touching the cornea with the cotton wool. This should cause
the patient to shut their eyelids.
For the motor supply, ask the patient to clench their teeth together, observing and feeling the bulk of
the masseter and temporalis muscles. Then ask them to open their mouth against resistance. Finally
perform the jaw jerk on the patient by placing your left index finger on their chin and striking it with
a tendon hammer. This should cause slight protrusion of the jaw.
72 NAC OSCE |A Comprehensive Review

5) The Facial nerve (CN VII) supplies motor branches to the muscles of facial expression. Therefore,
this nerve is tested by asking the patient to crease up their forehead (raise their eyebrows), close their
eyes and keep them closed against resistance, puff out their cheeks and show you their teeth.
6) The Vestibulocochlear nerve (CN VIII) provides innervation to the hearing apparatus of the ear and
can be used to differentiate conductive and sensori-neural hearing loss using the Rinne and Weber
tests. For the Rinne test, place a sounding tuning fork on the patients mastoid process and then next
to their ear and ask which is louder, a normal patient will find the second position louder. For Webers
test, place the tuning fork base down in the centre of the patients forehead and ask if it is louder in
either ear. Normally it should be heard equally in both ears.
7) The Glossopharyngeal nerve (CN IX) provides sensory supply to the palate. It can be tested with the
gag reflex or by touching the arches of the pharynx.
8) The Vagus nerve (CN X) provides motor supply to the pharynx. Asking the patient to speak gives a
good indication to the efficacy of the muscles. You should also observe the uvula before and during
the patient saying aah. Check that it lies centrally and does not deviate on movement.
9) The Accessory nerve (CN XI) gives motor supply to the sternocleidomastoid and trapezius muscles.
To test it, ask the patient to shrug their shoulders and turn their head against resistance.
10) The Hypoglossal nerve (CN XII) provides motor supply to the muscles of the tongue. Observe the
tongue for any signs of wasting or fasciculations. Then ask the patient to stick their tongue out. If the
tongue deviates to either side, it suggests a weakening of the muscles on that side.

3. Cerebellar Examination

Gait:
Ask the patient to stand up. Observe the patient's posture and whether they are steady on their feet.
Ask the patient to walk, e.g. to the other side of the room, and back. If the patient normally uses a
walking aid, allow them to do so.
Observe the different gait components (heel strike, toe lift off). Is the gait shuffling /waddling /
scissoring/ swinging?
Observe the patients arm swing and take note how the patient turns around as this involves good
balance and co-ordination.
Ask the patient to walk heel-to-toe to assess balance.
Perform Rombergs test by asking the patient to stand unaided with his eyes closed. If the patient
sways or loses balance this test is positive. Stand near the patient in case he falls.

Co-ordination:
Look for a resting tremor in the hands.
Test tone in the arms (shoulder, elbow, wrist)
Test for dysdiadochokinesis by showing the patient to clap by alternating the palmar and dorsal
surfaces of the hand. Ask to do this as fast as possible and repeat the test with the other hand.
Perform the finger-to-nose test by placing your index finger about two feet from the patients face. Ask
him to touch the tip of his nose with his index finger then the tip of your finger. Ask him to do this as
fast as possible while you slowly move your finger. Repeat the test with the other hand.
Perform the heel-to-shin test. Have the patient lying down for this and get him to run the heel of one
foot down the shin of the other leg and then to bring the heel back up to the knee and start again.
Repeat the test with the other leg.
Clinical Examination 73

Upper Limb Neurological Examination

1. Steps before beginning examination

Introduce yourself : aI am Dr.________, your attending physician and I'll be examining you today. At
any point of the examination you feel uncomfortable, please let me know and I'll stop the examination
right there.
Wash/Sanitize hands
Explain to the patient what you are about to do and gain informed consent.
Use proper draping techniques, verbalize the steps of the examination and your findings.

2. Inspection
General inspection of patient: general comfort, abnormal posture/movements, muscle wasting.
The upper body should be exposed for this examination. Observe the patient's arms, look for any
muscle wasting, fasciculations or asymmetry.

3. Tone
Examine the tone of the muscles. Start proximally at the shoulder, feeling how easy the joint is to
move passively. Then move down to the elbow, wrist and hand joints again assessing each one's tone in
turn.
Assess for spastic catch, clasp-knife rigidity, led-pipe or cog-wheel rigidity.

4. Power
Next assess the power of each of the muscle groups.
- Shoulder abduction (C5) & Shoulder adduction (C5/C6/C7)
- Elbow flexion (C5/C6) & Elbow extension (C7)
- Wrist flexion (C8) & Wrist extension (C8)
- Finger flexion (C8), Finger abduction (Tl), Finger adduction (Tl)
- Thumb abduction (C8)

5. Reflexes
There are three reflexes in the upper limb - the biceps, triceps and supinator reflexes.
The biceps reflex (C5/C6) is tested by supporting the patient's arm, with it flexed at roughly 60,
placing your thumb over the biceps tendon and hitting your thumb with the tendon hammer. It is
vital to get your patient to relax as much as possible and for you to take the entire weight of their arm.
The triceps reflex (C6/C7) is elicited by resting the patient's arm across their chest and hitting the
triceps tendon just proximal to the elbow.
Finally, with their arm rested on their abdomen, locate the supinator tendon (C5/C6) as it crosses the
radius, place three fingers on it and hit the fingers. This should give the supinator reflex. If you
struggle with any of these reflexes, asking the patient to clench their teeth should exaggerate the
reflex.
74 NAC OSCE |A Comprehensive Review

6. Sensation
This is tested in a number of ways. You should test
light touch, pin prick, vibration and joint position
sense and proprioception.

Ask the patient to place their arms by their sides with


their palms facing forwards. Lighdy touch the patient's
sternum with a piece of cotton wool so that they know
how it feels. Then, with the patient's eyes shut, lightly
touch their arm with the cotton wool. The places to
touch them should test each of the dermatomes. Tell
the patient to say yes every time they feel the cotton
wool as it felt before. Then repeat this using a light pin
prick.
To assess vibration you should use a sounding tuning
fork. Place the fork on the patient's sternum to show
them how it should feel. Then place it on the bony
prominence at the base of their thumb and ask them if
it feels the same. If it does, there is no need to check
any higher. If it feels different you should move to the
radial stylus and then to the olecranon until it feels
normal.
Finally, proprioception. Hold the distal phalanx of the thumb on either side so that you can flex the
interphalangeal joint. Show the patient that when you hold the joint extended, that represents 'Up'
whereas when you hold it flexed that represents 'Down'. Ask the patient to close their eyes and,
having moved the joint a few times hold it in one position - up or down. Ask the patient which
position the joint is in.

7. Coordination
Pronator drift - Ask patient to extend arms in front of them in supination and to close their eyes. A
positive result occurs when the arm falls downwards and pronates (cerebral damage), in cerebellar
lesions the arms may rise.
Assess for dysdiadochokinesia
Assess for finger to nose coordination and intentional tremor.

8. Function is a very important part of any neurological examination as this is the area which will affect
people's day to day lives the most. For upper limb you should ask people to touch their head with both
hands and then ask them to pick up a small object such as a coin which each hand.

9. Finish by thanking the patient and ensuring they are comfortable and well covered.
Clinical Examination 75

Lower Limb Neurological Examination

1. Steps before beginning examination


Introduce yourself : I am Dr._______ , your attending physician and I'll be examining you today. At
any point of the examination you feel uncomfortable, please let me know and Ill stop the examination
right there.
Wash/Sanitize hands
Explain to the patient what you are about to do and gain informed consent.
Verbalize the steps of the examination and your findings.
Make sure patient is adequately exposed, use proper draping techniques

2. Inspection
Observe the patient's legs, look for any muscle wasting, fasciculations or asymmetry.

3. Tone
Start by examining the tone of the muscles. Roll the leg on the bed to see if it moves easily and pull
up on the knee to check its tone. Also check for ankle clonus by placing the patients leg turned
outwards on the bed, moving the ankle joint a few times to relax it and then sharply dorsiflexing it.
Any further movement of the joint may suggest clonus.

4. Power
Next assess the power of each of the muscle groups.
- Hip flexion (L1/L2) & Hip extension (L5/S1)
- Hip abduction (L2/L3) & Hip adduction (L2/L3)
- Knee flexion (L5/S1) & Knee extension (L3/L4)
- Ankle dorsiflexion (L4/L5) & Ankle plantar flexion (S1/S2)
- Big toe flexion (S1/S2)

5. Reflexes
Test the patient's reflexes. There are three reflexes in the lower limb - the knee reflex, the ankle jerk
and the plantar reflex - elicited by stroking up the lateral aspect of the plantar surface.
The knee reflex (L3/L4) is tested by placing the patient's leg flexed at roughly 60, taking the entire
weight of their leg with your arm and hitting the patellar tendon with the tendon hammer. It is vital
to get your patient to relax as much as possible and for you to take the entire weight of their leg.
The ankle jerk (S1/S2) is elicited by resting the patient's leg on the bed with their hip laterally rotated.
Pull the foot into dorsiflexion and hit the calcaneal tendon.
Finally, with their leg out straight and resting on the bed, run the end of the handle of the tendon
hammer along the outside of the foot. This gives the plantar reflex (Si). An abnormal reflex would see
the great toe extending. If you struggle with any of these reflexes, asking the patient to clench their
teeth should exaggerate the reflex.
76 NAC OSCE |A Comprehensive Review

6. Sensation

The final test is sensation. However, this is tested in a number


of ways. You should test light touch, pin prick, vibration and
joint position sense and proprioception.
Ask the patient to place their legs out straight on the bed.
Lightly touch the patient's sternum with a piece of cotton wool
so that they know how it feels. Then, with the patient's eyes
shut, lightly touch their leg with the cotton wool. The places to
touch them should test each of the dermatomes - make sure
you know these! Tell the patient to say yes every time they feel
the cotton wool as it felt before. Then repeat this using a light
pin prick.
To assess vibration you should use a sounding tuning fork.
Place the fork on the patient's sternum to show them how it
should feel. Then place it on their medial malleolus and ask
them if it feels the same. If it does, there is no need to check
any higher. If it feels different you should move to the tibial
epicondyle and then to the greater trochanter until it feels
normal.
Finally, proprioception. Hold the distal phalanx of the great toe
on either side so that you can flex the interphalangeal joint. Anterior
Show the patient that when you hold the joint extended, that represents 'Up' whereas when you hold
it flexed that represents 'Down'. Ask the patient to close their eyes and, having moved the joint a few
times hold it in one position - up or down. Ask the patient which position the joint is in.

7. Function is a very important part of any neurological examination as this is the area which will affect
peoples day to day lives the most. For the lower limb you should assess the patient's walking. Observe
their gait and check for any abnormalities. Whilst they are standing you should perform Romberg's
test. Ask the patient to stand with their feet apart and then close their eyes. Stand next to the patient
in case he falls. Any swaying may be suggestive of a posterior column pathology.

8. Finish by thanking the patient and ensuring they are comfortable and well covered.
Clinical Examination 77

Musculo-skeletal system : Spine/Back


1. Steps before beginning examination
Introduce yourself : aI am Dr.________, your attending physician and I'll be examining you today. At
any point of the examination you feel uncomfortable, please let me know and I'll stop the examination
right there.
Wash/Sanitize hands
Explain to the patient what you are about to do and gain informed consent.
Ensure patient is adequately exposed.
Look for medical equipment/therapies
Show empathy.
Verbalize the steps of the examination and your findings.

2. Inspection
Ask for patients vitals
Observe patient : Is patient sitting comfortably? Gait? Position of comfort.
Observe the patient from behind :
- Pelvic and shoulder symmetry, palpate the pelvic brim to check for symmetry.
- Scoliosis
- Gibbus (dorsal spines abnormally prominent)
Observe patient from side :
- Kyphosis
- Increased lumbar lordosis
Check the spine for SEADS : S: Swelling, E: Erythema, ecchymosis, A: Atrophy/asymmetry (muscle
bulk), D: Deformity, S: Skin changes/scars/bruising

3. Range of Motion
Flexion : In the standing position by asking the patient to touch the toes. Normal - 90 .The normal
spine should lengthen more than 5 cm in the thoracic area and more than 7.5 cm in the lumbar area
on forward flexion.
Extension : Stabilize the patient, ask the patient to bend backwards. Normal - 30.
Lateral flexion : ask the patient to slide their hand straight down the thigh, first on the right and then
on the left, keeping the hips straight.
Observe for restricted movement and loss of symmetry.
Test for facet joint disease : Ask patient to extend their back as far as possible and to rotate (pain
suggests facet joint pathology).

4. Palpation
Examine the back and palpate for areas of muscle spasm and tenderness (paraspinal muscles).
Palpate spinous processes with thumb for tenderness
Sacroiliac joints, sacro iliac dimples, ask for tenderness.
78 NAC OSCE |A Comprehensive Review

5. Ankylosing spondylitis tests


Chest expansion : Measure with a tape measure (should be >5cm)
Schober'sTest : Draw a horizontal line 10cm above and one 5cm below the dimples of Venus (the
distance between these lines should increase to >20cm during lumbar flexion - in ankylosing
spondylitis the distance will not increase to >20cm)
Distance of tragus to wall when patient is standing with their back to the wall (useful for monitoring).

6. Cervical and thoracic movements (patient sitting on edge of bed)


Cervical movements
- Flexion (ask patient to touch chin to chest)
- Extension (ask patient to look to the ceiling as far back as possible)
- Lateral flexion (ask patient to touch their ear to the shoulder keeping the shoulder still)
- Spurling Maneuver : Extend head back & bring ear towards shoulder. Give gentle axial pressure
on the head. If patient complains of pain radiating from head to ipsilateral arm - diagnosis of
Radiculopathy is made.
- Rotation (ask patient to look over the left and right shoulder)
- Perform these movements passively if active movements are restricted.
Thoracic rotation : ask patient to fold their arms and twist around.

7. Tests with patient lying on their back


Straight leg raising test : ask the patient to lie with the spine on the table and to relax completely.
With the knee fully extended, first one leg and then the other is slowly lifted and flexed at the hip.
This produces stretch on the sciatic nerve, at which point sciatic pain is produced. If this maneuver
produces pain in the hip or low back with radiation in the sciatic area, the test is considered positive
for nerve root irritation. The angle of elevation of the leg from the table at the point where pain is
produced should be recorded.
FABER (Flexion Abduction External Rotation) : Ask the patient to lie supine on the exam table.
Place the foot of the affected side on the opposite knee. Pain in the groin area indicates a problem
with the hip and not the spine. Press down gently but firmly on the flexed knee and the opposite
anterior superior iliac crest. Pain in the sacroiliac area indicates a problem with the sacroiliac joints.
Bowstring test: Once the level of pain has been reached, flex the knee slighdy and apply firm pressure
with the thumb in the popliteal fossa over the stretched tibial nerve. Radiating pain and paraesthesiae
suggest nerve root irritation.

8. Tests with patient lying on their abdomen


Lasegue's sign: With the patient supine and hip flexed, dorsiflexion of the ankle causes pain or muscle
spasm in the posterior thigh if there is lumbar root or sciatic nerve irritation.
Femoral stretch test:With the patient prone and the anterior thigh fixed to the couch, flex each knee
in turn. This causes pain in the appropriate distributions by stretching the femoral nerve roots in L2-
L4.The pain produced is normally aggravated by extension of the hip. The test is positive if pain is
felt in the anterior compartment of thigh.
Clinical Examination 79

Hip Examination

1. Steps before beginning examination


Introduce yourself : I am Dr.________, your attending physician and I'll be examining you today. At
any point of the examination you feel uncomfortable, please let me know and I'll stop the examination
right there.
Wash/Sanitize hands
Explain to the patient what you are about to do and gain informed consent.
Ensure patient is adequately exposed.
Look for medical equipment/therapies
Ask which hip is painful, show empathy.
Verbalize the steps of the examination and your findings.

2. Inspection of hip (with patient standing up)


While the patient is standing, check the hip for SEADS : S: Swelling, E: Erythema, ecchymosis,
A: Atrophy/asymmetry (muscle bulk), D: Deformity, S: Skin changes (erythema/scars/abscess/sinuses)
Leg length discrepancy
Whilst the patient is still standing, perform the Trendelenberg test. This is done by asking the patient
to alternately stand on one leg. Stand behind the patient and feel the pelvis. It should remain at level
or rise slightly. If the pelvis drops markedly on the side of the raised leg, then it suggests abductor
muscle weakness on the leg the patient is standing on.

3. Gait - ask patient to walk across the floor. Look for any abnormalities, wresoFGAfr
Antalgic ~ Trauma, OA
hip, knee, foot movements, length of stride. Trendelenberg - weakness
of hip adductors
Festlnating - Parkinson's ds.
High stepping - Polio, MS
3. Inspection & Palpation of hip (with patient lying down) Scissor - Spastic cerebral
palsy
Inspection for hip and groin swellings (hernia, lymphadenopathy, Stomping - Friedreich's
saphenous varix, effusion) ataxia, tabes dorsalis
Inspect for obvious fixed flexion Spastc - Brain tumor, sturge
Weber's, cerebral palsy
Palpate anterior hip for lumps and tenderness.
Palpate the greater trochanter for any tenderness which might
suggest trochanteric bursitis.

4. Leg-length difference
Make an approximate judgment by aligning the medial malleoli and looking for discrepancy.
Measure true and apparent leg-length if appropriate. True leg length discrepancy is found by
measuring from the anterior superior iliac spine to the medial malleolus. Apparent leg length
discrepancy is measured from the umbilicus to the medial malleolus.
80 NAC OSCE |A Comprehensive Review

5. Active and passive movements


Assess active flexion, extension, abduction and adduction.
- Flexion : Flex the knee to 90 degrees and passively flex the hip by pushing the knee towards the
chest.
- Extension : is performed by placing your hand under the patients ankle and asking them to push
your hand into the bed.
Passively assess internal and external rotation of the hip (with hips at 90 flexion)
- Internal rotation : performed with the knee flexed and by everting the knee for internal rotation
- External rotation : performed with the knee flexed and inverting it for external rotation.

6. Special tests
Thomas test : Place your hand under the patient's lumbar spine to stop any lumbar movements and
fully flex one of the hips. Observe the other hip, if it lifts off the couch then it suggests a fixed flexion
deformity of that hip.

FABER (Flexion Abduction External Rotation) : Ask the patient to lie supine on the exam table.
Place the foot of the affected side on the opposite knee. Pain in the groin area indicates a problem
with the hip and not the spine. Press down gendy but firmly on the flexed knee and the opposite
anterior superior iliac crest. Pain in the sacroiliac area indicates a problem with the sacroiliac joints.
Clinical Examination 81

Knee Examination

1. Steps before beginning examination


Introduce yourself : I am Dr.________, your attending physician and I'll be examining you today. At
any point of the examination you feel uncomfortable, please let me know and I'll stop the examination
right there.
Wash/Sanitize hands
Explain to the patient what you are about to do and gain informed consent.
Ensure patient is adequately exposed (up to above knees).
Look for medical equipment/therapies
Ask about knee locking, giving way and pain, show empathy.
Verbalize the steps of the examination and your findings.

2. Inspection
Gait : Ask the patient to walk for you. Observe any limp or obvious deformities such as scars or
muscle wasting. Check if the patient has a varus (bow-legged) or valgus (knock-knees) deformity.
Also observe from behind to see if there are any obvious popliteal swellings such as a Baker's cyst.
While the patient is lying on the bed, make a general observation. Look for symmetry, redness, muscle
wasting, scars, rashes or fixed flexion deformities.

3. Palpation
Check the temperature using the backs of your hands, comparing it with other parts of the leg.
Palpate the border of the patella for any tenderness, behind the knee for any swellings, along all of the
joint lines for tenderness and at the point of insertion of the patellar tendon. Finally, tap the patella to
see if there is any effusion deep to the patella.
Landmarks of the knee : Tibial tuberosity, patellar tendon, quadriceps tendon, medial and lateral
femoral condyles.
Peripatellar area : push patella medially and rub right underneath the medial facet of patella and look
for tenderness ( Patellar - femoral stress S).
Joint line tenderness : bend the knee 90, palpate medial and lateral joint line.
Patella apprehension test - Move patella around and observe patient's face for pain.

4. Range of Motion
Active flexion and extension of knee - Observe for restricted movement and for displacement of
patella.
Passive flexion and extension of knee - feel for crepitus.
Straight leg raise - assessment of extensor apparatus.
82 NAC OSCE |A Comprehensive Review

5. Special tests

Tests to Detect Arthritis:


Crepitus: Crepitus is the sensation that is felt when rough cartilage or exposed bone is rubbing as the
knee is bent. The examiner will feel, and may hear, this grinding as the knee is bent back and forth.
Deformity: As the knee cartilage is worn away, the knees may become progressively knock-kneed or
bow-legged.
Limited Motion: The range of motion of the knee typically becomes limited if arthritis, bone spurs,
and swelling prevents normal mobility.

Tests to Detect a Tom Meniscus:


Joint Line Tenderness
Joint line tenderness is a very non-specific test for a meniscus tear. The area of the meniscus is felt,
and a positive test is considered when there is pain in this area.
McMurrays Test
McMurrays test is performed with the patient lying flat on his back and the examiner bending the
knee. A click is felt over the meniscus tear as the knee is brought from full flexion to full extension.

Tests to Detect an ACL Tear:


Lachman Test
The Lachman test is the best test to diagnose an ACL tear. With the knee slighdy bent, the examiner
stabilizes the thigh while pulling the shin forward. A tom ACL allows the shin to shift too far
forward.
Anterior Drawer Test
This test is also performed with the patient lying flat on his back. The knee is bent 90 degrees and the
shin is pulled forward to check the stability of the ACL.

Tests to Detect Other Ligament Injuries:


Posterior Drawer Test
The posterior drawer is performed similarly to the anterior drawer test. This test detects injury to the
PCL. By pushing the shin backward, the function of the PCL is tested.
Collateral Ligament Stability
Side-to-side stability of the knee detects problems of the collateral ligaments, the MCL and LCL.
With the patient lying flat, and the knee held slightly bent, the shin is shifted to each side. Damage to
the LCL or MCL will allow the knee to "open up" excessively, a problem called varus (LCL) or valgus
(MCL) instability.

Tests to Detect Kneecap Problems:


Patellar Grind
The patient lies supine with the leg extended. The examiner reproduces the patient's knee pain by
pushing the kneecap down and asking the patient to flex his thigh muscles. Damaged cartilage can
cause a grinding sensation called crepitus.
Patellar Tenderness
The examiner can slightly lift up the kneecap and place direct pressure on the under surface of the
kneecap. By doing so, the examiner is looking for sensitive regions of cartilage.
Patellar Apprehension
This is a sign of an unstable kneecap. While the examiner places pressure on the kneecap, the patient
may complain of the sensation that the kneecap is going to 'pop out' of its groove.
Clinical Examination 83

Foot and Ankle Examination

1. Steps before beginning examination

Introduce yourself : I am Dr._______ , your attending physician and I'll be examining you today. At
any point of the examination you feel uncomfortable, please let me know and I'll stop the examination
right there.
Wash/Sanitize hands
Explain to the patient what you are about to do and gain informed consent.
Ensure patient is adequately exposed (up to above knees).
Look for medical equipment/therapies
Ask if patient is able to bear weight, show empathy.
Verbalize the steps of the examination and your findings.

2. Inspection

Gait : watch the patient walk, observing for a normal heel strike, toe-off gait. Also look at the
alignment of the toes for any valgus or varus deformities. Assess ability to weight-bear on affected
side.
While patient is standing check the foot arches checking for pes cavus (high arches) or pes planus
(flat feet).
Inspection of the foot with patient sitting and feet overhanging
- Check the foot and ankle for SEADS : S: Swelling, E: Erythema, ecchymosis,
A: Atrophy/asymmetry (muscle bulk), D: Deformity, S: Skin changes/scars/bruising.
- Check the symmetry, nails (psoriasis), skin, toe alignment, look for toe clawing, joint swelling and
plantar and dorsal calluses.
Finally you should look at the patients shoes, note any uneven wear on either sole and the presence of
any insoles.

3. Palpation of ankle/foot

Feel each foot for temperature, comparing it to the temperature of the rest of the leg.
Feel for distal pulses.
Squeeze over the metatarsophalangeal joints observing the patient's face for any pain.
Palpate over the midfoot, ankle and subtalar joint lines for any tenderness. Feel the Achilles tendon
for any thickening or swelling. Palpate medial and lateral malleoli for any tenderness.
84 NAC OSCE ) A Comprehensive Review

4. Range of Motion

Assess all active and passive movements of the foot. These movements are inversion, eversion,
dorsiflexion and plantarflexion.
- Subtalar joint - inversion and eversion
- Ankle joint - dorsiflexion and plantar flexion
- Big toe - dorsiflexion and plantar flexion
- Mid-tarsal joints - which are tested by fixing the ankle with one foot and inverting and everting
the forefoot with the other.

5. Special tests

Ankle Anterior Drawer Test - assesses for lateral ankle sprain


Patient is seated, stabilize the tibia with one hand while grasping heel and pulling it anteriorly with
the other. Greater than 3 mm anterior movement may be significant. 1 cm is significant and indicates
anterior talofibular ligament rupture.
Positive Test - laxity in the ligament with exaggerated anterior translation
Talar Tilt Test - assesses integrity of the deltoid ligament/lateral ankle sprain
Patient is seated, stabilize the leg and foot while adducting and inverting the calcaneus apply a varus
force. The calcaneus is then abducted and everted applying a valgus force.
Positive Test - pain or laxity in the ligament
Thompsons Test - assesses for Achilles' tendon rupture
Patient is prone, squeeze the gastrocnemius and soleus muscles while noting any movement at the
ankle and foot
Positive Test - no movement or plantarflexion at all indicates a 3rd degree strain of the Achilles'
tendon
Plantar Fasciitis Test - assesses for inflammation of the plantar fascia
Patient is supine, dorsiflex the ankle and extends all toes then press in the medial border of the plantar
fascia
Positive Test - pain is consistent with plantar fasciitis

Ottawa Ankle rules


For taking ankle series x-rays(AP and lateral ankle)
X-ray if there is pain over the malleolar zone AND tenderness on palpation of the medial/lateral
malleolar tip and posterior aspect of medial/lateral malleolus OR
Patient unable to bear weight immediately and in ER.

Forfoot series (AP and Lateralfoot)


X-ray if there is pain in midfoot zone AND bony tenderness over the navicular or base of 5th
metatarsal OR
Unable to bear weight immediately and in ER
Clinical Examination 85

Shoulder Examination

1. Steps before beginning examination

Introduce yourself : I am Dr.________, your attending physician and I'll be examining you today. At
any point of the examination you feel uncomfortable, please let me know and I'll stop the examination
right there.
Wash/Sanitize hands
Explain to the patient what you are about to do and gain informed consent.
Look for medical equipment/therapies, ensure patient is adequately exposed.
Ask which shoulder is painful. Verbalize the steps of the examination and your findings.

2. Inspection

Start by exposing the joint and observe the shoulder joint looking from the back, side and front for
any scars, deformities or muscle wasting (SEADS). Also compare both sides for symmetry.
With the patient standing, ask the patient to place their hands behind their head and behind their
back and observe for and deformities.

3. Palpation

Feel over the joint and its surrounding areas for the temperature of the joint as raised temperature may
suggest inflammation or infection in the joint.
Systematically feel along both sides of the bony shoulder girdle. Start at the sternoclavicular joint,
work along the clavicle to the acromioclavicular joint
Feel the acromion and then around the spine of the scapula.
Feel the anterior and posterior joint lines of the glenohumeral joint and finally the muscles around the
joint for any tenderness.

4. Range of Motion

The movements of the joint should start being performed actively.


Ask the patient to bring their arm forward (flexion), bend their arm at the elbow and push backwards
(extension),
Bring their arm out to the side and up above their head (abduction). When testing adduction perform
the scarf test (The scarf test is performed with the elbow flexed to 90 degrees, placing the patient's
hand on their opposite shoulder and pushing back, again look for any discomfort. )
Rotation
- Internal rotation : Ask the patient to place hands in the small of their back, and slide them up the
back as far as possible.
- External rotation : Ask patient to rotate their arms outwards, keeping the elbows flexed and by
the side of the body.
Once all of these movements have been performed actively, you should perform them passively and
feel for any crepitus whilst moving the joints.
86 NAC OSCE |A Comprehensive Review

5. Special Tests

Tests for Rotator Cuff


i. Supraspinatus
Empty Can Test ( tests integrity of Supraspinatus) : The patient stands with arms extended at the
elbows and abducted in the scapular plane and with thumbs pointed to the floor. The examiner applies
downward pressure to the arms and the patient attempts to resist.
Positive test : Pain, muscle weakness or both.
Apley's Scratch Test- Reach over shoulder to "scratch" between scapula. Measure to which vertebrae
thumb can reach.
ii. Infraspinatus
External Rotation Lag Sign : The elbow is passively flexed to 90 degrees, and the shoulder is held at
20 degrees abduction (in the scapular plane) and near maximal external rotation by the examiner. The
patient is then asked to actively maintain the position of external rotation in abduction as the
examiner releases the wrist while maintaining support of the limb at the elbow. The sign is positive
when a lag, or angular drop occurs. The magnitude of the lag is recorded to the nearest 5 degrees.
iii. Subscapularis
Gerber Lift-OfFTest : With the patients hand on the small of the back, the arm is extended and
internally rotated. The examiner then passively lifts the hand off the small of the back, placing the
arm in maximal internal rotation. The examiner then releases the hand. If the hand falls onto the
back because the subscapularis is unable to maintain internal rotation, the test result is positive.
Patients with subscapularis tears have an increase in passive external rotation and a weakened ability
to resist internal rotation.

Tests for Shoulder Instability


Apprehension Sign for Anterior Instability : The test is performed by abducting the shoulder to 90
degrees, and then slowly externally rotating the shoulder toward 90 degrees. A patient with anterior-
inferior instability will usually become "apprehensive" either verbally or with distressing facial
expressions.

Tests for Subacromial Impingement


Neer Impingement Sign : Place one hand on the posterior aspect of the scapula to stabilize the
shoulder girdle, and, with the other hand, take the patient's internally rotated arm by the wrist, and
place it in full forward flexion.If there is impingement, the patient will report pain in the range of 70
degrees to 120 degrees of forward flexion as the rotator cuff comes into contact with the rigid
coracoacromial arch.
Hawkins Impingement Sign : The examiner places the patient's arm in 90 degrees of forward flexion
and forcefully internally rotates the arm, bringing the greater tuberosity in contact with the lateral
acromion. A positive result is indicated if pain is reproduced during the forced internal rotation.

Tests for Long Head of the Biceps


Speed's Maneuver : The patient's elbow is flexed 20 degrees to 30 degrees with the forearm in
supination and the arm in about 60 degrees of flexion. The examiner resists forward flexion of the
arm while palpating the patient's biceps tendon over the anterior aspect of the shoulder.
Yergason test : The patient's elbow is flexed to 90 degrees with the thumb up. Forearm is in neutral.
The examiner grasps the wrist, resisting attempts by the patient to actively supinate the forearm and
flex the elbow. Pain suggests biceps tendonitis.
Clinical Examination 87

Elbow Examination

1. Steps before beginning examination

Introduce yourself : I am Dr._______ , your attending physician and I'll be examining you today. At
any point of the examination you feel uncomfortable, please let me know and I'll stop the examination
right there.
Wash/Sanitize hands
Explain to the patient what you are about to do and gain informed consent.
Look for medical equipment/therapies, ensure patient is adequately exposed.
Verbalize the steps of the examination and your findings.

2. Inspection

SEADS : S: Swelling, E: Erythema, ecchymosis, A: Atrophy/asymmetry (muscle bulk), D: Deformity,


S: Skin changes/scars/bruising. Check for a fixed flexion deformity. Look at the underside of the
elbows to check for any psoriatic plaques, cysts or rheumatoid nodules.

3. Palpation

Feel the elbow, assessing the joint temperature relative to the rest of the arm.
Palpate the olecranon process as well as the lateral and medial epicondyles for tenderness (medial for
golfers elbow and lateral for tennis elbow), and cubital fossa for tenderness.
Palpate joint line with elbow flexed to 90 for tenderness and swelling.

4. Range of Motion

The movements at the elbow joint are all fairly easy to describe and assess. These are flexion,
extension, pronation and supination. Once these have been assessed actively they should be checked
passively checking for power and crepitus.
Test for varus /valgus instability.

5. Neurological Examination of hand


Motor Sensory
- Median nerve (thumb abduction) - Median nerve (pulp of index finger)
- Radial nerve (wrist extension) - Radial nerve (1st dorsal interosseous space)
- Ulnar nerve (finger abduction) - Ulnar nerve (pulp of 5thfinger)

6. Special Tests

Tennis Elbow : Tennis elbow localises pain over the lateral epicondyle, particularly on active extension
of the wrist with the elbow bent.

Golfer's Elbow : Golfer's elbow pain localises over the medial epicondyle and is made worse by
flexing the wrist.
88 NAC OSCE |A Comprehensive Review

Hand and Wrist Examination

1. Steps before beginning examination

Introduce yourself : I am Dr.________, your attending physician and I'll be examining you today. At
any point of the examination you feel uncomfortable, please let me know and I'll stop the examination
right there.
Wash/Sanitize hands
Explain to the patient what you are about to do and gain informed consent.
Look for medical equipment/therapies
Verbalize the steps of the examination and your findings.

2. Inspection

Inspect hands :
- Skin (rashes, Gottron's patches, nodules, Raynaud's phenomenon, sclerodactyly, scars, skin
atrophy)
- Nails (pitting, onycholysis, splinter haemorrhages, clubbing)
- Muscles (swelling, wasting)
- Joints (swellings, subluxation /deviation of wrist, swan neck /Boutoniere's deformity,
Heberden's/Bouchard's nodes, Z deformity of thumb)
- Inspect palm (palmar erythema, pallor, cyanosis), muscle wasting.
Inspect elbows :
- Psoriatic skin lesions
- Rheumatoid nodules
- Scars

3. Palpation

Assess the temperature over the joint areas and compare these with the temperature of the forearm.
Start proximally and work towards the fingers, feeling the radial pulses and the wrist joints. Then feel
the muscle bulk in the thenar and hypothenar eminences. In the palms, feel for any tendon thickening
and assess the sensation over the relevant areas supplied by the radial, ulnar and median nerves.
Squeeze over the row of metacarpophalangeal joints whilst watching the patient's face for any
discomfort.
Bi-manually palpate MCP and interphalangeal joints.
Clinical Examination 89

4. Range of Motion

Ask the patient to perform the following movements in the sequence mentioned below and observe
for range of movement :
- Make a fist
- Pronate wrist
- Extend little finger (extensor digiti minimi is usually the first tendon to rupture in rheuatoid
arthritis)
- Extend all fingers
Assess function
- Pinch grip
- Opposition (touch thumb to each finger)
- Power grip (ask patient to squeeze your fingers)
- Froment's test (for ulnar nerve palsy). In this test the patient attempts to grip a paper with thumb
and index finger while the examiner tries to pull the paper out of the patient's grip.
- Ask patient to write something /undo a button.
Assess power
- Wrist extension (radial nerve)
- Thumb abduction (median nerve)
- Finger abduction (ulnar nerve)

5. Neurovascular Examination

Nerve Sensation Motor


Median Lateral portions of the pulp of the Resisted palmar abduction of the
index and middle fingers thumb
Ulnar Lateral pulp areas of the little finger Abduction of the fingers against
resistance
Radial Web space between the thumb and Wrist extension
index finger (anatomical snuff box)

6. Special Tests

Phalen's test : Forced flexion of the wrist, either against the other hand or by the examiner for 60
seconds will recreate the symptoms of carpal tunnel syndrome.

Finkelsteins test is used to diagnose DeQuervain's tenosynovitis. Patient is told to flex the thumb
and clench the fist over the thumb followed by ulnar deviation. If there is an increased pain in the
radial styloid process and along the length of the extensor pollicis brevis and abductor pollicis longus
tendons, then the test is positive for De Quervains syndrome.

Tinel's sign : Use the index finger to tap over the carpal tunnel at the wrist. A positive test results
when the tapping causes tingling or paresthesia in the area of the median nerve distribution, which
includes the thumb, index finger, and middle and lateral half of the ring finger. A positive Tinel's sign
at the wrist indicates carpal tunnel syndrome.
90 NAC OSCE |A Comprehensive Review

Breast Examination

Introduce yourself : I am Dr._______ , your attending physician and I'll be examining you today. At
any point of the examination you feel uncomfortable, please let me know and I'll stop the examination
right there.
Wash/Sanitize hands
Explain to the patient what you are about to do and gain informed consent.
Verbalize the steps of the examination and your findings. Ask which side the problem is.
Make sure patient is adequately exposed, use proper draping techniques

1. General Inspection (with patient sitting on side of bed)

Inspect with :
- Patient's arm by their sides.
- Patients arms behind their head (tenses skin)
- Patient's hands on their hips (tenses pectoralis major)
These manoeuvers test forT4 disease - invasion of chest wall / skin. Inspect for :
- Obvious masses
- Scars
- Radiotherapy tattoos
- Skin changes
- Peau d'orange
- Dimpling
- Nipple retraction
- Paget's disease.

2. Inspection (with patient lying down)

- Breasts size, symmetry and contour.


- Areola pigmentation, nipple pigmentation, shape, ulceration and discharge.
- Skin color, thckening, venous pattern and edema.

3. Palpate
- Palpate normal breast followed by abnormal breast.
- Palpate all quadrants, nipple and axillary tail of each breast.
- Describe any masses : position, size shape, mobility, number, tenderness, consistency.
- Palpate axillary, supraclavicular and infraclavicular lymph nodes.

4. Auscultate lungs.
Clinical Examination 91

Thyroid Examination

1. Steps before beginning examination

Introduce yourself : I am Dr._______ , your attending physician and I'll be examining you today. At
any point of the examination you feel uncomfortable, please let me know and I'll stop the examination
right there.
Wash/Sanitize hands
Explain to the patient what you are about to do and gain informed consent.
Ensure patient is adequately exposed.
Look for medical equipment/therapies
Show empathy.
Verbalize the steps of the examination and your findings.

2. Inspection

Ask for patients vitals.

Observe patient : Is patient anxious? Weight gain/loss? Note hoarseness of voice.

Feel pulse - rate/rhythm/volume

Face : Facial expression ( dull in hypothyroidism)


Periorbital myxedema
Loss of l/3rd of eyebrows
Hair - texture/alopecia
Exophthalmos (look from behind patient), lid lag
Ophthalmoplegia (ask patient to follow your finger then ask for diplopia)
Chemosis (redness and watering of eyes)
Hands : Temperature, moist palms, texture, color.
Assess for fine tremors, palmar eythema
Pemberton's sign - is the development of facial flushing, distended neck and head superficial
veins, inspiratory stridor and elevation of the jugular venous pressure (JVP) upon raising of
the patients both arms above his/her head simultaneously. (Thoracic inlet obstruction
- e.g. due to goitre)
Carpal Tunnel Syndrome (Tinels and Phalen's Test) - associated with hypothyroidism.
Arm reflexes - brisk in hyperthyroidism.
Neck : Stand in front of the patient, inspect for neck swellings/goitre
- scars, sinuses, dilated vessels.
92 NAC OSCE |A Comprehensive Review

3. Examination of the Thyroid gland and cervical lymph nodes.

Swallow tests - Ask patient to swallow water and observe for movement of any masses.
Tongue protrusion - Thyroglossal cyst moves on tongue protrusion.
Stand behind the patient and palpate. Assess size, texture, smoothness, margins and mobility of the
thyroid gland (including when swallowing). Note the temperature over gland and adjacent skin.
Palpate cervical lymph nodes.
Percuss over sternum - Retrosternal goitre.
Auscultate for thyroid bruit - Grave's disease.

4. Examination of legs.

Prtibial myxoedema
Peripheral edema due to congestive cardiac failure.
Delayed relaxation of ankle reflex in hypothyroidism.

5. Thank the patient after the examination.


Clinical Examination 93

Mini Mental State Examination

1. ORIENTATION Maximum score = 10


What is today's date? 1
What is the year? 1
What is the month? 1
What is the day today? 1
Can you tell me what season it is? 1
Can you also tell me the name of the location we are in? (Hospital/clinic) 1
What floor are we on? 1
What city are we in? 1
What country are we in? 1
What state are we in? 1
II. IMMEDIATE RECALL Maximum score = 3
Ask the patient if you may test his/her Ball 1
memory. Say the words "ball" , "flag" ,
Flag 1
"tree" clearly and slowly. Then ask the
patient to repeat the words. Check for each Tree 1
correct response. The first repetition
determines the score. If the patient does
not repeat all three correctly, keep saying
them up to six tries until the patient can Number of Trials :__
repeat them.
III. ATTENTION AND
CALCULATION
A. Counting Backwards Test Record each response Maximum score = 5
Ask the patient to begin with 100 93 1
and count backwards by 7. Record
each response. Check one box at 86 1
right for each correct response. The 79 1
score is the number of correct
subtractions. 72 1
65 1
B. Spelling Backwards Test
Ask the patient to spell the word D 1
"WORLD" backwards. Record each
response. Use the instructions to L 1
determine which are correct
responses, and check one box at R 1
right for each correct response.
0 1

W 1
94 NAC OSCE |A Comprehensive Review

C. Final Score
Compare the scores of the Counting
Backwards and Spelling Backwards Final Score :
tests. Write the greater of the two (Max of 5 or Greater of
scores in the box labeled FINAL the two scores)
SCORE at right, and use it in
deriving the TOTAL SCORE.
IV. RECALL Maximum score = 3
Ask the patient to recall the three Ball 1
words you previously asked him/her
to remember. Check the Box at right Flag 1
for each correct response. Tree 1
V. LANGUAGE Maximum score = 9
Naming Watch 1
Show the patient a wrist watch and ask Pencil 1
him/her what it is. Repeat for a pencil.
Repetition
Ask the patient to repeat "No ifs, ands, or Correct repetition 1
buts."
Three - Stage Command
Establish the patient's dominant hand. Give Takes paper in hand 1
the patient a sheet of blank paper and say,
Folds paper in half 1
'Take the paper in your right/left hand, fold
it in half and put it on the floor." Puts paper on the floor 1
Reading
Hold up the card that reads, "Close your
eyes." So the patient can see it clearly. Ask
him/her to read it and do what it says. Closes eyes 1
Check the box at right only if he/she
actually closes his/her eyes.
Writing
Give the patient a sheet of blank paper and
ask him/her to write a sentence. It is to be
written spontaneously. If the sentence Writes sentence 1
contains a patient and a verb, and is
sensible, check the box at right. Correct
grammar and punctuation are not
necessary.
Copying
Show the patient the drawing of the
intersecting pentagons. Ask him/her to
draw the pentagons (about one inch each Copies pentagons 1
side) on the paper provided. If ten angles
are present and two intersect, check the
box at right. Ignore tremor and rotation.
Clinical Examination 95

""i

DERIVING THE TOTAL SCORE

Add the number of correct responses. The maximux is 30.

23-30 Normal
23-19 Borderline
Less than 19 Impaired
This is a blank page
Clinical Cases
This is a blank page
Clinical Cases - Protocol for history taking 99

Protocol for history taking

A candidate gets 2 minutes outside the station to read the clinical case senario on the door before entering. It is
essential to get yourself organised in these 2 minutes.

Read the question properly, understand the requirement and follow instructions (e.g. if you are asked
to do a physical examination, do not start taking history. You will be losing valuable time)

You will be given a pencil and a booklet with blank pages. It is a good practice to jot down notes.
Write the name, age, sex and chief complaint of the patient.
For history of present illness, you can use the mnemonic OCDPQRSTUV+AAA.
Past and Social History : PAM HUGS FOSS
Write down your differential diagnosis.

So your note should look something like this :

Name of the patient, age, sex and chief complaints

O - Onset P - Past medical history


C - Course A - Allergies
D - Duration M - Medication
P - Place H - Hospitalizations
Q_- Quality U - Urinary problems
R - Radiation G - GIT problems
S - Severity S - Sleep
T - Timing F - Family history
U - You (associated with your daily activities) O - Obstetrical history
V - Deja vu (Has it happened before?) S - Social history
A - Aggravating factors S - Sexual history
A - Alleviating factors
A - Associated symptoms
Differential diagnosis :

Knock the door before entering, relax, take a deep breath, smile and enter the room with confidence.
Hand over the stickers to the physician examiner.

Introduce yourself to the patient : e.g. Good morning/afternoon/evening, Mr./Miss______, I am


Dr._____and Im your attending physician today.
So, Mr./Miss_____, what complaint has brought you here today?
Show empathy : Oh! I am sorry to hear that, I shall try my best to help you.
100 NAC OSCE |A Comprehensive Review

Data Collection : OCDPQRSTUV+AAA

Onset : When did it start? Sudden? Gradual?


Course : Is it getting worse, better or just the same?
Duration : You said it started...... ago, does it come and go? How often does it come? For how long
does it stay each time?
Place : Show me exactly where it hurts?
Quality : Tell me how it feels like? Is it sharp/burning/dull/crampy?
Radiation : Does it travel to anywhere?
Severity : On a scale from 1 to 10, with 1 being the mildest and 10 the worst pain, how bad is it?
Timing : Is it worse at a particular time of the day?
U (You) your daily activities : Does it interfere in your day to day activities? Does it change with your
daily activities like posture, rest, eating, exertion?
V (Deja vu) : Has it happened before? When? What happened then? What medication?

Aggravating factors : What makes it worse?


Alleviating factors : What make it better?
Associated symptoms : Have you noticed anything that occurs with it?
Al : Associated constitutional symptoms like fever, shortness of breath, cough, nausea,
vomiting, diarrhea, headache, fatigue (FSC NVD HF)
A2 : Associated symptoms to particular system
Respiratory : Chest pain, shortness of breath, cough, sputum, wheezing, runny nose, post
nasal drip, contact with ill person, night sweats, questions for pulmonary embolism (leg pain,
long travel, surgery and OCP use in females)
CVS : chest pain, orthopnea, paroxysmal nocturnal dyspnea, palpitations, tachycardia.
GIT : pain in abdomen, stool, bowel movements, vomiting, jaundice, blood in stools, diet,
travel.
Neurology : headache, loss of consciousness, weakness, paresthesias
Higher Mental Function : orientation, memory, consciousness.
Motor : weakness of limbs.
Sensory : tingling sensation.
Cerebellum : gait, balance.
Cranial Nerves : speech, swallowing, vision, hearing.

PAM HUGS FOSS

Ok, Mr./Miss_____, Now I need to ask you about your health in general. Is that okay with you?

Past Medical History : What other medical problems do you have? (Diabetes/Hypertension/Asthma /
Cancer?)

Allergies : Do you have any allergies? Are you allergic to any drugs?
Clinical Cases - Protocol for history taking 101

Medication : Do you take medicines at present?

Hospitalizations : (medical/surgical/trauma)

Urinary problem : burning sensation while passing urine? Blood in urine?

GIT : bowel movements? Loose stools? Constipation? Blood in stools?

Sleep?

lam going to ask you a few personal questions that will help me in my diagnosis. Is that okay with
you? Let me begin by asking you about your family health.

Family history : similar complaints in the family? Cancer in the family? Depression? Suicide?

Obstetrical History : When was your last pap smear? Wast it normal? Any history of STIs?

Now I need to ask you about your sexual health. Whatever you tell me will be kept confidential. Is
that okay with you?

Sexual History : Are you in a physical relationship? How long?


Do you practice safe sex?
Any risk of STIs?

Social History : Smoking, how many packs? Alcohol, amount?


T :Treatment history
Recreational drugs?
R: Route
A :Addictlon/toxicity/amount
Is there anything else that you want to share? P: Pattern of use
P: Prior abstinence
E: Effects of the drug
WRAP UP
D: Duration of use
102 NAC OSCE |A Comprehensive Review

Tammy Robbins, a 48 years old ladypresented with heart racing and chest discomfort for the past 3 days.
Take a focused history and perform focused physical examination.
Vitals: BP - 90/70 mm Hg, HR - 146/min, irregular, RR - 12/min,Temp - 37.5C

Clinical Info: Ms Tammy Robbins is a known hypertensive with CAD for the past 10 years, who presented
with sudden onset of palpitations and chest discomfort for the past 3 days. Her symptoms are worsening for
the past 24 hours. She has dyspnea. She has dizziness for the past 12 hours. Pedal edema is 2 +. She had 2
vessel angioplasty done 5 years ago. ECG shows absent P waves with irregular narrow QRS complexes.
Bilateral basal rales present on lung auscultation.
Clinical Case : Atrial Fibrillation (examination on page 65)

HOPI Past History


OCD PQRST UV + AAA Do you have any medical illnesses?
How did it start? Sudden or gradual. Are you allergic to any medications?
Is it getting worse/better or no changes in Any surgeries in the past?
the symptoms? Past h/o recurrent infections?
Duration of palpitations?
Associated with chest discomfort? Family and Social History
Describe the type of chest discomfort? Do you smoke? Duration & frequency.
Do you have chest pain? Do you consume alcohol? Duration oc
Any shortness of breath? frequency.
Any dizziness/light headedness? Do you take any recreational drugs?
Any fever/cough/nausea/vomiting? Any family history of cancers/ medical
Any hemoptysis? illnesses?
Any chills/night sweats?
Any malaise/fatigue/weakness?
Any swelling of feet?
Any numbness/paresthesias?
Any visual problems?
Any relieving factors?
Any aggravating factors?
Any recent trauma?
List of current medications/compliance?

Differential Diagnosis Management


Atrial fibrillation secondary to: Treat the primary cause.
1. Congestive heart failure. Admit in cardiac care unit.
2. Ischemic heart disease. Rate control by beta blockers, calcium
3. Hypertension. channel blockers or digoxin.
4. Thyroid disease. Anticoagulation with heparin, then warfarin.
Rhythm control by electro or medical
Investigations cardioversion.
CBC, electrolytes, glucose. Assess Stroke risk by using CHADS2 score.
LFT, RFT,TSH.
CK, LDH, Cardiac enzymes.
12 lead ECG.
Echocardiogram.
Chest X ray.
Clinical Cases - Medicine 103

Simon Charles, a 20 years old male presented to your clinic with shortness of breath for the past 24 hours.
Take a focused history and perform focused physical examination.
Vitals: BP - 110/80 mm Hg, HR - 110/min, RR - 22/min,Temp - 37.5C.

Clinical Info: Mr Simon Charles has a h/o of Asthma since the past 10 years. He recently cleaned his
basement 1 day ago and his asthma symptoms exacerbated. He is having wheezing, chest tightness,cough and
SOB. He is currently on inhalers with no night symptoms. On examination, he has dyspnea and wheezing
present in all lung fields. He has mild exacerbation of his symptoms and needs only outpatient treatment.
Clinical Case : Asthma (examination on page 69)

HOPI Past History


OCD PQRST UV + AAA Do you have asthma/other allergies?
How did it start? Sudden or gradual. Are you on any medications ana compliance?
Is it getting worse/better or no changes in Are you allergic to any medications?
the symptoms? Any hospitalizations for asthma?
Do you wake up in night with shortness of H/o any other medical illness?
breath?
Do you have noisy breathing? Family and Social History
H/o fever/sore throat/rash? Do you smoke? Duration & frequency.
Any night time cough? Do you consume alcohol? Duration &
Any sputum production? frequency.
Any cnest pain with deep inspiration? Do you use recreational drugs? TRAPPED.
Any recent activity whicn worsened your Any family history of asthma/allergies?
svmptoms?
Any exposure to cold air/dust/mites?
Any pets at home?
Any changes in bowel & urinary habits?
Any contact with a sick person?
Any recent travel?
Any recurrent episodes?
Do symptoms affect your daily activities?

Differential Diagnosis Management


Asthma. Ventolin 2-4 puffs MDI q4-6h.
Acute bronchitis. Fluticasone 2-4 puffs bid(Inhaled steroids)
GERD. Asthma education:
Pneumonia. Inhaler use.
Spacer and holding chamber use.
Investigations Symptom monitoring.
CBC. Early recognition of exacerbation.
Pulmonary function tests. Avoid environmental allergens.
Peak flow meter. Encase mattress and pillow in
Chest X ray. impermeable cover.
Reduce indoor humidity to <50%.
104 NAC OSCE |A Comprehensive Review

Larry Edwards, a 55 years old man presented with blood in sputum and shortness of breath for the past 5
days. Take a focused nistory and perform focused physical examination.
Vitals: BP - 160/110 mm Hg, HR - 96/min, RR - 18/min,Temp - 37.5C.

Clinical Info: Mr Larry Edwards is a known hypertensive who presented with shortness of breath and blood
in sputum for 5 days. It is gradual in onset. He has chest pain also. No fever or recurrent pneumonia. He is
non compliant with his medications. He is on Losartan, Aspirin, Atorvas, multi vitamins. He has not taken
his anti-hypertensives for 4 weeks. Has paroxysmal nocturnal dyspnea and orthopnea.
Clinical Case : Congestive Heart Failure (examination on page 65)

HOPI Past History


OCD PQRST UV + AAA Do you have diabetes or hypertension?
How did it start? Sudden or gradual. Are you on any medications?
Is it getting worse/better or no changes in Compliance with medications?
the symptoms? Are you allergic to any medications?
Duration of blood in sputum? Any surgeries in the past?
Amount /color of blood of sputum? Past h/o recurrent infections?
Any of shortness of breath?
Present at rest or with exertion? Family and Social History
H/o orthopnea? Do you smoke? Duration & frequency.
H/o paroxysmal nocturnal dyspnea? Do you consume alcohol? Duration &
Any chest pain/palpitations? frequency.
Type of pain - sharp or dull? Do you use recreational drugs? TRAPPED.
Does the pain radiate or shoot anywhere? Any family history of cancers/ medical
Severity of pain on a scale of 1-10. illnesses?
Do your symptoms change with time?? Any family history of heart disease?
H/o fever/cough/sore throat/rash?
Any chills/nignt sweats?
Any swelling of face or feet?
Any change in weight/appetite?
Any bruises on the body?
Any trauma recendy?

Differential Diagnosis Management


Pulmonary Edema due to CHF. Symptomatic treatment.
Pulmonary embolism. Admit in cardiac care unit.
Bronchiectasis. Inj Lasix 40 mg IV stat.
Drug induced coagulopathy. Beta blockers & ACE inhibitors.
Pneumonia. Inj Morphine 2-4 mg IV.
Nasal oxygen.
Investigations Sublingual Nitroglycerines.
CBC, electrolytes, glucose. Position - 45 degree elevation of head end.
LFT, RFT.
PT, PTT, INR.
12 lead ECG.
Echocardiogram.
Chest X Ray.
Clinical Cases - Medicine 105

Jack Allen, a 65 years old man presented with sudden onset of right arm weakness 4 hours ago. Take a focused
history and perform focused physical examination.
Vitals: BP - 160/90 mm Hg, HR - 96/min, RR - 12/min,Temp - 37.5C.

Clinical Info: Mr Jack Allen presented with sudden onset of right arm weakness with numbness and
paresthesias 4 hours ago. He has slurring of speech, blurring ofvision and mild headache. He has no
nausea,vomiting or head trauma. No weakness of lower limbs or left arm. No incontinence. He is hypertensive
for the past 10 years and non compliant to medications.
Clinical Case : Cerebrovascular Attack (examination on page 71)

HOPI Past History


OCD PQRST UV + AAA Do you have diabetes or hypertension?
How did it start? Sudden or gradual. Are you on any medications/compliance?
Is it getting worse/better or no changes in Are you allergic to any medications?
the symptoms? Any surgeries in the past?
Duration of weakness?
Where is the weakness located? Family and Social Histoiy
Is there any associated pain in the arm? Do you smoke? Duration & frequency.
Any numbness/tingling/paresthesias? Do you consume alcohol? Duration &
Any limitations in right arm movements? frequency.
Any abnormal position of right arm? Do you use recreational drugs? TRAPPED.
Do your symptoms change with time? Any family history of cancers/ medical
Any changes in vision? illnesses?
Any changes in speech?
Any difficulty swallowing/drooling of saliva?
Any headache/trauma?
Any weakness of left arm or lower limbs?
Any seizures?
Any changes in facial expression?
H/o fever/cough/sore throat/rash?
Any chest pain/shortness of breath?
Any changes in bowel & urinary habits?

Differential Diagnosis Management


Cerebrovascular Attack. Admit urgently.
Transient Ischaemic Attack. Urgent neurological consult.
Sub Arachnoid Hemorrhage. Neurovitals q 1 hourly.
Nasal oxygen.
Investigations Tab Aspirin 325 mg PO OD.
CBC, ESR, Glucose. Blood pressure control.
Lipid profile, RFT. Thrombolysis to be done only if presented
PT, PTT, INR. within 3 hours of onset of symptoms.
ECG.
CT MRI Head.
Echo, Carotid doppler.
106 NAC OSCE |A Comprehensive Review

Allan Smith, a 70 years old man presented with light headedness and dizziness for 2 days. Take a focused
history and perform focused physical examination.
Vitals: BP - 110/80 mm Hg, HR - 56/min, irregular, RR - 12/min,Temp - 37.0C.

Clinical Info: Mr Allan Smith is a known hypertensive for the past 20 years on medications. He is
experiencing light headedness and dizziness for the past 2 days. He has palpitations and mild chest pain for 2
weeks. He is breathless on exertion. Has 2+ pedal edema. No fainting episode. No trauma recently. No visual
changes or limb weakness. He is on Losartan, Ramipril, Digoxin, Atorvas, Aspirin, Nexium and Calcium.
ECG shows Type 2 Second Degree AV block.
Clinical Case : Digoxin Toxicity (examination on page 65)

HOPI Past History


OCD PQRST UV + AAA Do you have diabetes or hypertension?
How did it start? Sudden or gradual. Are you allergic to any medications?
Is it getting worse/better or no changes in Any surgeries in the past?
the symptoms? Past h/o recurrent infections?
Duration of dizziness?
Any relieving factors? Family and Social History
Any aggravating factors? Do you smoke? Duration & frequency.
Any episode of fainting? Do you consume alcohol? Duration oc
Any weakness/tingling/paresthesias of limbs? frequency.
Any chest pain/palpitations? Do you use recreational drugs? TRAPPED.
Any shortness of breath? Any family history of cancers/ medical
Any visual changes/headache? illnesses?
Any hearing loss/ear discharge?
Any excessive sweating?
Any fever/cough/vomiting?
Any abdominal pain/loss of appetite?
Any changes in bowel/urinary habits?
Any swelling of ankles?
Do your symptoms vary with change in head
position?
List of current medications & compliance?
Any changes in medications & dosages?
Any recent trauma?

Differential Diagnosis Management


Digoxin toxicity. Admit in cardiac care unit.
Arrhythmia. Stop Digoxin.
TIA. Start Digibind in case of massive overdose or
Anxiety disorder. refractory toxicity.
Medication induced bradycardia. External Pacemaker.

Investigations
Serum Digoxin level.
CBC, electrolytes, RFT.
INR/PTT, glucose.
ECG, 24 hour Holter monitor.
Echocardiogram,Carotid Doppler.
Clinical Cases - Medicine 107

Lisa Giroux, a 25 years old lady presented with lump in the neck for the past 7 days. Take a focused history
and perform focused physical examination.
Vitals: BP - 120/88 mm Hg, HR - 96/min, RR - 12/min,Temp - 38.5C.

Clinical Info: Ms Lisa Giroux noticed 2 lumps on the right side of her neck below the mandible. She has
positive history of fever for 5 days,sore throat and fatigue. On examination she has 2 enlarged, tender
submandibular lymph nodes. Monospot test is positive.
Clinical Case : Infectious Mononucleosis (Sore throat )

HOPI Past History


OCD PQRST UV + AAA Do you have diabetes or hypertension?
How did it start? Sudden or gradual. Are you on any medications?
Is it getting worse/better or no changes in Are you allergic to any medications?
the symptoms? Any surgeries in the past?
Since how long have you noticed the lumps? Past h/o recurrent infections?
Where are the lumps located?
How does the lumps feel like? Family and Social History
Is there any pain associated with the lumps? Do you smoke? Duration & frequency.
Type of pain - sharp or dull pain? Do you consume alcohol? Duration &
Does the pain radiate or shoot anywhere? frequency.
Severity of pain on a scale of 1-10. Do you use recreational drugs? TRAPPED.
Do your symptoms change with time? Any family history of cancers/ medical
H/o fever/cough/sore throat/rash? illnesses?
Any chest pain/shortness of breath?
Any abdominal pain?
Any weakness or myalgia?
Any chills/night sweats?
Any change in voice?
Any change in appetite?
Any change in weight?
Any changes in bowel & urinary habits?
Any contact with a sick person?
Recent travel?

Differential Diagnosis Management


Infectious mononucleosis. Symptomatic treatment.
Acute bacterial pharyngitis. Maintain adequate hydration.
Acute viral pharyngitis. Rest.
Lymphoma. Analgesics for pain.
Toxoplasmosis. Treat Streptococcal pharyngitis, if coexists.
Avoid active sports.
Investigations Steroids used only for severe symptoms.
CBC.
Monospot test.
Throat swab for gram stain, culture 8c
sensitivity.
108 NAC OSCE |A Comprehensive Review

Jason Hardinge, a 26 years old university student wants to discuss confidential issues with a doctor. Take
focused history and address his concerns.
Vitals: BP - 120/88 mm Hg, HR - 88/min, RR - 12/min,Temp - 37.5C
Clinical Info: Mr Jason Hardinge is having difficulty in maintaining erection duringintercourse for the past 4
months. He is currently in a monogamous relationship with his girlfriend. He is on raroxetin for his mood
disorder for 6 months. No other medical illnesses. Girlfriend is very understanding. He has no morning or
night tumescence. Has no erection with self stimulation. He is very anxious about this issue.
Clinical Case : Impotence

HOPI Sexual orientation -


OCD PQRST UV + AAA heterosexual/homosexual/bisexual?
How did it start? Sudden or gradual. Current relationship?
Is it getting worse/better or no changes in Performance anxiety?
the symptoms? Any stresses in the relationship?
Duration of impotence? Any stresses at home or school?
Course & frequency of impotence? Sexual partner's expectations?
Description of the problem: no erection at Are you on any medications?
all, cannot sustain erection, ejaculate too Compliance/type of medications - details?
quickly to satisfy partner?
Cannot achieve orgasm or orgasm without Past History
ejaculation? Do you have diabetes or hypertension?
Any retrograde ejaculation? Are you allergic to any medications?
Circumstances under which impotence Any surgeries in the past?
occurs only with certain partners, only at Any trauma in the past?
certain times or locations, what percentage of Past h/o recurrent infections?
the time?
Is impotence related to lack of sexual desire? Family and Social Histoiy
Presence and firmness of morning or Do you smoke? Duration & frequency.
nocturnal erections. Do you consume alcohol? Duration oc
Can sustain erections with self stimulation? frequency.
Associated problems: anxiety attacks, Do you use recreational drugs? TRAPPED.
anhedonia/aepression, perineal or peripheral Any family history of cancers/medical
numbness, poor peripheral circulation. illnesses?

Management
Impotence secondary to antidepressants. Complete physical exam.
Reassurance, counseling both patient &
impotence : (IMPOTENCE)
Causes of imp partner.
lie, Mechanical, Psychological,
Iatrogenic, Inform that symptoms are due to side effects
Occlusive vascular, Trauma, Extra factors, of anti-depressants.
Neurogenic, Chemical, Endocrine. Symptoms are reversible by changing the
dose or the type of drug.
Investigations Substitute with another anti-depressant:
CBC, blood glucose,TSH. Minimal to no sexual dysfunction
Se Testosterone. Nefazodone (Serzone)
Urinalysis. Bupropion (Wellbutrin)
Endocrine lab tests, if indicated: Low risk of sexual dysfunction (10-15%)
FSH,LH, Prolactin. Fluvoxamine (Luvox)
Citalopram (Celexa)
Venlafaxine (Effexor)
Avoid alcohol/smolting.
Medical treatment: Tab Sildenafil 25-50 mg
PO 0.5 to 4 hours prior to coitus.
Clinical Cases - Medicine 109

Taylor Jackson, a 18 years old boy presented with fever, neck stiffness and photophobia to the ER.Take a
focused history and perform focused physical examination.
Vitals: BP- - 90/70 mm Hg,,Hf R - 110/min, RR - 12/min,Temp - 39.0C.

Clinical Info: Mr Taylor Jackson has high grade fever for the past 3 days alone with neck stiffness. He has
photophobia for the past 1 day. He is alert & conscious. No seizures. Has heaaache with nausea & vomiting.
No ear discharge. Has a purpuric rash on chest and lower limbs. No recent trauma. Has h/o contact with sick
rson with similar symptoms. O/E: Febrile, Brudzinski's and Kernig's sign are positive,
S linical Case : Meningitis (examination on page 71)

HOPI Past History


OCD PQRST UV + AAA Do you have medical illnesses?
How did fever start? Sudden or gradual. Are you on any medications?
Is it getting worse/better or no changes in Are you allergic to any medications?
the symptoms? Any surgeries in the past?
Duration of fever? Past h/o recurrent infections?
Continuous/intermittent/remittent?
Duration of neck stiffness? Family and Social History
Associated with headache/nausea/vomiting? Do you smoke? Duration & frequency.
Any photophobia/phonophobia? Do you consume alcohol? Duration &
Any fatigue/malaise? frequency.
Any fainting/seizures/confusion/irritability? Do you use recreational drugs? TRAPPED.
H/o fever/cough/sore throat/rash? Any family history of cancers/ medical
Any chills/night sweats? illnesses?
Any chest pain/shortness of breath?
Any abdominal pain?
Any changes in bowel & urinary habits?
Any contact with a sick person?
Recent travel?
Recent head trauma?

Differential Diagnosis Management


Bacterial Meningitis Admit under isolation.
Encephalitis. Start empiric antibiotics prior to LP.
Intra-cerebral abscess. Inj Cefotaxime 2 g IV q4h +
Inj Ampicillin 50 mg/kg IV q6h.
Investigations Inj Dexamethasone 10 mg q6h IV x 4
CBC, electrolytes, glucose. days.
RFT, ABG. Treat all close contacts.
Lumbar puncture - gram stain, microscopy,
culture/sensitivity (rule out papilledema).
Blood culture/sensitivity, gram stain.
Urinalysis.
CT scan head.
Chest X Ray.
110 NAC OSCE |A Comprehensive Review

James Irwin a 30 years old man presented to your clinic with symptoms of headache. Take a focused history
and address his concerns.
Vitals: BP - 120/88 mm Hg, HR - 96/min, RR - 12/min,Temp - 37.5C.

Clinical Info: Mr James Irwin presented with unilateral, pulsating headache, grade 7/10 for the past 6
months. He experiences aura prior to the onset of headache. Associated with nausea,vomiting and
photophobia. Stimulated by stress and excessive caffeine intake. One episode lasts for 8-12 hours. He had 6
attacks in past 6 months. Currently on Advil prn.
Clinical Case : Headache (Migraine)

HOPI Past History


OCD PQRST UV + AAA Do you have diabetes or hypertension?
How did it start? Sudden or gradual. Any head trauma in the past?
Location of pain? Are you on any medications?
Is it getting worse/better or no changes in Are you allergic to any medications?
the symptoms? Any surgeries in the past?
Since how long have you noticed the pain? Past h/o recurrent infections?
Number of episodes of headache till now?
How many hours pain lasts? Family and Social Histoiy
How does the pain feel like? Do you smoke? Duration & frequency.
Does the pain radiate or shoot anywhere? Do you consume alcohol? Duration &c
Severity of pain on a scale of 1-10. frequency.
Any provoking factors like food, alcohol, Do you use recreational drugs? TRAPPED.
caffeine, chocolate, wine, stress. Any family history of cancers/ medical
How does the pain decrease? illnesses?
Do your symptoms change with time?
H/o fever/cough/sore throat/nasal stuffiness?
Any excessive lacrimation?
Any visual changes prior to onset of
headache?
Any neck pain/rigidity/seizures?
Any weakness/muscle pain/paresthesias?
Any mood changes/decreased sleep/energy?

Differential Diagnosis Management


Migraine with Aura. Symptomatic treatment.
Tension Headache. Avoid triggers.
Cluster Headache. Mild attack - ASA, Ibuprofen.
Temporal Arteritis Moderate attack - Anti-emetics,
Ibuprofen, Triptans, ergots.
Investigations Severe attack - Anti-emetics, Ergotamines,
CBC, ESR Triptans.
Migraine prophylaxis - Propranolol,
Amitriptyline, Verapamil.
Clinical Cases - Medicine 111

Michael Smith, a 55 years old man presented with chest discomfort for the past 1 hour. Take a focused
history and perform focused physical examination.
Vitals: BP - 160/90 mm Hg, H K - 96/min, RR - 12/min,Temp - 37.5C.

Clinical Info: Mr Michael Smith presented with left sided chest discomfort for the past 1 hour. He has pain
in his left shoulder and jaw. He has shortness of breath along with palpitations. He is a known hypertensive
and diabetics on oral medications. His wife states he is non-compfiant with his medications. ECG shows ST
elevation in leads II,III and avF.
Clinical Case : Chest Pain (Myocardial Infarction) (examination on page 65)

HOPI Past History


OCD PQRST UV + AAA Do you have diabetes or hypertension?
How did it start? Sudden or gradual. Are you on any medications/compliance?
Is it getting worse/better or no changes in Are you allergic to any medications?
the symptoms? Any surgeries in the past?
Duration of chest discomfort? Past h/o recurrent infections?
Location of chest discomfort?
How does the discomfort feel like? Family and Social History
Any pain associated with chest discomfort? Do you smoke? Duration & frequency.
Does the pain radiate or shoot anywhere? Do you consume alcohol? Duration Sc
Severity of pain on a scale of 1-10. frequency.
Any shortness of breath? Do you use recreational drugs? TRAPPED.
Any palpitations/diaphoresis/dizziness? Any family history of cancers/ medical
Do your symptoms become change with illnesses?
time?
H/o fever/cough/sore throat/rash?
Any hemoptysis?
Any heart burn?
Any abdominal pain?
Any weakness or myalgia?
Any changes in bowel & urinary habits?
Any swelling of feet?
Any relieving factors?
Any aggravating factors?

Differential Diagnosis Management


Myocardial Infarction. Admit to cardiac care unit.
Unstable Angina. Urgent cardiology consult.
Pericarditis. Nasal oxygen.
Panic Attack. Tab Aspirin 160-325 mg chewable stat.
Inj Morphine 2-4 mg IV stat & pm.
Investigations Sublingual Nitroglycerin 0.4 mg stat, repeat
CBC, electrolytes, glucose. 3 times.
INR/PTT. Ini Atenolol 5 mg IV bolus over 5 mins, then
Serial CK-MB & Troponin q8h X 3. repeat.
ABG. Then switch to oral Atenolol 50 mg.
CXR. Cardiac monitoring every 30 minutes-1
12 lead ECG. hour.
Bed rest.
Consider PCI, in case of no
contraindications.
112 NAC OSCE |A Comprehensive Review

Adam Sawyer, a 18 years old male presented with fever, cough for 1 week along with shortness of breath. Take
a focused history and perform focused physical examination.
Vitals: BP - 110/70 mm Hg, HR - 96/min, RR - 20/min,Temp - 38.5C.

Clinical Info: Adam Sawyer has fever and expectorant cough for the past 1 week. He has wheezing and
shortness of breath for 2 aays. On auscultation of chest, there is decreased breath sounds on left side with
rales present.
Clinical Case : Pneumonia (examination on page 69)

HOPI Past History


OCD PQRST UV + AAA Do you have diabetes or hypertension?
How did it start? Sudden or gradual. Are you on any medications?
Is it getting worse/better or no changes in Are you allergic to any medications?
the symptoms? Any surgeries in the past?
Duration of cough? Past h/o recurrent infections?
Type of cough - dry or expectorant?
Is there any chest pain? Family and Social History
Location of chest pain? Do you smoke? Duration & frequency.
Type of pain - sharp or dull? Do you consume alcohol? Duration &
Does the pain radiate or shoot anywhere? frequency.
Severity of pain on a scale of 1-10. Do you use recreational drugs? TRAPPED.
Pain present with deep breathing? Any family history of cancers/ medical
Onset of shortness orbreath? illnesses?
Any noisy breathing present?
Any chills/night sweats?
Any changes in bowel & urinary habits?
Any contact with a sick person?
Recent travel?

Differential Diagnosis Management


Community Acquired Pneumonia. Outpatient treatment.
Acute exacerbation of COPD. Tab Doxycycline 100 mg PO bid x 7 days.
Acute bronchitis. Tab Azithromycin 500 mg PO OD x 5 days.
Asthma. Rest and adequate hydration.
Symptomatic treatment.
Investigations Inpatient treatment for 48-72 hours.
CBC. Tab Levofloxacin 750 mg PO q24h.
Sputum gram stain with culture/sensitivity. Tab Amoxicillin 1000 mg PO aid +
Electrolytes. Tab Clarithromycin 500 mg PO bid.
Renal function tests.
Chest X Ray.
Clinical Cases - Medicine 113

Helen Solazzo is an ICU nurse who had a needle stick injury 30 minutes ago while drawing blood sample
from a patient .Take a focused history and address her concerns.
Vitals: bP - 120/88 mm Hg, HR - 86/min, RR - 12/min,Temp - 37.0C.

Clinical Info: Ms Helen Solazzo had a needle stick injury in the ICU 30 minutes ago. She was drawing blood
sample at that time. She was wearing gloves. She has no nigh risk behavior. Her immune status for
HIV/HCV/HBsAg is negative as of 1 year ago. Patient's immune status is unknown as of now.
Physician examiner states that the patient's result come back positive for HIV.
Clinical Case Diagnosis: Post exposure prophylaxis for HIV.

HOPI Past History


Mechanism of injury. Do you have medical illnesses?
Was the nurse wearing gloves? Are you on any medications?
Were adequate occupational precautions Are you allergic to any medications?
taken? (like gloves, mask.) Any surgeries in the past?
Depth of needle penetration? Past h/o recurrent infections?
Type of needle- hollow or solid?
Needle gauge? Family and Social History
Was any blood present on the tip of needle? Do you smoke? Duration & frequency.
Any contact with patient's body fluids? Do you consume alcohol? Duration &
Steps taken after injury? frequency.
Immune status of tne nurse? Do you use recreational drugs? TRAPPED.
When was the last immune status checked? Any family history of cancers/ medical
Immunization for Hepatitis A and Hepatitis illnesses?

Any high risk behavior like unprotected sex?


Any similar incident in the past?
Is the immune status of the patient known?
Patient's medical history?

Investigations Management
For Healthcare professional: Reassurance.
CBC, electrolytes. Refer to Infectious Disease clinic.
RFT, LFT. Report to occupational health dept within 72
HIV, HCV, HBsAg. hours & every 2 weekly.
For the patient: inform the pt. Certify to worker's compensation board for
CBC, electrolytes. file claim.
HIV, HCV, HBsAg. Advise about safe sex practices.
In case of positive HIV/HCV/HBsAg do Avoid pregnancy/breast feeding.
viral loads & CD 4 counts. Repeat blood work 6 weeks,12weeks,6
months and 12 months.
Patient HIV + then start the nurse on post
exposure prophylaxis for 4 weeks.
Counsel about side effects of medications.
114 NAC OSCE |A Comprehensive Review

Jacob Sandler, a 50 years old man presented with hemoptysis and right sided calf swelling for the past 2 days.
He had knee replacement surgery 1 week ago. Take a focused history and perform focused examination.
Vitals: BP - 140/80 mm Hg, HR - 110/min, RR - 18/min,Temp - 37.5C.

Clinical Info: Mr Jacob Sandler had a right knee replacement 1 week ago. He now presented with 2 episodes
of hemoptysis and right calf swelling witn tenderness. He has no fever or infection of surgical wound.
Homans sign is positive with ECG showing S1Q3T3 pattern.
Clinical Case : Pulmonary Embolism (examination on page 69)

HOPI Past History


OCD PQRST UV + AAA Do you have diabetes or hypertension?
How did it start? Sudden or gradual. Are you on any medications?
Is it getting worse/better or no changes in Are you allergic to any medications?
the symptoms? Any surgeries in the past?
Duration of blood in sputum? Past h/o recurrent infections?
Amount /color of blood of sputum?
Onset of shortness of breath/chest pain? Family and Social Histoiy
Present at rest or with exertion? Do you smoke? Duration & frequency.
H/o orthopnea? Do you consume alcohol? Duration &,
H/o paroxysmal nocturnal dyspnea? frequency.
Duration of leg swelling? Do you use recreational drugs? TRAPPED.
Where is the leg swelling located? Any family history of cancers/ medical
Any leg pain associated with swelling? illnesses?
Type of pain - sharp or dull pain?
Does the pain radiate or shoot anywhere?
Severity of pain on a scale of 1-10.
Any pain during rest?
Any pain in the night time?
Any skin discoloration of legs/nail changes?
Any skin ulceration of legs?
Any fever/cough/cold?
Any headache/dizziness?
Any weakness/muscle pain?
Any prolonged immobilization?
Any pain/discharge from the wound?
Any pre-op or intra-op complications?

Differential Diagnosis Management


Pulmonary embolism. Admit in Intensive care unit.
Deep Vein Thrombosis. Elevate head end to 45 degree.
Pulmonary edema due to CHF Nasal oxygen.
Myocardial infarction. Give chewable ASA 160-325 mg
immediately.
Investigations Secure IV access, bolus IV Lasix 40 mg.
CBC, RFT. Ventolin if wheezes are heard.
Electrolytes, glucose. Sublingual nitro spray, if blood pressure is
Serial CK-MB and Troponin q8h X 3. adequate.
Arterial blood gases. Inj Morphine 1 mg IV.
D-dimer, PT, PTT, INR, factor assay. Inj Heparin 7500 U IV bolus, then infuse at
ECG, Echocardiogram. 1200 U/h, then switch to warfarin.
Chest X Ray. Continue anticoagulation for 3 months.
PET Scan or VQscan.
Doppler of lower limbs.
Clinical Cases - Medicine 115

Jasper Preudhomme, a 16 years old boy is a known epileptic presented to your clinic for the first time. Take a
focused history and address his concerns.
Vitals: BP - 120/88 mm Hg, HR - 96/min, RR - 12/min,Temp - 37.0C.

Clinical Info: Mr Jasper Preudhomme is a known epileptic for the past 6 years. He is on regular anti
epileptics and is non-compliant. His last seizure was 2 months ago. He recently started consuming alcohol
with friends. His main concern is to get a driver's license.
Clinical Case : Seizure disorder

HOPI Past History


Age of onset. Do you have diabetes or hypertension?
Prcipitants: Sleep deprivation, drugs, Any surgeries in the past?
alcohol, TV screen,emotional upset. Past h/o recurrent infections?
Describe type of seizures.
Salivation, cyanosis, tongue biting, Family and Social History
incontinence, automatisms, motor vs. Do you smoke? Duration & frequency.
visual/gustatory/olfactory. Do you consume alcohol? Duration &
Frequency & duration of seizures. frequency.
What body parts affected and in what order? Do you use recreational drugs? TRAPPED.
Premonitory signs (presence of aura: implies Any family history of cancers/ medical
focal attack). illnesses?
Post-ictal state (decrease in level of
consciousness, headache, sensory
phenomena, tongue soreness, limb pains,
Todd's paralysis - hemiplegia)
Degree of control achieved with
medications.
Was a CT scan done when seizures were first
diagnosed?
Number and description of recent seizures.
Are they different from previous seizures?
Is the patient having any new symptoms like
headache, vomiting, new neurological
deficits?
Side effects of antiepileptics: drowsiness,
poor concentration, poor performance in
school, ataxias, peripheral neuropathy, acne,
nystagmus, dysarthria, hypertrichosis,
gingival hypertrophy.

Diagnosis Management
Seizure Disorder Discuss compliance of medications.
Regular follow up.
Investigations Avoid alcohol consumption/smoking.
CBC, electrolytes. Avoid recreational drugs.
Serum drug levels. Inform to the patient Ministry of
EEG. Transportation regulations require patient to
be seizure free for 1 year or more.
Notify Ministry of Transportation as
required by law.
116 NAC OSCE |A Comprehensive Review

Samantha Ho, a 56 years old woman presented to your clinic with symptoms of headache and blurry vision.
Take a focused history and address her concerns.
Vitals: BP - 130/88 mm Hg, HR - 86/min, RR - 12/min,Temp - 37.5C.

Clinical Info: Ms Samantha Ho presented with unilateral, left temporal side pulsating headache, grade 7/10
for the past 2 weeks. She experiences headache while chewing and combine her hair. Associated with blurring
of vision and diplopia. One episode lasts for 30 minutes. Currendy on advilprn, atenolol 50 mg OD and
multivitamins.
Clinical Case : Temporal Arteritis

HOPI Past History


OCD PQRST UV + AAA Do you have diabetes or hypertension?
How did it start? Sudden or gradual. Any head trauma in the past?
Location of pain? Are you on any medications?
Is it getting worse/better or no changes in Are you allergic to any medications?
the symptoms? Any surgeries in the past?
Since how long have you noticed the pain? Past h/o recurrent infections?
Number of episodes of headache till now?
For how long the pain lasts? Family and Social Histoiy
How does the pain feel like? Do you smoke? Duration & frequency.
Does the pain radiate or shoot anywhere? Do you consume alcohol? Duration &
Severity of pain on a scale of 1-10. frequency.
Any provolang factors like food, alcohol, Do you use recreational drugs? TRAPPED.
caffeine, chocolate, stress, combing hair. Any family history of cancers/ medical
How does the pain decrease? illnesses?
Do your symptoms change with time?
H/o fever/cough/sore throat/nasal stuffiness?
Any excessive lacrimation?
Any visual changes with headache?
Any neck pain/rigidity/seizures?
Any weakness/muscle pain/paresthesias?
Any mood changes/decreased sleep/energy?

Differential Diagnosis Management


Temporal arteritis. High dose corticosteroids.
Migraine. Tab Prednisone 60 mg PO OD until
TIA. symptoms subside and ESR is normal, then
40 mg PO OD for 4-6 weeks. Then taper to
Investigations 5-10 mg PO OD for 2 years.
CBC, ESR, CRP, LFT. Relapses occur in 50% if treatment is
Temporal artery biopsy. stopped before 2 years.
Visual acuity. Monitor ESR regularly.
Fundoscopy. If visual symptoms present, then admit &
start Inj Prednisolone 1000 mg IV ql2h for
5 days.
Clinical Cases - Medicine 117

Jason Scott, a 30 years old manjpresented with yellowish discoloration of eyes and skin for the past 1 week.
Take a focused history and perform focused physical examination.
Vitals: BP - 120/88 mm Hg, HR - 96/min, RR - 12/min,Temp - 38.0C.

Clinical Info: Mr Jason Scott presented with yellowish discoloration of eves and skin for the past 1 week. It
has progressed gradually. He has right upper quadrant abdominal pain. He has loss of appetite, malaise,
nausea and vomiting. His urine is high colored and stool is pale colored. He has low graae fever. He has few
tattoos on his body along with body piercing. He is a chronic alcoholic, smoker and fv drug user for the past
10 years.
Clinical Case : Viral Hepatitis (examination on page 63)

HOPI Past History


OCD PQRST UV + AAA Do you have medical illnesses?
How did it start? Sudden or gradual. Are you on any medications?
Is it getting worse/better or no changes in Are you allergic to any medications?
the symptoms? Any surgeries/dental procedures in the past?
Duration of yellowish discoloration? Past h/o recurrent infections?
Any fever/chills/night sweats?
Any abdominal pain/location/duration? Family and Social History
Any nausea/vomiting? Do you smoke? Duration & frequency.
Any pruritus/rash? Do you consume alcohol? Duration &
Any bruises/spontaneous bleeding? frequency.
Any loss of appetite/weight? Do you use recreational drugs? TRAPPED.
Any bowel complaints/color of stools? Any tattoos/body piercing?
Any urinary complaints/color of urine? Are you in a sexual relationship?
Any fatigue/malaise? Do you practice safe sex?
Any confusion/irritability/seizures? Are you at risk of sexually transmitted
Any aggravating factors? infections?
Any relieving factors? Any family history of medical illnesses?
Any contact with sick person? Immunization history.
Any recent travel?

Differential Diagnosis Management


Acute viral Hepatitis. Symptomatic treatment.
Alcoholic hepatitis. Treat the primary cause.
Bacterial hepatitis. Educate about safer sex practices.
Notify public health in case of HAV,
Investigations HBsAg,HCV or HIV positive serology.
CBC, electrolytes, glucose.
LFT, RFT.
INR, PT, PTT.
HAV, HBsAg, HCV, HIV serology.
Ultrasound abdomen.
118 NAC OSCE |A Comprehensive Review

Anna Levy, a 32 years old lady presented with lower abdominal pain and vaginal spotting for 2 days.. Take a
focused history.
Vitals: BP - 120/80 mm Hg. HR - 90/min. RR - 12/min.Temp - 37.5C

Clinical Info: Ms Anna Levy presented with h/o lower abdominal pain and vaginal spotting for 2 days. LMP:
6 weeks ago. Bi-manual exam has cervical motion tenderness with open os and bleeding +++.
Clinical Case : Abortion

HOPI Obstetrical History


OCD PQRST UV + AAA Do you have children? If yes, then ask for
How did it start? Sudden or gradual. Gravidity, Term/Premature deliveries,
Is it getting worse/better or no changes in Abortions, Live/Multiple births,
the symptoms? complications in pregnancy.
Since how long have you noticed the pain? H/o ectopic pregnancy?
Where is the pain located?
Sharp or dull pain? Past History
Does the pain radiate or shoot anywhere? Do you have diabetes or hypertension?
Severity of pain on a scale of 1-10. Are you on any medications?
Type 8c amount of vaginal bleeding? Are you allergic to any medications?
Number of pads changed? Any surgeries in the past?
Passage of clots per vagina? Past h/o sexually transmitted infections?
Do your symptoms change with time?
H/o fever/nausea/vomiting? Family and Social History
Any changes in bowel 8c urinary habits? Do you smoke or consume alcohol?
Date of last menstrual period? Do you use recreational drugs?
Last intercourse? Currendy in a relationship? How long?
Blood group? Sexual orientation?
Are you at risk of abuse?
Gynecological History Any family history of cancers?
Age of onset of menses
Regular/irregular?
Duration of menses?
Amount of bleeding/passage of clots?
Any dysmenorrhea?

Differential Diagnosis Management


Spontaneous abortion. Admit.
Incomplete abortion. Dilatation 8c curettage in case of incomplete
Ectopic pregnancy. abortion.
Implantation bleed. IVF for hemodynamic stability.
Postcoital trauma Serial Beta HCG till levels<l.

Investigations
CBC, electrolytes, renal function tests.
Beta HCG
Pelvic ultrasound.
Blood group 8c type.
Clinical Cases - Obstetrics & Gynecology 119

Rachel Owens, a 42 years old primigravida who is 9 weeks pregnant. She came to your office to know about
her genetic risks. Take a focused history and address her concerns.

Clinical Info: Ms Rachel Owens conceived naturally and this is her first pregnancy. She is sure of her dates.
She didn't have any antenatal visit yet. This is her first visit. Her home pregnancy test was positive twice. No
family history of genetic disorders. She is only taking prenatal vitamins. No h/o medical illnesses.
She does not smoke or consume alcohol.
Clinical Case : Antenatal Visit

HOPI Past History


Congratulate the patient. Do you have diabetes or hypertension?
When was your last period? Are you on any medications?
Conceived naturally or with treatment? Are you allergic to any medications?
Any antenatal visits till date? Any surgeries in the past?
Any blood tests and ultrasound done? Past h/o sexually transmitted infections?
Any lower abdominal pain? When was your last PAP test & results?
Any bleeding per vagina?
Any radiation exposure till now? Family and Social History
Any medication taken in the past 2 months? Do you smoke or consume alcohol?
Any fever/nausea/vomiting? Do you use recreational drugs?
Any vaginal discharge? Currendy in a relationship? How long?
Intake of folic acid prior to conception? Are you at risk of abuse?
Blood group? Any family history of cancers?
Any family history of genetic disorders?
Gynecological History Any h/o genetic aisorders in the partners
Age of onset of menses family?
Regular/irregular? Support system at home?
Duration of menses? Any stressors at work or home?
Amount of bleeding/passage of clots?
Any dysmenorrhea?

Obstetrical History
Do you have children? If yes, then ask for
Gravidity, Term/Premature deliveries,
Abortions, Live/Multiple births,
complications in pregnancy.
H/o ectopic pregnancy?

Investigations Management
CBC Give antenatal brochures.
Urine culture/sensitivity, microscopy. Discuss about genetic screening &
Beta HCG Counseling.
ABO Rh, type. Referral to an obstetrician and prenatal
Blood sugar,TSH. genetics screening.
Measles,Mumps,Rubella,Varicella, VDRL. Nutrition &c exercise in pregnancy.
HIV,HBsAg,HCV. Avoid alcohol/smoking/teratogenic
Pelvic ultrasound. medications.
Discuss about risk of Down's/Turner's &
other genetic disorders in elderly
primigravida.
120 NAC OSCE |A Comprehensive Review

Lisa Raymond, a 28 years old lady presented to the ER with lower abdominal pain on the left side for the
past 12 hours. Take a focused history and perform a focused examination (Page 63).
Vitals: BP - 100/70 mm Hg. HR - 98/min. RR - 16/min.Temp - 37.5C

Clinical Info: Ms Lisa Raymond, presented with h/o left side lower abdominal pain for 12 hours with mild
spotting. LMP: 2 months ago. Bi-manual exam has cervical motion tenderness & left adnexal fullness.
Clinical Case : Ectopic Pregnancy

HOPI Obstetrical History


OCD PQRST UV + AAA Do you have children? If yes, then ask for
How did it start? Sudden or gradual. Gravidity, Term/Premature deliveries,
Is it getting worse/better or no changes in Abortions, Live/Multiple births,
the symptoms? complications in pregnancy.
Since how long have you noticed the pain? H/o ectopic pregnancy?
Where is the pain located?
Sharp or dull pain? Past History
Does the pain radiate or shoot anywhere? Do you have diabetes or hypertension?
Severity of pain on a scale of 1-10. Are you on any medications?
Type Sc amount of bleeding per vagina? Are you allergic to any medications?
Do your symptoms change with time? Any surgeries in the past?
H/o fever/nausea/vomiting? Past h/o sexually transmitted infections?
Any changes in bowel & urinary habits? When was your last PAP test & results?
Date of last menstrual period?
Any trauma recendy? Family and Social History
When was your last meal? Do you smoke or consume alcohol?
Blood group. Do you use recreational drugs?
Currendy in a relationship? How long?
Gynecological History Sexual orientation?
Age of onset of menses Are you at risk of abuse?
Regular/irregular? Any family history of cancers?
Duration of menses?
Amount of bleeding/passage of clots?
Any dysmenorrhea?

Differential Diagnosis Management


Ectopic pregnancy. Admit
Threatened abortion. IVF
Pelvic Inflammatory Disease. Urgent gynecology consult
Ovarian torsion. Rhogam if needea.
Endometriosis. Medical treatment:
Inj Methotrexate 50 mg/m2 given IM as stat
Investigations dose.
CBC. Serial Beta HCG till levels drop to <1.
Blood group, type and cross match. Contraception advice till completion of
Beta HCG treatment.
Electrolytes, Renal function tests. Surgical treatment : Laparoscopy/
Pelvic ultrasound. Laparotomy.
Clinical Cases - Obstetrics & Gynecology 121

Cathy Davies, a 32 years old lady presented with inability to conceive for the past 3 years. Take a focused
history and address her concerns.
Vitals: BP - 120/80 mm Hg. HR - 88/min. RR - 12/min.Temp - 37.0C

Clinical Info: Ms Cathy Davies has been unable to conceive for the past 3 years with unprotected intercourse.
She has not taken any treatment so far. Her periods are irregular witn prolonged intervals. She has weight
gain for past 2 years and hirsutism. She is in a monogamous relationship. No other stressors. She has a 5 years
old daughter conceived naturally. No other medical illnesses.
Clinical Case : Infertility

HOPI Gynecological History


Since how long trying to conceive? Last menstrual period?
Frequency of sexual intercourse? Age of onset of menses
Awareness of ovulation period? Regular/irregular?
Any difficulty /pain during/after intercourse? Duration of menses?
Any bleeding after intercourse? Amount of bleeding/passage of clots?
Any weight gain recendy? Any dysmenorrhea?
Any excessive hair growth?
Any vaginal discharge/type/duration? Obstetrical
Any abdominal pain/back pain? Do you have children? If yes, then ask for
Any swelling/masses noticed in the body? Gravidity, Term/Premature deliveries,
Any heat/ccSd intolerance? Abortions, Live/Multiple births,
Any nipple discharge/type/duration? complications in pregnancy.
H/o repetitive infections?
H/o fever/nausea/vomiting? Past History
Do you have diabetes/hypertension/thyroid
Male Partner History: diseases?
Any difficulty during intercourse? Are you on any medications?
H/o premature ejaculation? Are you allergic to any medications?
H/o difficulty maintaining ejaculation? Any surgeries in the past?
Any abdominal surgeries? Past h/o sexually transmitted infections?
Any trauma to the pelvic area in the past? When was your last PAP test & results?
Any medical illnesses?
H/o mumps in childhood? Family and Social History
Are you on any medications/allergies? Do you smoke or consume alcohol?
Do you smoke or consume alcohol? Do use recreational drugs?
Do you use recreational drugs? Are you at risk of abuse?
H/o sexually transmitted infections? Any family history of cancers?
Any stressors at home or work?
H/o of conception in past relationships?

Diagnosis Hystero-salphingogram.
Secondary infertility. Laparoscopy.
Semen analysis
Investigations
CBC,FBS,TSH. Management
Day 3 FSH,LH,PRLDHEAS,Free Complete physical examination of both the
testosterone. partners.
Day 21-23 Progesterone. Treat the cause.
Basal body temperature monitoring. Supportive counseling.
Pelvic ultrasound Timing of the intercourse in relation to
ovulation.
Referral to infertility specialist.
122 NAC OSCE |A Comprehensive Review

Alyssa Jones, a 18 years old girl came to your office requesting for contraceptive pills. Take a focused history
and address her concerns.
Vitals: BP - 120/80 mm Hg. HR - 80/min. RR - 12/min.Temp - 37.0C

Clinical Info: Ms Alyssa Jones is a 18 years old student with no significant history of medical illnesses. She is
in an active sexual relationship for the past 4 months. Had one episode of STI 6 months ago. LMP was 1
week ago. No family history of cancers. Currently using barrier contraception.
Clinical Case : OCP Counseling

HOPI Gynecological History


When was the last menstrual period? Age of onset of menses
Do you get headaches often? Regular/irregular?
Do you nave any bleeding disorders? Duration of menses?
Any thromboembolic events? Amount of bleeding/passage of clots?
Any liver disease? Any dysmenorrhea?
Any uncontrolled high blood pressure? Current contraception use?
Any heart disease?
Any mass in the breast? Obstetrical History
Any abnormal vaginal bleeding? Have you been pregnant before?
Any history of sexually transmitted If yes, then ask for Gravidity,
infections? Term/Premature deliveries, Abortions,
Any abnormal PAP results? Live/Multiple births, complications in
When was the last PAP done? pregnancy.
Sexual orientation? H/o ectopic pregnancy?
Currently in a relationship? How long?
Number of sexual partners? Past History
Do you practice safe sex? Any hospitalizations?
Do you smoke or consume alcohol? Any surgeries in the past?
Do you use recreational drugs?
Are you on any medications? Family History
Are you allergic to any medications? Family history of hypertension/diabetes?
Family history of breast/ovarian/endometrial
cancers?
Family history of bleeding disorders?

Investigations Management
PAP test & complete physical. Tab Yasmin one tab OD for 28 days.
Vaginal & Cervical swabs, culture/sensitivity. Begin pill on first Sunday after onset of
Menses
Benefits of OCP If Menses start on Sunday, then start pill
Prevention of unwanted pregnancy. Day 1
Reduced blood loss. Use barrier Contraception for Days 1-7
Decreased dysmenorrhea. If pill started after Day 5:
Cycle regularization. OCP may not suppress Ovulation for first
Decreased risk of breast/ovarian/endometrial cycle
cancers. Use barrier Contraception for first month.
Decreased acne. Follow up 6 weeks after the start of the pill.
Decreased osteoporosis.
Decreased PMS symptoms.
Reversible contraception.
Clinical Cases - Obstetrics & Gynecology 123

Maria Santosa, a 28 years old lady presented with lower abdominal pain, dyspareunia and vaginal discharge
for 1 week. Take a focused history and perform focused examination.
Vitals: BP - 120/80 mm Hg. HR - 90/min. RR - 12/min.Temp - 38.5C

Clinical Info: Ms Maria Santosa presented with h/o lower abdominal pain for 1 week with dyspareunia and
foul smelling vaginal discharge. She has mild fever for 2 days. H/o unprotected intercourse +. H/o of past
infection 3 months ago. LM r: 1 week ago. Bi-manual exam has cervical motion tenderness & right adnexal
fullness.
Clinical Case : Pelvic Inflammatory Disease (examination on page 63)

HOPI Obstetrical History


OCD PQRST UV + AAA Do you have children? If yes, then ask for
How did it start? Sudden or gradual. Gravidity, Term/Premature deliveries,
Is it getting worse/better or no changes in Abortions, Live/Multiple births,
the symptoms? complications in pregnancy.
Since how long have you noticed the pain? H/o ectopic pregnancy?
Where is the pain located?
Sharp or dull pain? Past History
Does the pain radiate or shoot anywhere? Do you have diabetes or hypertension?
Severity of pain on a scale of 1-10. Did you suffer from any diseases in the past
H/o painful intercourse? like tuberculosis?
Type & amount of vaginal discharge? Are you on any medications?
Do your symptoms change with time? Are you allergic to any medications?
H/o fever/cough/rash/vomiting? Any surgeries in the past?
Any chills/night sweats? Past h/o sexually transmitted infections?
Any changes in bowel & urinary habits? When was your last PAP test & results?
Any spotting per vagina?
Date of last menstrual period? Family and Social History
Do you practice safe sex? Do you smoke or consume alcohol?
Number of sexual partners? Do you use recreational drugs?
Currently in a relationship? How long?
Gynecological History Sexual orientation?
Age of onset of menses Are you at risk of abuse?
Regular/irregular? Any family history of cancers?
Duration of menses?
Amount of bleeding/passage of clots?
Any dysmenorrhea? lntra uterine devices?

Differential Diagnosis Management


Pelvic Inflammatory Disease caused due to Inj Ceftriaxone 250 mg IM stat dose with
sexually transmitted infections, IUDs. Tab Doxycycline 100 mg PO bid x 14 days.
Ovarian cyst. Reportable disease.
Endometriosis. Treat partners.
Ovarian torsion. Avoid intercourse till completion of
Acute appendicitis. treatment.
Practice safe sex.
Investigations Retest with cervical swabs after 4-6 weeks.
CBC
Vaginal & Cervical swabs, culture/sensitivity.
Urine culture/sensitivity.
Beta HCG
Pelvic ultrasound.
124 NAC OSCE |A Comprehensive Review

Julia Marshall, a 30 years old lady presented to the ER with bright red vaginal bleeding for the past 1 hour.
She is 36 weeks pregnant. Take a focused history and address her concerns.
Vitals: BP - 100/70 mm Hg. HR - 100/min. RK - 14/min.Temp - 37.5C

Clinical Info: Ms Julia Marshall is G2 T l PO AO L l at 36 weeks gestation. She has painless vaginal bleeding
for the past 1 hour. Has no contractions. Fetal heart rate is 130/minute. She has a previous history of cesarean
section.
Clinical Case : Placenta Previa

HOPI Gynecological History


OCD PQRST UV + AAA Regular/irregular menses?
How did it start? Sudden or gradual.
Is it getting worse/better or no changes in Obstetrical History
the symptoms? Do you have children? If yes, then ask for
Since how long have you noticed the Gravidity, Term/Premature deliveries,
bleeding? Abortions, Live/Multiple births,
Amount of bleeding? complications in pregnancy.
Color of bleeding? H/o ectopic pregnancy?
Number of pads changed? H/o placenta previa/ abruptio placentae?
Any passage of clots? H/o of cesarean section and the reason?
Any leaking noticed per vagina? Any surgeries on the uterus?
Any trauma recendy?
Are fetal movements felt? Past History
Any abdominal pain? Do you have diabetes or hypertension?
Any fever/nausea/vomiting? Are you on any medications?
Any changes in bowel & urinary habits? Are you allergic to any medications?
When was the last intercourse?
Date of last menstrual period? Family and Social History
Anv complications in the antenatal period? Do you smoke or consume alcohol?
When was the last antenatal visit? Do you use recreational drugs?
Any bleeding episode during pregnancy? Are you at risk of abuse?
Blood group? Any family history of medical illnesses?

Differential Diagnosis Management


Placenta previa. Admit in the hospital.
Vasa previa. Left lateral decubitus position.
Abruptio placentae. Electronic fetal heart monitoring.
Blooay show. IVF.
Maternal vital monitoring.
Investigations Inj Rhogam, if needed.
CBC, electrolytes, renal function tests. GA < 37 weeks - mild bleeding - admit &
Fetal ultrasound. observation.
Blood group, type and cross match. GA > 36 weeks - profuse bleeding with feto-
maternal compromise - Urgent Cesarean
section.
Clinical Cases - Obstetrics & Gynecology 125

Elaine Abraham, a 32 years old lady primigravida, at 34 weeks gestation presented to the ER with headache,
abdominal pain and blurring of vision. Take a focused history and address her concerns.
Vitals: BP - 150/100 mm Hg. HR - 90/min. RR - 14/min.Temp - 37.0C. FHR - 148/min.

Clinical Info: Ms Elaine Abraham has a history of pregnancy induced hypertension since 28 weeks. Her BP
is controlled by dietary restrictions and low salt intake, ohe has epigastric pain, blurring and headache for the
ast 4-6 hours. She has facial and ankle edema ++. There are no contractions. Fetal movements are felt. No
E leeding. Urine dipstick is positive for proteinuria.
Clinical Case : Pre Eclampsia

HOPI Gynecological History


OCD PQRST UV + AAA Regular/irregular?
How did it start? Sudden or gradual.
Is it getting worse/better or no changes in Obstetrical History
the symptoms? Do you have children?
Since how long have you noticed the pain? If yes, then ask for Gravidity,
Where is the pain located? Term/Premature deliveries, Abortions,
Sharp or dull pain? Live/Multiple births, complications in
Does the pain radiate or shoot anywhere? pregnancy.
Severity of pain on a scale of 1-10. H/o ectopic pregnancy?
Onset of headache? Last antenatal visit?
Location of headache?
Type of headache? Past History
Onset of blurring of vision? Do you have diabetes or hypertension?
Do your symptoms change with time? Are you on any medications?
H/o fever/cough/rash/vomiting? Are you allergic to any medications?
Any changes in bowel & urinary habits? Any surgeries in the past?
Any bleeding per vagina?
Any swelling of the body? Family and Social History
Fetal movements? Do you smoke or consume alcohol?
Any contractions/ leaking per vagina? Do you use recreational drugs?
Total weight gain in the pregnancy? Are you at risk of abuse?
Any antenatal complications like high blood Any family history of hypertension/diabetes?
pressure/diabetes/ seizures?

Diagnosis Management
Gestational hypertension with Pre Admit in the hospital.
eclampsia. Electronic Fetal monitoring.
Bed rest in left lateral decubitus position.
Investigations Hourly maternal vital signs with
CBC, electrolytes, renal function tests. intake/output charting.
Urinalysis, 24 hour urinary protein, liver Inj Magnesium sulphate 4 mg IV bolus over
function tests, uric acid, Ll )H, albumin. 20 min,then 2-4g/h for maintenance.
INR, PTT, Fibrinogen. Monitor signs for magnesium toxicity.
Non stress test, Bio-physical profile. Inj Labetalol 20-50 mg IV qlOminutes till
Fetal ultrasound. Br< 140/90 mmHg.
Deliver the baby.
RISK FACTORS FOR PIH:
Maternal: Primigravida or new paternity, Family hx of Preeclampsia, Diabetes Mellitus, Obesity, Maternal age >40 years,
Preexisting Hypertension, Anti-Phospholipid Antibody syndrome.
Fetal: IUGR, Oligohydraminos, Gtn. hydrops, Multiple pregnancy.
126 NAC OSCE |A Comprehensive Review

Michael Walter a 18 months old boy brought to your office by his mother regarding poor weight gain. Take
history from the mother & address nis concerns.

Clinical Info: Michael's mother is concerned regarding poor weight gain for his age & height. He has no
fever/nausea/vomiting/cough. No h/o recurrent infections. No urinary or bowel complaints. He's picky eater
who gets distracted while eating food. His diet consists of excessive juice & milk. No family stress present.
Clinical Case Diagnosis: Failure to thrive due to inadequate dietary intake.

HOPI Maternal malnutrition.


OCD PQRST UV + AAA Maternal exhaustion or Major Depression.
Duration of poor weight gain? Any stress at home?
Sudden or gradual decline in weight? Any signs of physical/psychological/family
Quality ana Quantity of food? abuse?
Who feeds the child? Family history of short stature/FTT in
Does the child feed self(e.g. spoon, cup)? siblings.
Psychosocial events around feeding time. Any developmental delay?
Is the child distracted or not supervised?
Are there food battles or food refusal? Birth history
Discuss food preparation (e.g. formula too Gestational age at birth and birth weight.
dilute). Mode of delivery: cesarean, induction,
Beverages (e.g. Milk, juice, water, soda). forceps or vacuum delivery.
Stool habits (e.g. frequency and consistency). Any fetal distress?Was meconium passed in
Pica history. utero?
Detailed nursing or breast feeding history. APGAR score at birth, 1 minute & 5
Infrequent brief feedings. minute?
Current weight & heignt? Was resuscitation required?
Highest weight? When was breast feeding started?
Any fever/nausea/vomiting/cough? Color of 1st stool, when was 1st stool
Any diarrhea/constipation? passed?
Any urinary complaints? Color of urine, when was 1st urine passed?
H/o recurrent infections? Any antenatal/post partum complications.
Maternal ingestion of alcohol/diuretics. Immunization history.
Inadequate milk supply. Developmental milestones.
Inadequate milk let down. Detailed dietary history.

Differential Diagnosis Karyotype.


Physiologic cause-Familial short stature. Wrist A ray.
Organic cause-Cleft palate, Choanal atresia,
GRD, Celiac ds, Giardiasis, Protein losing Management
enteropathy, Milk protein allergy, Liver ds. Complete physical examination.
Chronic diarrhea. Reassure parents.
Hyperthyroidism, Immunodeficiency. Height & weight measurement.
Prenatal causes-Intrauterine infection, Head circumference.
maternal malnutrition,Fetal alcohol Assess feeding process & parent-child
syndrome. interaction.
Chromosomal disorders. Determine & treat underlying etiology.
Institute nutritional therapy.
Investigations Eat in a comfortable, stress free environment
CBC,ESR, electrolytes, RFT, LFT. with positive reinforcement.
Blood sugar,TSH,Se Ferritin,Sweat chloride. Consume food from all four food groups.
Urinalysis-routine microscopy, C/S. Maintain dietary intake diary.
Stool for fat content,ova & parasites.
Se Calcium, Phosphate, Albumin.
Clinical Cases - Pediatrics 127

Benjamin Smith a 15 months old boy has been brought to the ER with fever and 2 episodes of seizures. He is
stabilized now. Take history & address the concerns of an over anxious mother.

Clinical Info: Beniamin Smith was having a runny nose and high grade fever for the past 3 days. His fever did
not subside with Tylenol. He had 1st episode of tonic-clonic seizure 6 hours ago at home. This was the first
occurrence. He haa no other symptoms. No family history of seizures. No complications during birth or
development so far. Immunization is up to date.
No signs of child abuse.
Diagnosis: Febrile seizures.

HOPI History of problems during the pregnancy


OCD PQRST UV + AAA and birth.
Describe seizure duration? Developmental history.
What body parts are affected and in what Child's medical history.
order, premonitory signs? Immunization history.
Post-ictal state (decrease in level of Family history of seizure disorder.
consciousness, headache, weakness). Screen for signs of child abuse.
Previous seizure?
Events during the seizure time?
How did the seizure stop?
Onset of fever? Sudden or gradual?
Duration of fever?
Type of fever-continuous, remittent,
intermittent.
Any nausea/vomiting?
Any ear/eye discharge/runny nose?
Any rash?
Any cough/sore throat/difficulty swallowing?
Any difficulty breathing?
Any bowel or urinary complaints?
Any sick contacts?
Ask about preceding trauma or illness or
medications taken?

Differential Diagnosis Management


Febrile seizure. Symptomatic treatment.
Meningitis. Antipyretics for fever prn.
Encephalitis. Maintain hydration.
Counseling & reassurance for parents.
Investigations Recurrence - rectal or sublingual Lorazepam.
CBC, electrolytes, RFT. Treat underlying cause of fever.
ABG, Blood glucose.
Urinalysis.
Blood culture &c sensitivity.
LP-gram stain, culture Sc sensitivity.
128 NAC OSCE |A Comprehensive Review

Nick Chang is a 15 years old boy brought by his mother with fever and rash for the past 2 days.
Take history & address her concerns.

Clinical Info: Nick has high fever for the past 2 days. He has developed a diffuse rash in the last 24 hours
which is spreading from head to trunk. He also has cough, sore throat and redness of eyes. He has no altered
level of consciousness/irritability. He is alert and feeding well. Has h/o sick contacts with similar complaints
in the day care. His immunization is up to date.
Diagnosis: Measles.

HOPI Immunization history till date?


OCD PQRST UV + AAA Any recurrent infections?
Onset of fever- sudden or gradual? Development milestones for age?
Duration of fever? Any similar symptoms in the past?
Type of fever-
continuous,remittent,intermittent?
Highest recorded temperature?
Relieving factors for the fever?
Onset of rash?
Type of rash?
Location of rash?
Rash becoming better or worse?
Any vesicles noticed with the rash?
Any swelling in the body?
Any ear/eye discharge?
Any excessive crying/irritability?
Any changes in alertness of the child?
Any cough/sore throat/runny nose?
Any nausea/vomiting/difficulty swallowing?
Any changes in urinary/bowel habits?
Feeding well or not?
Any seizures?
Any recent sick contacts?
Any travel?

Differential Diagnosis Management


Measles. Symptomatic treatment.
Rubella. Maintain adequate hydration.
Varicella zoster. Rest.
Erythema infectiosum. Antipyretics for fever prn.
Educate parents about complications.
Investigations
CBC with differential.
Clinical Cases - Pediatrics 129

Marie Jones delivered baby Anthony 36 hours old and now the newborn has jaundice, lethargy and crying.
The serum bilirubin is 220 mmol ( N < 200). Take history & address her concerns.

Clinical Info: Anthony was born to a primigravida by normal vaginal delivery. Mother noticed yellowish
discoloration of his eyes in the morning. She had no antenatal complications. She had premature rupture of
membranes prior to onset of labor at 38 weeks. She was put on antibiotics. Her labor was 18 hours long. The
labor was induced. Apgar was 9/10. Baby is a little lethargic and not feeding well. Has no fever/altered
consciousness. No seizures.
Clinical Case Diagnosis: Neonatal Jaundice due to Sepsis.

HOPI APGAR score at birth, 1 minute & 5


Mother's obstetrical history: minutes?
GTPAL Was resuscitation required?
H/o neonatal jaundice in past pregnancies. When was breast feeding started?
Maternal medical history esp. liver disease. Is the baby feeding well?
Illness during pregnancy esp. diabetes, Color of 1st stool, when was 1st stool
rubella, toxoplasmosis, hemes, CMV. passed?
Teratogenic medications during pregnancy. Color of urine, when was 1st urine passed?
Radiation exposure in pregnancy? Any vomiting/regurgitation?
Drug and alcohol use during pregnancy? Decreased neonatal muscle tone?
Any pets in the house? Any fever, irritability, lethargy,seizure?
Maternal & Paternal blood type.
Complications of present pregnancy. Past History
- Gestational hypertension or diabetes, Do you have diabetes or hypertension?
hyper/hypothyroid, hypercoagulation. Are you on any medications?
Any antenatal/post partum complications? Are you allergic to any medications?
Any surgeries in the past?
Newborn history: Past h/o recurrent infections?
Gestational age at birth,
Mode of delivery: cesarean, induction, 9 Family and Social History
forceps or vacuum delivery. Social support.
Duration of rupture of membranes (ROM)? Any family history of medical illnesses?
Was ROM artificial or prolonged?
Any fetal distress?Was meconium passed in
utero?

Differential Diagnosis Management


Sepsis. Prevent kernicterus.
Breast feeding jaundice. Treat underlying cause.
Hemolysis. Monitor neonatal vitals.
Physiologic jaundice. Maintain hydration.
Initiate phototherapy.
Investigations Reassurance for parents.
CBC with reties,electrolytes, RFT.
Se bilirubin(conjugated & unconjugated) &
albumin.
Blood group (maternal, paternal &
neonatal).
Coomb's test.
Blood & urine culture.
TSH, G6PD.
Chest X Ray, LP.
130 NAC OSCE |A Comprehensive Review

Sean Radcliffe is a 8 years old boy whose parents have concern about bed wetting. Take history from the
father Sc address his concerns.

Clinical Info: Sean has been wetting his bed since the last 3 years. He never had bladder control. He has no
fever/vomiting. No h/o recurrent infections. He wets bed 2-3 times in the night. No day time wetting present.
No encoparesis. Parents have not taken any treatment so far and have tried toilet training in past with no
success. No stresses at home or school.
Clinical Case Diagnosis: Primary nocturnal enuresis.

HOPI Enuresis in other siblings?


OCD PQJtST UV + AAA Any stresses at home or school?
Type of voiding - Involuntary or intentional. Any new habits or regression to old habits?
Number of times wets bed in the night? Trial of toilet training in the past?
Has the child ever been dry?(primary or Any treatment in the past for enuresis?
secondary)
Is there aaytime Enuresis?(complicated Birth history
Enuresis) Gestational age at birth and birth weight.
Wetting pattern - daynight or night only. Mode of delivery: cesarean, induction,
Any dysuria/pyuria/foul smelling urine? forceps or vacuum delivery.
Involuntary passage of stool in the sleep? Any fetal distress?Was meconium passed in
Functional bladder disorder signs like utero?
- Voids >7 times per day with urgency Sc in APGAR score at birth, 1 minute Sc 5
small volumes. minute?
- Withholds urine until last minute, wets Was resuscitation required?
more than once nightly. When was breast feeding started?
Has enuresis on only a few nights per week? Color of 1st stool, when was 1st stool
Voids large volumes when enuresis occurs? passed?
Bowel or bladder habit changes recently. Color of urine, when was 1st urine passed?
Infrequent or difficult stool passage? Any antenatal/post partum complications.
Any changes in appetite/weight? Immunization history.
Any fever/nausea/vomiting?
Any recurrent infections?
Amount of fluid intake prior to sleep?
Any neurological disorders?
Any genitourinary surgeries?

Differential Diagnosis Management


Primary nocturnal enuresis. Complete physical examination.
Urinary tract infection. Reassure parents.
Urinary tract anomalies like small bladder. Schedule voiding times.
Psychological (death in the family, sexual Bed wetting alarm.
abuse). Void before bedtime.
Limit fluids 1 hour before bedtime.
Investigations Voiding diary to be maintained.
CBC,electrolytes, RFT, LFT. Positive reinforcement for dry nights.
Blood sugar. Pharmacological therapy like
Urinalysis-routine microscopy, C/S. Imipramine/DDAVP.
Ultrasound abdomen.
Clinical Cases - Pediatrics 131

Ally Singer's 6 weeks old baby boy Alex is vomiting for the past 2 days. Take history 8c address her concerns.

Clinical Info: Alex had 4 episodes of projectile non bilious vomiting in the past 48 hours. He vomits after
feeding. No fever. Looks lethargic 8c dehydrated but alert. No seizures. Had only one bowel movement in last
24 hours. No sick contacts. O/E: Palpable abdominal mass in the right hypochondrium.
Clinical Case Diagnosis : Pyloric stenosis.

HOPI Mother's obstetrical history


OCD PQRST UV + AAA GTPAL
Number of episodes of vomiting? Maternal medical history esp. liver disease.
Duration of vomiting? Illness during pregnancy esp. diabetes,
Type of vomiting - projectile/non projectile? rubella, toxoplasmosis, herpes, CMV.
Color/contents of vomitus? Teratogenic medications during pregnancy.
Any excessive crying? Radiation exposure in pregnancy.
Feeding pattern in the last 48 hours? Drug and alcohol use auring pregnancy.
Decreased neonatal muscle tone? Complications of present pregnancy.
Any fever, irritability, lethargy,seizure? - Gestational hypertension or diabetes,
Last bowel movement? - hyper/hypothyroid, hypercoagulation.
Foul smelling urine 8c color of urine? Any antenatal/post partum complications?
Current weight.
Any sick contacts.

Newborn history
Gestational age at birth and birth weight.
Mode of delivery: cesarean, induction,
forceps or vacuum delivery.
Any fetal distress? Was meconium passed in
utero?
APGAR score at birth, 1 minute 8c 5
minute?
Was resuscitation required?
When was breast feeding started?
H/o neonatal jaundice.
Color of 1st stool, when was 1st stool
passed?
Color of urine, when was 1st urine passed?

Differential Diagnosis Management


Pyloric stenosis. Admit.
Tracheo-esophageal fistula. Urgent Pediatric surgery consult.
Duodenal atresia. IVF to maintain hydration.
Malrotation of gut. Surgery - Pyloromyotomy.
Gastro-esophageal reflux.
Investigations
CBC,electrolytes, RFT, LFT.
ABG.
Urinalysis.
Ultrasound abdomen.
Abdominal X ray.
132 NAC OSCE |A Comprehensive Review

John Andrews is a 3 years old boy who is not speaking well. Take history & address his father's concerns.

Clinical Info: John Andrews has h/o recurrent ear infections. He had 3 episodes in the last 6 months. He has
runny nose and mild cough too. He can speak in sentence of 3-4 words. He can count to 5. But for the past 3
months he is not learning new words or numbers. He responds to loud sounds. No other complaints. Social
interaction is very good. No birth or developmental complications till date.
Diagnosis: Speech delay secondary to recurrent otitis media.

Duration of speech delay?


Who noticed it first?
Any ear discharge/runny nose?
Any recurrent infections?
Any fever/cough/sore throat?
Does the child wake up in response to sounds?
Startle to loud sounds?
Comes when called?
Understands spoken instructions?
Ask about swimming.
Enquire about verbal cues.
How many languages are spoken in the household?
Child's social interaction with others.
Does the child talk less in particular situations?
How many words are spoken by the child?
Detailed developmental history.
Was the child screened for hearing at birth?
Any regression in habits?
Immunization history till date.
Family history of speech delay.
Any complications during pregnancy or birth?
Detailed birth history.
Exposure to toxins during pregnancy?
Any ototoxic drugs used in infancy?
Screen for signs of child abuse.

Differential Diagnosis Management


Hearing loss secondary to Otitis media. Reassurance for parents.
Selective mutism. Complete physical assessment.
Expressive speech delay. ENT referral.
Autism. Speech therapy.
Positive reinforcement & encouragement.
Investigations
Hearing test like Pure Tone Audiometry.
Clinical Cases - Psychiatry 133

Gabriella Anderson, a 18 years old girl came to your office with complaints of gaining weight. Take history 8c
counsel.
Clinical Info: Ms Gabriella Anderson presented with gaining 5 lbs in the last 1 month. She looks
underweight for her age and height. She is exercising 3 times a day. She doesnt binge or induce vomiting.
Lately she is taking small portions of meals due to fear of gaining weight. She has no medical illnesses. No
past nistory of psychiatric illness. Currently not taking any medications.
Clinical Case : Anorexia

HOPI Do you take any medications? Ask details.


When did you notice the change in weight? Do you consume alcohol?
Duration of symptoms? Amount/frequency?
Amount of weight gain? Do you use recreational drugs? Ask
Lowest and highest weight you had? TRAPPED.
Are you afraid of gaining weight?
How do you try to control your weight? Past Psychiatric History
How do you think your body looks? Any similar symptoms in past?
Does your body weight 8c shape have an Any h/o mania /depression / delusion /
impact on your self opinion? delirium?
Last menstrual period /regularity / Any contacts with mental health
complications? professionals?
Any abdominal pain/nausea/vomiting? Any past problems with law?
Bowel and bladaer habits?
Any skin changes? Family Histoiy
Any intolerance to temperature changes? Any family history of similar complaints?
Any recent stressors at nome or work? Any family history of other psychiatric
Changes in sleep pattern? illnesses?
Feeling of guilt /hopelessness /helpless / Any family history of suicide/alcohol/drug
worthless? abuse?
Changes in mood?
Any thoughts of harming self/suicide? Social History
Any thoughts of harming someone else? Support system at home/work?
Any plans at the moment? Current living situation?
Do you feel persistently cheerful/high? Relationship nistory?
Do you have any medical/surgical illnesses? Education nistory?
Ask details. Any risk of physical/sexual/mental abuse?

Differential Diagnosis Management


Anorexia nervosa. Complete physical assessment.
Bulimia nervosa. Antiaepressants.
Mood disorders. Suj3j|>ortive psychotherapy.
Medical cause of weight loss.
Body Dysmorphic disorder. Make plans for weight gain.
Community resources for eating disorders.
Investigations Educational brochures.
CBC, electrolytes, renal 8c Liver function Admit if weight <85% of ideal weight.
tests. HR < 40 bpm.
TSH, blood glucose, ECG. Hypovolemia.
Urine toxicology screen. Hypokalemia
Beta HCG, LH,FSH. Hypoglycemia.
Hypothermia.
134 NAC OSCE |A Comprehensive Review

Amanda Sawyer, a 20 years old girl brought to your office by her mother for vomiting and weight loss. Take
history & counsel.

Clinical Info: Ms Amanda Sawyer presented with vomiting after meals. She has fear of weight gain. H/o
binging & induced vomiting present. H/o laxative abuse and excessive exercise. She has no apparent psycho
motor or suicidal ideation. Sne has no medical illnesses. No past history of psychiatric illness. Currendy not
taking any medications.
Clinical Case : Bulimia

HOPI Do you take any medications? Ask details.


Onset of vomiting? Do you consume alcohol?
Duration of symptoms? Amount/frequency?
Type of vomitus/contents? Do you use recreational drugs? Ask
Projectile/non projectile? TRAPPED.
H/o abdominal pain/site/type?
Do you force yourself to vomit? Past Psychiatric History
Do you binge large amount of food? Any similar symptoms in past?
Amount of weight gain/loss? Any h/o mania /depression /delusion /
Lowest and highest weight you had? delirium?
How do you try to control your weight? Any contacts with mental health
Does your body weight & shape have an professionals?
impact on your self opinion? Any past problems with law?
Last menstrual period /regularity /
complications? Family Histoiy
Any skin changes? Any family history of similar complaints?
Any intolerance to temperature changes? Any family history of other psychiatric
Any recent stressors at nome or work? illnesses?
Changes in sleep pattern? Any family history of suicide/alcohol/drug
Feeling of guilt /hopelessness /helpless / abuse?
worthless?
Changes in mood? Social History
Any thoughts of harming self/suicide? Support system at home/work?
Any thoughts of harming someone else? Current living situation?
Do you feel persistently cheerful/high? Relationship nistory?
Do you have any medical/surgical illnesses? Education nistory?
Ask details. Any risk of physical/sexual/mental abuse?

Differential Diagnosis Management


Bulimia nervosa. Complete physical assessment.
Anorexia nervosa. Antidepressants.
Mood disorders. Suj3|)ortive psychotherapy.
Medical cause of weight loss.
Body Dysmorphic disorder. Make plans for weight gain.
Community resources for eating disorders.
Investigations Educational brochures.
CBC, electrolytes, renal & Liver function Admit if weight <85% of ideal weight.
tests. HR < 40 bpm.
TSH, blood glucose, ECG. Hypovolemia.
Urine toxicology screen. Hypokalemia
Beta HCG, LH,FSH. Hypoglycemia.
Hypothermia.
Clinical Cases - Psychiatry 135

Derek Paul, a 65 years old man admitted in surgical floor presented with strange behavior for the past 4
hours. You are on call surgical resident for the shift. Take nistory & counsel.

Clinical Info: Mr Derek Paul had partial right hip replacement 3 days ago. His post op recovery till now has
been uneventful. Evening shift nurse noticed significant change in his behavior. He is agitated, resdess with
acute memory loss. He is disoriented to time, place 8c person. He is having delusional thoughts of ants
crawling. He is on oral antibiotics, antihypertensives, blood thinners. He is a chronic alcohol abuser.
Clinical Case : Delirium

HOPI Past Psychiatric History


When did you notice the change in Any similar symptoms in past?
behavior? Any h/o mania / depression /delusion /
Duration of symptoms? delirium?
Symptoms have worsened or improved? Any contacts with mental health
Do you feel persistently cheerful/high? professionals?
Do you sense things that others around you Any past problems with law?
don t?
Do you hear any voices? Family History
Do you feel restless/agitated? Any family history of similar complaints?
Any changes in memory? Any family history of other psychiatric
Orientation to time/place/person? illnesses?
Any fever/nausea/vomiting? Any family history of suicide/alcohol/drug
Any chest pain/ abdomind pain? abuse?
Any changes in bowel/urinary habits?
Last meal? Social History
Any changes in mood? Support system at home/work?
Any thoughts of harming self/suicide? Current living situation?
Any thoughts of harming someone else? Relationship nistory?
Any plans at the moment? Education history?
Do you have any medical/surgical illnesses? Any risk of physical/sexual/mental abuse?
Ask details.
Do you take any medications? Ask details. T : Treatment history
List of current medications. R : Route
Do you consume alcohol? A : Addiction /toxicity /Amount
Amount/frequency? P : Pattern of use
Do you smoke? Duration/frequency? P : Prior abstinence
Do you use recreational drugs? Ask E : Effects of drug
TRAPPED. D: Duration of use

Differential Diagnosis Management


Acute Delirium. Complete physical examination.
Alcohol withdrawal. Re-evaluate current medications.
Medication induced delirium. Maintain hydration.
Sepsis. Avoid sedatives,narcotics,BZD.
Place in a quiet, well lit room.
Investigations Vital qlhourly.
CBC, electrolytes, renal function tests. Inj Haloperidol 5 mg IM stat with Inj
TSH, blood glucose, blood gases. Lorazepam 2mg IJVlstat.
Urinalysis with toxicology screen. Reassurance.
Blood alcohol levels.
MSE/MMSE.
136 NAC OSCE |A Comprehensive Review

Claire Wiggins, a 72 years old lady brought to your office by her son with strange behavior. Take history from
the patient and address her concerns.

Clinical Info: Ms Claire Wiggins is forgetting things and daily tasks for the past 1 year. Her symptoms have
become worse for the last 6 months. Recently she forgot her way back home. She lives alone. Son has noticed
changes in her dressing and poor hygiene. She has no apparent psycho-motor or suicidal ideation. She has
hypertension. No past history of psychiatric illness. Currently on oral antihypertensives, statins, zoloft,
multivitamins.
Clinical Case : Dementia

HOPI Do you smoke? Duration/frequency?


When did you notice the change in Do you use recreational drugs? Ask
behavior? TRAPPED.
Duration of symptoms?
Have you found yourself forgetting things? Past Psychiatric History
(establish onset, duration, degree) Any similar symptoms in past?
Do you ever get confused or disorientated? Any h/o mania/ depression/ delusion/
Do you have trouble understanding what delirium?
people say to you? Any contacts with mental health
Do you have trouble finding the right words professionals?
to say? Any past problems with law?
Symptoms getting gradually worse over
months? Family History
Have you seen, heard or felt anything that Any family history of similar complaints?
other people told you didnt exist? Any family history of other psychiatric
Have you noticed a change in your sense of illnesses?
smell? Any family history of suicide/alcohol/drug
Have you had any incontinence? abuse?
Any cnanges in gait?
Any mood changes? Social History
Any fever/nausea/vomiting/abdominal pain? Support system at home/work?
Has there been a change in personality? Current living situation?
Do you have any medical/surgical illnesses? Relationship nistory?
Ask details. Education history?
Do you take any medications? Ask details. Any risk of physical/sexual/mental abuse?
Do you consume alcohol? Activities of daily living & Instrumental
Amount/frequency? Activities of daily living.

Differential Diagnosis MRI/CT Head.


Alzheimer's disease. MSE/MMSE.
Multi-Infarct dementia.
Dementia with Lewy bodies. Management
Depression. Complete physical examination.
Polypharmacy. Involve social worker.
Living environment assessment.
Investigations Educational brochures.
CBC, electrolytes, renal function tests. Access to community resources.
Liver function tests,Vit B12 levels. Orientation cues: clocks,calendars,notes.
TSH, blood glucose, ECG. Resources for the family.
Urinalysis, urine toxicology screen. Acetylcholinesterase inhibitors.
Safely home registry.
Clinical Cases - Psychiatry 137

Julian Smith, a 56 years old lady brought to your office by her husband with strange behavior. Take history &
counsel.

Clinical Info: Ms Julian Smith has h/o of change in mood for the past 1 month after loosing her job. She has
changes in mood, sleep and appetite. She has lost 10 lbs in the last 1 month. She has lack ofinterest in social
activities. She has no apparent psycho-motor or suicidal ideation. She has no medical illnesses. No past
history ofpsychiatric illness. Currently not taking any medications.
Clinical Case : Depression

HOPI Do you take any medications? Ask details.


When did you notice the change in Do you consume alcohol and smoke?
behavior? Amount/frequency?
Duration of symptoms? Do you use recreational drugs? Ask
Symptoms have worsened or improved? TRAPPED.
Any recent stressors at home or work?
Changes in sleep pattern? Past Psychiatric History
H/o lack of interest recendy? Any similar symptoms in past?
Feeling of guilt/hopelessness/ Any h/o mania/depression/delusion/
helpless/worthless? delirium?
Lack of energy? Any contacts with mental health
Changes in mood? professionals?
Are you crying a lot? Any past problems with law?
Is your mood always low or it alternates?
Any changes in concentration? Family History
Any changes in appetite? Any family history of similar complaints?
Any changes in weight? Any family history of other psychiatric
Any changes in memory? illnesses?
Do you feel slowed down? Any family history of suicide/alcohol/
Do you feel resdess/agitated? drug abuse?
Any thoughts of harming self/suicide?
Any thoughts of harming someone else? Social History
Any plans at the moment? Support system at home/work?
Do you feel persistendy cheerful/high? Current living situation?
Do you sense things that others around you Relationship history?
dont? Education history?
Do you have any medical/surgical illnesses? Any risk of physical/sexual/mental abuse?
Ask details.

Differential Diagnosis Management


Major Depressive Episode. Start SSRI.
Adjustment disorder with depressed mood. Psychotherapy/CBT.
Bipolar affective disorder. Educational brochures about depression.
Anxiety disorder. Info about the side effects of medications.
Emphasis on long term management,follow
Investigations up and compliance.
CBC, electrolytes, renal function tests. Referral to community resources.
TSH, blood glucose, urinalysis. Contract: Pt to contact you/family doctor/
MSE/MMSE. nearest emergency/friend/family/crisis help
line in case of suicidal or homicidal ideation.
138 NAC OSCE |A Comprehensive Review

David Rosenberg, a 26 years old man brought to the ER by the police because he was throwing stones at a
public building. Take history & counsel.

Clinical Info: Mr David Rosenberg presented with irrational behavior for the past 10 days. He is having
racing thoughts, increased activity, decreased sleep and increased vocalization. He has constant flight ofideas
during the interview with easy distractibility. He is restless while sitting and at times agitated. No medical
illnesses but is a chronic cocaine abuser.
Clinical Case : Mania

HOPI Do you have any medical/surgical illnesses?


When did you notice the behavioral Ask details.
changes? Do you take any medications? Ask details.
Duration of symptoms? Do you consume alcohol and smoke?
Symptoms have worsened or improved? Amount/frequency?
Any recent stressors at home or work? Do you use recreational drugs? Ask
Do you get easily distracted while talking or TRAPPED.
working?
Did you have reckless driving & drinking Past Psychiatric History
episode? Any similar symptoms in past?
Have vou gone for shopping sprees/ Any h/o mania/depression/delusion/
gambling/ excessive spending? delirium?
Do you feel like a special person with special Any contacts with mental health
talent, power or on a mission? professionals?
Do people say you jump from topic to topic? Any past problems with law?
Did anyone say that you have increased
activity/energy/appetite/resdess/agitation? Family Histoiy
Changes in Seep pattern? Any family history of similar complaints?
Did anyone say that you are more talkative Any family history of other psychiatric
than usual? illnesses?
H/o lack of interest recendy? Any family history of suicide/alcohol/drug
Feeling of guilt/hopelessness/helplessness/ abuse?
worthlessness?
Changes in mood? Social History
Any thoughts of harming self/suicide? Support system at home/work?
Any thoughts of harming someone else? Current living situation?
Any plans at the moment? Relationship history?
Do you sense things that others around you Education history?
don t? Any risk of physical/sexual/mental abuse?

Differential Diagnosis Management


Cocaine induced Maniac episode. Start Mood stabilizers - Lithium/
Bipolar disorder. Valproate /CBZ.
Personality disorder. Psychotherapy/CBT.
Substance abuse. Admit in case of acute mania.
Info about the side effects of medications.
Investigations Emphasis on long term management,follow
CBC, electrolytes, renal function tests. up and compliance.
TSH, blood glucose, urinalysis. Referral to community resources.
VDRL, HI vT Contract: Pt to contact you/ family doctor/
Urine toxicology screen. nearest emergency/friend/family/crisis help
Urgent psychiatric consult. line in case of suicidal or homicidal ideation.
Clinical Cases - Psychiatry 139

Brad Daniels, a 22 years old man came to your office with light headedness, trembling and chest pain for the
past 4 hours. Take nistory & counsel.
Clinical Info: Mr Brad Daniels is a university student who presented with sudden onset of light headedness,
trembling of body and chest pain prior to his presentation in class. He also complaints of palpitations and
shortness of breath. He had similar episodes in the past. No past history of psychiatric or medical illnesses.
Not taking any medications currently.
Clinical Case : Panic Attack

HOPI Worry about consequences of another


When did you notice the symptoms? attack?
Duration of symptoms? Any thoughts of harming self/suicide?
Symptoms have worsened or improved? Any thoughts of harming someone else?
Any recent stressors at home or work? Do you have any medical/surgical illnesses?
Do you have excessive sweating? Do you take any medications? Ask details.
Do you experience tremors? Do you consume alcohol/smoke?
Do you have unsteadiness/dizziness? Amount/frequency?
Do you sense things that others around you Do you use recreational drugs? Ask
don t? (Derealization) TRAPPED.
Do you ever feel you are outside of yourself?
(Depersonalization) Past Psychiatric History
Do you have excessive heart rate? Any similar symptoms in past?
Nausea/vomiting? Any h/o mania/ depression/ delusion/
Any tingling/paresthesias? delirium?
Any shortness of breath? Any contacts with mental health
Any fear of dying/loosing control/going professionals?
crazy? Any past problems with law?
Do you have chest pain?
Do you have chills/choking sensation? Family Histoiy
Changes in sleep pattern? Any family history of similar complaints?
Feeling of guilt/ hopelessness/ helplessness/ Any family history of other psychiatric
worthless? illnesses?
Any changes in mood? Any family history of suicide/alcohol/drug
Any changes in concentration? abuse?
Any changes in appetite/weight?
Any changes in bowel habits? Social History
Any changes in memory? Support system at home/work?
Do you feel slowed down? Current living situation?
Do you feel restless/agitated? Relationship nistory?
Amount of caffeine intake? Education history?
Any anticipatory anxiety? Any risk of physical/sexual/mental abuse?

Differential Diagnosis Management


Panic Attack. Start SSRI.
Panic disorder. Psychotherapy/CBT.
Anxiety disorder. Educational brochures about anxiety attacks.
Mood disorder. Info about the side effects of medications.
Emphasis on long term management,follow
Investigations up and compliance.
CBC, electrolytes, renal function tests. Muscle relaxation, deep breathing &
TSH, blood glucose, urinalysis. biofeedback.
ECG. Contract: Pt. to contact you/family doctor/
nearest emergency/friend/family/crisis help
line in case of suicidal or homicidal ideation.
140 NAC OSCE |A Comprehensive Review

Liam Pinkerton, a 24 years old male was brought to the ER with complaints of alien attacks . Take history &
counsel.
Clinical Info: Mr Liam Pinkerton is brought by police with complaints of being attacked by aliens in the last
48 hours. He is talking to himself and avoiding direct eye contact. He is restless and agitated and feels
threatened. He is hearing strange voices for the past 1 month along with disorganized speech and behavior.
He is a chronic cocaine user for the past 3 years and increased consumption in last 48 hours.
Clinical Case : Schizophrenia

HOPI Get emotional or excited about things you


Thought phenomena: are used to?
Think your thoughts are interfered with in Have drive and ambition for anything at the
any way? moment?
Think others can read your thoughts? Any mood changes recendy?
Think an outside entity is affecting your Do you feel restless/agitated?
thoughts? Any changes in appetite/weight?
Hear echo's of your thoughts like a voice? Any fever/nausea/vomiting/trauma?
Delusions: Currendy on any medications?
Anything bothering you at the moment? Do you consume alcohol?
Ever felt that someone is out to get you? Amount/frequency?
Different understanding of things to other Do you smoke? Duration/frequency?
people? Do you use recreational drugs? TRAPPED?
Do you get any special information from Past Psychiatric History
things which happen? Any similar symptoms in past?
Do you get special messages or Any h/o mania/ depression/ delusion/
communication from newspapers, radio, delirium?
television or any other source? Any contacts with mental health
Delusions of control; influence or passivity: professionals?
Do you feel that a force can control or Any past problems with law?
influence you?
Think that some of your actions or thoughts Family History
are not being controlled by you? Any family history of similar complaints?
Hallucinations: Any family history of other psychiatric
Hear voices when there is no one present? illnesses?
Does someone talk to you? Any family history of suicide/alcohol/drug
How many? Who are they? What do they abuse?
say? Social History
Do they ever tell you to harm/kill yourself or Support system at home/work?
others? Current living situation?
How often do you hear voices? Relationship nistory?
How does it make you feel? Education nistory?
Can you control or stop them? Any risk of physical/sexual/mental abuse?
Blunted affect /Apathy, loss of drive?

Differential Diagnosis Management


Schizophrenia. Admit in hospital under FORM 1.
Substance induced Acute Psychosis. Start Anti-Psvchotics.
Acute Delusional disorder. Inj Haloperiaol 5mg IM stat.
Schizoaffective disorder. Urgent psychiatric consultation.
Investigations Detoxification program.
CBC, electrolytes, renal function tests. Involve social worker.
Liver function test, blood gases.
Blood glucose, urinalysis.
Urine toxicology screen.
Clinical Cases - Psychiatry 141

Erica McCain is a 16 years old girl brought to the ER with ASA overdose. She is stabilized now. Take
history & counsel.
Clinical Info: Ms Erica McCain a 16 years old school going girl took 30 tabs of Aspirin after smashing her
arents car in a tree. She attempted to commit suicide to prevent embarrassment. She went to her friend's
Eouse after the accident. Her grandma brought her to the ER. Has h/o previous attempt 1 year ago. Is
currendv consulting a psychiatrist on a regular basis. Presendy on antidepressants. Show EMPATHY!
Clinical Case : Suicide

HOPI Do you use recreational drugs? Ask


Analyze the attempt- TRAPPED.
Vvhen /What method / Source of
method / Circumstances which lead to Past Psychiatric History
the attempt Any similar attempts in the past?
What were your thoughts while hurting Outcome/admission, if any?
yourself? Treatment given for such attempts?
What did you think would be the outcome? Any h/o mania/ depression/ delusion/
Changes in mood? delirium?
H/o lack of interest recently? Any contacts with mental health
Feeling of guilt/hopelessness/helplessness/ professionals?
worthlessness? Any past problems with law?
Lack of energy/concentration?
Is your mooa always low or it alternates? Family Histoiy
Any changes in appetite/weight? Any family history of suicidal attempts?
Do you feel restless/agitated? Any family history of other psychiatric
Any thoughts of harming someone else? illnesses?
Any plans at the moment? Any family history of suicide/alcohol/drug
Do you sense things that others around you abuse?
dont?
Do you have any medical/surgical illnesses? Social History
Ask details. Support system at home/work?
Do you take any medications? Ask details. Current living situation?
Do you consume alcohol? Relationship nistory?
Amount/frequency? Education history?
Do you smoke? Frequency/duration? Any risk of physical/sexual/mental abuse?

Assess Suicidal risk: SAD PERSONS


Suicidal Attempt. Sex - Male> female
Age - Bimodal: 15-25 years and > 65 years.
Investigations Depression
CBC, electrolytes, renal function tests. Previous attempts
Liver function tests, Blood gases. Ethanol use
Se. salicylate levels q2h till levels fall. Rational thinking
Blood glucose, urinalysis. Suicide in family
Organized plan
Management No support
Admit under FORM 1. Sickness
Gastric Lavage. Based on the score from the scale:
Urine alkalinization. 0-2 - Send home with family.
Urgent psychiatric evaluation. 3-4 - Close follow up, consider admission.
Arrange family meeting and involve social 5-6 - Strongly consider admission.
worker. 7-10 - Admit.
Start Lithium or Clozapine.
142 NAC OSCE |A Comprehensive Review

Brandon Rodrigues, 28 young man comes with recent onset of back pain and limp. Take focused history and
preform a focused examination.

Clinical info: Mr Brandon Rodrigues had a sudden onset of sharp lower back pain 2 days ago after lifting
heavy boxes at home. Pain is located in the lumbar area, grade 8/10 and is constandy present. He has
numbness and paresthesias present in his left leg for the past 12 hours. No weakness or loss of sensation in
the lower limbs. No urinary retention or bowel incontinence. He does not smoke or consume alcohol. O/E
there is tenderness in the L4 - L5 area & decreased sensation in the L4 - L5 dermatomal distribution.
Clinical Case : Back Pain

HOPI Past History


OCD PQRST UV + AAA Do you have diabetes or hypertension?
How did it start? Sudden or gradual. Are you on any medications?
Is it getting worse/better or no changes in Are you allergic to any medications?
the symptoms? Any surgeries in the past?
Since how long have you noticed the back
pain? Family and Social History
Where is the pain located? (Pain worse in Do you smoke? Duration & frequency.
back than in buttocks or legs suggests Do you consume alcohol? Duration oc
mechanical back pain. Pain worse in buttocks frequency.
suggests radiculopathy) Do you use recreational drugs? TRAPPED.
Type of pain -sharp or dull? Any family history of cancers/ medical
Does the pain radiate or shoot anywhere? illnesses?
Severity of pain on a scale of 1-10. Erectile dysfunction (Cauda equina
Do your symptoms change with time? syndrome)
Any aggravating or relieving factors? (Pain
worse lying down and bilateral leg weakness RED FLAGS (BACKPAIN)
suggests spinal stenosis or ankylosing Bowel or Bladder dysfunction
spondylosis, ) Anesthesia (Saddle; - perineal numbness
Effect on activities of daily living, functional Constitutional Symptoms - Malignancy
limitation? K : Chronic disease
H/o fever/ fatigue /weight loss/night Paresthesias
sweats? Age > 50
Burning micturation, joint pain, uveitis? IV drug users
Morning stiffness? Associated numbness, Neuromotor deficits
weakness?
Bladder retention /bowel incontinence?

Differential Diagnosis Management


Disc herniation. Reassurance and education if no cause (70%
Spondyloarthropathy. improve in 2 weeks, 90% in 6 weeks)
Radiculopathy. Limited bed rest.
Mechanical back pain. Activity modification.
Heat/Cold therapies.
Investigations Low stress aerobic activities in first 2 weeks.
Lumbo-sacral X-ray. Encourage early return to work/ activities.
MRI lumbo-sacral area NSAIDS/ Acetaminophen.

Surgery indicated in
Cauda Equina.
Worsening neurological deficit.
Intractable pain not responding to
conservative treatment.
Clinical Cases - Surgery________________________________________________________ 143

Nicole Davy, a 75 years old lady presented with enlarging mole on her nose .Take a focused history and
perform focused physical examination.
Vitals: BP - 120/88 mm Hg, HR - 86/min, RR - 12/min,Temp - 37.0C

Clinical Info: Ms Nicole Davy has an enlarging mole on her nose which is changing in color and shape over
the past 1 month. She is a Caucasian retired woman who spends 6 months in Florida during winters in
Canada. Recently noticed irregular edges of her mole and got concerned. She worked as a radiation technician
for 30 years prior to retirement. She had a similar mole which was cancerous and removed 10 vears ago. O/E:
There is a small 0.5 x 0.5 cm pearly papule on her lateral left side of nose, with irregular rollea out margins
and minimal discharge.
Clinical Case Diagnosis: Basal Cell Carcinoma.

HOPI Past History


OCD PQRST UV + AAA Do you have diabetes or hypertension?
How did it start? Sudden or gradual. Are you on any medications?
Is it getting worse/better or no changes in Are you allergic to any medications?
the symptoms? Any surgeries in the past?
Duration of the mole? Past h/o recurrent infections?
Location of the mole?
What changes have you noticed in the mole? Family and Social History
Any itching? Do you smoke? Duration & frequency.
Any ulceration? Do you consume alcohol? Duration &
Any discharge or bleeding noticed? frequency.
Any change in color of the mole? Do you use recreational drugs? TRAPPED.
Any other suspicious moles elsewhere? Any family history of cancers/ medical
Amount of sun exposure? illnesses?
Any radiation exposure? Occupation?
Any swelling noticed in the body?
Any fever/nausea/vomiting/decreased
appetite?
Any weight loss/night sweats?
Any headache/chest pain/bony pain?
Any visual changes?
Any changes in bowel & urinary habits?

Differential Diagnosis Management


Basal Cell Carcinoma. Complete physical examination.
Squamous Cell Carcinoma. Electrodessication & curettage.
Nodular malignant melanoma. Surgical excision microscopically
Intradermal melanocytic nevus. controlled surgery.
Cryotherapy.
Investigations 95% cure rate in lesions less than 2 cm.
Punch/excision biopsy. Life long follow up.
Imiquimod 5% cream locally, if surgical
treatment not possible.
144 NAC OSCE |A Comprehensive Review

Jacob Simpson, 62 year old man presents to the Emergency Department with 12 hours suprapubic discomfort
and inability to urinate. Take a focused history & perform a focused examination.

Clinical Info: Mr Jacob Simpson presented with acute urinaiy retention for the past 12 hours. He is having
difficulty passing urine for the past 4 months, which has gradually increased. He has hesitancy, urgency,
increased frequency and weak stream. No hematuria or UTI. O/E there is a palpable supra-pubic mass.
Catheterization[yields 1200cc urine.
Clinical Case : Benign Prostatic Hyperplasia

HOPI Past History


OCD PQRST UV + AAA Do you have diabetes or hypertension?
How did it start? Sudden or gradual. Are you on any medications?
Is it getting worse/better or no changes in Are you allergic to any medications?
the symptoms? Previous renal colic /diagnosed prostate
Duration of inability to pass urine? hypertrophy?
Any pain associated? H/O prostate cancer, prostatism,
Location of pain? nephrolithiasis, UTIs? H/O pelvic radiation?
Type of pain - sharp or dull? Any h/o past surgeries? TURP?
Severity of pain on a scale of 1-10.
Do your symptoms change with time? Family and Social History
Difficulty initiating or maintaining urinary Any family history of cancer / similar
stream? complaints?
H/O weak/interrupted urinary stream. Currendy in a relationship?
Feeling of incomplete bladder emptying? Practicing safe sex?
Any nocturia/ urgency/ increased frequency? Do you think you are at risk of getting STIs?
Color of urine? Any blood in urine? Do you smoke? Duration & frequency.
H/O recurrent urinary infections? Do you consume alcohol? Duration oc
H/O renal pain or groin pain? frequency.
H/O fever, night sweats, weight loss, fatigue? Do you use recreational drugs? TRAPPED.
Any bowel complaints?
Any perineal numbness /leg weakness?

Differential Diagnosis Management


Benign rrostatic Hyperplasia. DRE (Digital rectal examination).
Urinary Tract Infection. Watchful waiting ( 50% resolve
Prostatitis. spontaneously).
Prostate Cancer. Lifestyle modifications - evening fluid
restrictions, planned voiding.
Investigations Urological consultation.
Urea / Creatinine. Medication - Alpha-adrenergic antagonists
Urinalysis, culture & sensitivity. (Terazosin, Doxazosin, Tamsulosin).
Prostate Specific Antigen (PSA). 5-alpha-reductase inhibitors ( Finasteride).
Renal and pelvic ultrasound. Surgery (TURP, open prostatectomy).
Cystoscopy. Minimally invasive (Stents, Microwave
therag^, Laser ablation, Cryotherapy, HIFU,
Clinical Cases - Surgery 145

Lydia Jones, a 30 years old office lady presented to your office with right hand numbness and weakness for 2
months. Take a focused history and address her concerns.

Clinical info: Ms Lydia Jones presented with gradual onset of right hand numbness and weakness for the
past 2 months. Her symptoms have worsened. Associated with paresthesias and pain in fingers at the end of
the day. She has difficulty opening jars, turning keys and night pains. She has no medical illness. Not on any
medications. She is an office administrator.
Clinical Case : Carpal Tunnel Syndrome

HOPI Past History


OCD PQRST UV + AAA Do you have diabetes or hypertension?
Handedness - left or right? Are you on any medications?
How did it start? Sudden or gradual. Are you allergic to any medications?
Is it getting worse/better or no changes in Any surgeries in the past?
the symptoms?
Duration of numbness? Family and Social Histoiy
Location of numbness? Do you smoke? Duration & frequency.
Any paresthesias/tingling/swelling? Do you consume alcohol? Duration &
Any weakness or muscle pain? frequency.
Is mere any associated pain? Any family history of thyroid disease?
Any difficulty opening jars/turning keys? Any family history of cancers/ medical
Any symptoms in night time? illnesses?
Any abnormal position of the hand?
Any neck pain/upper arm weakness?
Similar symptoms in other parts of the body?
Do your symptoms change with time?
Any changes in vision/speech/headache?
Any changes in gait?
Occupation?

Differential Diagnosis Management


Carpal Tunnel Syndrome. Modify manual work.
Cervical radiculopathy. Wrist splint (often worn at night).
TIA. NSAIDs, local corticosteroid injections.
Control underlying systemic contributors
Investigations (e.g. diabetes, hypothyroidism, arthritis).
CBC, blood glucose, electrolytes. Surgical decompression via flexor
C-spine X ray. retinaculum release.
Nerve conduction studies. Orthopedic or Plastic surgery consult.
146 NAC OSCE |A Comprehensive Review

Ruth Gagnon, a 60 years old woman presented to your office with right sided calf swelling for the past 2 days.
Take a focused history and perform focused examination.
Vitals: BP - 140/80 mm Hg, HR - 86/min, RR - 12/min,Temp - 37.5C

Clinical Info: Ms Ruth Gagnon is having right calf swelling and redness for the past 2 days. She has leg pain
also. She is a known hypertensive on medications. She recendy traveled for 20 hours in an overnight flight.
She is compliant with her medications. She has past history of breast cancer treatment 5 years ago with no
complications.
Clinical Case : Deep Vein Thrombosis

HOPI Past History


OCD PQRST UV + AAA Do you have diabetes or hypertension?
How did it start? Sudden or gradual. Are you on any medications?
Is it getting worse/better or no changes in Are you allergic to any medications?
the symptoms? Any surgeries in the past?
Duration of leg swelling? Past h/o recurrent infections?
Where is the leg swelling located?
Any leg pain associated with swelling? Family and Social History
Type of pain - sharp or dull pain? Do you smoke? Duration & frequency.
Does the pain radiate or shoot anywhere? Do you consume alcohol? Duration &
Severity of pain on a scale of 1-10. frequency.
Any pain during rest? Any family history of thyroid disease?
Any pain in the night time? Do you take any recreational drugs?
Any skin discoloration of legs/nail changes? Any family history of cancers/medical
Any skin ulceration of legs? illness?
Any fever/cough/cold?
Any chest pain/ shortness of breath?
Any headache/dizziness?
Any weakness/muscle pain?
Any recent travel?
Any prolonged immobilization?

Differential Diagnosis Management


Deep Vein Thrombosis. Urgent medicine consult.
Ruptured Baker's Cyst. Low molecular weight heparin x 3 days.
Cellulitis. Then switch to Warfarin.
Unilateral lymphedema. IVC filters (only if anticoagulation is
contradicted).
Investigations
CBC, ESR, Lipid profile, RFT.
D-dimer.
PT, PTT, INR, factor assay.
Deficiency of Antithrombin III, protein C or
S.
Lupus anticoagulant.
Doppler of lower limbs.
Impedance plethysmography.
Clinical Cases - Surgery 147

Mathew Hobbs, a 55 years old man presented with numbness in his both feet. He is a known diabetic. Take a
focused history and perform focused physical examination.
Vitals: BP - 130/90 mm Hg, HR - 86/min, RR - 14/min,Temp - 38.5C.

Clinical Info: Mr Mathew Hobbs is a known diabetic for the past 15 years. He is on oral hypoglycemics. His
last fasting glucose was 7.6 mmol/L. On examination, both feet were normal.
Clinical Case : Diabetic Foot

HOPI Past History


OCD PQRST UV + AAA Do you have diabetes or hypertension?
How did numbness start? Sudden or Are you on any medications?
gradual. Are you allergic to any medications?
Is it getting worse/better or no changes in Any surgeries in the past?
the symptoms? Past h/o recurrent infections?
Duration of numbness?
Location of numbness? Family and Social History
Is there any pain associated with numbness? Do you smoke? Duration and frequency.
Type of pain - sharp or dull pain? Do you consume alcohol? If yes, duration,
Does the pain radiate or shoot anywhere? amount and frequency.
Severity of pain on a scale of 1-10. Do you use recreational drugs? TRAPPED
Do your symptoms change with time? Any family history of cancers/medical
H/o tingling? illnesses?
Any recent injury to feet? Exercise schedule.
Any blisters/calluses on the feet? Dietary restrictions for diabetes.
Any swelling of feet? Compliance with medications?
Any changes in vision?
H/o dizziness / LOC?
H/o chest pain/orthopnea?
H/o excessive sweating?
Any changes in appetite?
Any changes in weight?
Any changes in bowel Sc urinary habits?
Last fasting blood sugar, eye ana foot exam?

Diagnosis Management
Diabetic foot Self foot exam daily.
Foot examined regularly at physician visits.
Investigations Perform Peripheral Neuropathy Testing.
Fasting blood glucose. Check for pedal pulses.
HbAlC. Evaluate Sc aggressively treat new foot
Fasting lipids, Renal function tests. wound.
ECG. Avoid foot trauma
Fundoscopy. Do not walk barefoot.
Urinalysis with urine dip. Cut nails carefully.
Avoid excessive heat or chemicals.
148 NAC OSCE |A Comprehensive Review

Wayne Singer, 68 year old man presented with difficulty swallowing for the past 4 months. Take a focused
history and perform a focused examination.

Clinical Info: Mr Wayne Singer has difficulty swallowing for the past 4 months. It has gradually increased
from solids to liquids. He feels a lump in the throat. He has chest pain when he eats food. He has noticed
weight loss, night sweats and decreased appetite in the last 3 months. He is a chronic smoker for the past 30
years.
Clinical Case : Difficulty Swallowing ( Ca Oesophagus)

HOPI Past History


OCD PQRST UV + AAA Do you have diabetes or hypertension?
How did it start? Sudden or gradual. Are you on any medications?
Is it getting worse/better or no changes in Are you allergic to any medications?
the symptoms? Any surgeries in the past?
Duration of difficulty swallowing? Any history of goitre or thyroid problems?
Is there difficulty transferring food from Any radiation exposure?
mouth to esophagus (suggestive of
oropharyngeal dysphagia) or further down Family and Social History
(suggestive of esophageal dysphagia)? Do you smoke? Duration &, frequency.
Is me problem worse with solids (suggests Do you consume alcohol? Duration &
mechanical obstruction) or liquids (suggests frequency.
neuromuscular dysfunction, often can't Any family history of thyroid disease?
swallow both solids and liquids)? Any family history of cancers/ medical
Is there a sensation of lump in the throat illnesses?
(globus hystericus)?
Progression of difficulty swallowing solids to
difficulty in swallowing liquid? (Suggests a
worsening stricture of growing tumor)
Any swelling the neck?
Aggravating, relieving factors? The
association of intermittent obstruction and
chest pain suggests esophageal spasm.
H/O peptic ulcer, reflux, hiatus hernia?
H/O fever, night sweats, weight loss, fatigue,
hematemesis, black stools?

Differential Diagnosis Chest X-ray.


Esophageal Cancer. LFTs, RFT.
Stricture due to GERD/Trauma. Abdominal ultrasound.
Neuromuscular obstruction - achalasia, CBC.
cranial nerve palsy, MS, supranuclear palsy,
stroke, motor neuron disease, myasthenia Management
gravis, muscular dystrophy. Urgent surgical consult.

Investigations
Endoscopy with biopsy.
Upper Gl series.
CT chest (for mediastinal and lymph node
involvement).
Clinical Cases - Surgery________________________________________________________ 149

Brad Chisolm, a 35 years old man presented with bloody vomiting to the ER for the past 2 hours. Take a
focused history and perform focused physical examination.
Vitals: BP - 90/60 mm Hg, HR - 116/min, RR - 12/min,Temp - 37.0C

Clinical Info: Mr Brad Chisolm presented with acute onset of blood in vomitus, 2 episodes in 2 hours ago.
He has no history of trauma. Non alcoholic, non smoker. He has been having chronic knee pain after a
skateboarding accident 2 weeks ago. He is taking Ibuprofen for the past 2 weeks 4-5 times a day. Has
moderate epigastric pain. No hemoptysis, hematuria or hematochezia. No surgeries/ medical illnesses.
Clinical Case : Hematemesis

HOPI Past History


OCD PQRST UV + AAA Do you have medical illnesses?
How did it start? Sudden or gradual. Any surgeries in the past?
Is it getting worse/better or no changes in
the symptoms? Family and Social History
Duration of blood in vomitus? Do you smoke? Duration & frequency.
Amount of blood? Do you consume alcohol? Duration &
Color of blood? frequency.
Number of episodes? Any family history of cancers/ medical
Is there any abdominal pain? Do you take any recreational drugs?
Type of pain - sharp or dull pain? Any family history of cancers/ medical
Does the pain radiate or shoot anywhere? illnesses?
Severity of pain on a scale of 1-10.
Do your symptoms change with time?
Any blood wnile coughing?
Any blood in stools/last bowel movement?
Any blood in urine?
Any trauma to abdomen?
Any fever recently?
Any dizziness/fainting?
Currently on any mdications?
Last meal?
Any allergic reactions?

Differential Diagnosis Management


Gastric ulcer. ABC.
Acute Esophagitis. Admit.
Acute gastritis. NPO.
Drug induced coagulopathy. NG tube.
IVF via large bore cannulas.
Investigations Inj Ranitidine 50 me IV bolus and q8h.
CBC, electrolytes, glucose. In case of perforated ulcer - surgery consult.
LFT, RFT.
PT, PTT, INR.
Blood group & cross match.
Urgent endoscopy.
Upright abdominal X Ray.
150 NAC OSCE |A Comprehensive Review

Mary Laplante, a 40 years old lady presented with swelling in the neck for the past 1 month. Take a focused
history and perform focused physical examination.
Vitals: BP - 120/88 mm Hg, HR - 96/min, RR - 12/min,Temp - 37.5C.

Clinical info: Ms Mary Laplante noticed this swelling in the anterior neck with no other prominent
symptoms. On examination, there is 2cm x 2cm mobile, non tender thyroid enlargement in the left lobe.
Clinical Case : Neck Swelling

HOPI Past History


OCD PQRST UV + AAA Do you have diabetes or hypertension?
How did it start? Sudden or gradual. Are you on any medications?
Is it getting worse/better or no changes in Are you allergic to any medications?
the symptoms? Any surgeries in the past?
Since how long have you noticed the lump? Any history of goitre or thyroid problems?
Where is the lump located? Any radiation exposure?
How does the lump feel like?
Is there any pain associated with the lump? Family and Social History
Type of pain - sharp or dull pain? Do you smoke? Duration and frequency.
Does the pain radiate or shoot anywhere? Do you consume alcohol? If yes, duration,
Severity of pain on a scale of 1-10. amount and frequency.
Do your symptoms change with time? Any family history oi thyroid disease?
H/o fever/cough/sore throat? Any family history of cancers/medical
Any weakness or myalgia? illnesses?
Any change in voice?
Any change in vision?
Any change in appetite?
Any change in weight?
Any changes in bowel & urinary habits?
Any temperature intolerance?
Any palpitations or tremors?
Any swelling of face or feet?
Last menstrual period?
Any changes in menstrual cycles?

Differential Diagnosis Investigations


Toxic nodular goitre. TSH.
Hashimotos thyroiditis. Free T3 and T4.
Thyroid cyst. Thyroid ultrasound.
Thyroid adenoma. Thyroid biopsy.
Thyroid lymphoma. Antimicrosomal & anti-thyroglobulin abs.

Management
Refer to an endocrinologist.
Clinical Cases - Surgery________________________________________________________ 151

Judy Frances, a 25 year old female presented to your office with lower abdominal pain for the past 1 day. Take
a focused history and perform a focused examination.

Clinical Info: Ms Judy Frances had a gradual onset of right lower abdominal pain 24 hours ago. The pain has
gradually increased in intensity, grade 7/10. She has fever, nausea and vomiting since morning. No bowel or
urinary complaints. No trauma. Her LMP was one week ago. O/E peritoneal signs are present and tenderness
at Mcfeurney's point.
Clinical Case : rain Abdomen /Acute Abdomen

HOPI Past History


OCD PQRST UV + AAA Do you have diabetes or hypertension?
How did it start? Sudden or gradual. Are you on any medications?
Is it getting worse/better or no changes in Are you allergic to any medications?
the symptoms? Any surgeries in the past?
Duration of abdominal pain?
Where is the pain located? Family and Social Histoiy
Type of pain - sharp or dull? Do you smoke? Duration & frequency.
Does the pain radiate or shoot anywhere? Do you consume alcohol? Duration &
Severity of pain on a scale of 1-10. frequency.
Do your symptoms change with time? Do you use recreational drugs? TRAPPED.
Any aggravating or relieving factors (change Any family history of cancers/ medical
in position/food intake)? illnesses?
Effect on activities of daily living, functional
limitation?
H/o fever, nausea/vomiting, dizziness /
fainting?
Any urinary complaints?
Loose stools /black stools or blood in stools?
Change in stool caliber?
Females :
LMP?
Pain associated with periods?
Vaginal discharge?

Differential Diagnosis Management


Appendicitis. Admit.
surg*cal consultation.
Renal colic due to nephrolithiasis.
PID. IV Antibiotics.
Ectopic pregnancy. NPO, NG tube.
Ruptured Ovarian cyst or ovarian torsion.
Mittelschmerz syndrome.

Investigations
Abdominal X-ray 3 views
Abdominal 8c pelvic ultrasound
CBC, Electrolytes, Urea, Creatinine
INR/PTT, Glucose, beta HCG
Urinalysis
Stool for occult blood
Cervical swabs for culture/ PAP smear
152 NAC OSCE |A Comprehensive Review

Ronald Mandel, a 65 years old man presented to your office with bilateral leg pain for the past 2 weeks. Take
a focused history and perform focused examination.
Vitals: BP - 140/90 mm Hg, HR - 86/min, RR - 12/min,Temp - 37.5C

Clinical Info: Mr Ronald Mandel is a known hypertensive for 15 years. He was diagnosed with CAD 5 years
ago. He is having bilateral lower leg pain for the past 2 weeks. Pain starts only when he has walked for 10-15
minutes. Pain subsides after taking rest. He has paresthesias too. No weakness or night pain. He is a chronic
smoker and alcoholic for 20 years.
Clinical Case : Peripheral Vascular Disease (Examination on page - 67)

HOPI Past History


OCD PQRST UV + AAA Do you have diabetes or hypertension?
How did it start? Sudden or gradual. Are you on any medications?
Is it getting worse/better or no changes in Are you allergic to any medications?
the symptoms? Any surgeries in the past?
Duration of leg pain? Past h/o recurrent infections?
Where is the leg pain located?
Type of pain - sharp or dull pain? Family and Social History
Does the pain radiate or shoot anywhere? Do you smoke? Duration & frequency.
Severity of pain on a scale of 1-10. Do you consume alcohol? Duration &
Distance or time walked before pain starts. frequency.
Speed of walking before pain starts. Any family history of thyroid disease?
Degree of incline walked. Do you take any recreational drugs?
Any pain during rest? Any family history of cancers/medical
Any pain in the night time? illnesses?
Any skin discoloration of legs/nail changes?
Any skin ulceration of legs?
Any chest pain/ shortness of breath?
Any headache/dizziness?
Any weakness/muscle pain?
Do the symptoms come back with exertion?
H/o impotence?

Differential Diagnosis Management


Peripheral Vascular Disease. Symptomatic treatment.
Osteoarthritis. Tab Aspirin 75-150 mg OD.
Neurogenic claudication. Clopidogrel.
Baker's cyst. Vascular surgery opinion.
Reduction of CAE) risk factors.
Investigations Alcohol & smoking cessation.
CBC, Lipid profile.
Serum Homocysteine, Apolipoprotein A.
Serum Creatinine.
Hemoglobin AlC ,fasting glucose.
Urinalysis.
Ankle brachial index, Doppler of lower
limbs.
CT Angiography.
Clinical Cases - Surgery 153

Alex Pereira, a 45 years old man presented with high grade fever with chills on day 3 after his abdominal
surgery. Take a focused history and perform focused physical examination.
Vitals: BP - 110/80 mm Hg, HR - 96/min, RR - 12/min,Temp - 38.5C.
Clinical Info: Mr Alex Pereira was operated 3 days ago for acute appendicitis. The morning shift nurse
noticed temperature of 39.8 C. He is complaining of chills , rigors and nausea. He has pain at the wound site.
No burning in urine. Had one bowel movement in the morning. No chest pain or shortness of breath. IV
antibiotics were stopped on post op day 2. No other complications. O/E: Wound site is tender, erythematous
with yellowish discharge.
Clinical Case : Post Operative fever secondary to wound infection

HOPI Past History


OCD PQRST UV + AAA Do you have diabetes or hypertension?
How did it start? Sudden or gradual. Are you allergic to any medications?
Associated with any chills/rigors? Any surgeries in the past?
Duration of fever? Past h/o recurrent infections?
Fever pattern- continuous,intermittent,
remittent? Family and Social History
H/o cough/sore throat/rash? Do you smoke? Duration & frequency.
Any chest pain/shortness of breath? Do you consume alcohol? Duration &
Any abdominal pain? frequency.
Any burning in urine/cloudy urine? Do you take any recreational drugs?
Any dysuria/hematuria? Any family history of cancers/ medical
Last bowel movement? illnesses?
Any pain at the wound site?
When was the wound dressing changed?
Any pain at the IV site?
Any pain in calves?
Any leg swelling?
Current list of medications?
Any changes in medications?
Last meal?
Reason for surgery?
Any pre-op/intra-op surgical complications?

Differential Diagnosis Management


Post op wound infection. Start IV Antibiotics & Antipyretics.
Urinary tract infection. Wound drainage.
Intra-abdominal abscess. Wound dressing for healing by
Septic thrombophlebitis. secondary intention.
Vitals q4n till fever subsides.
Investigations
CBC, electrolytes, glucose.
LFT, RFT.
Urine routine and culture/sensitivity.
Wound swab, culture/sensitivity, gram stain.
Blood culture/sensitivity.
Ultrasound abdomen.
154 NAC OSCE |A Comprehensive Review

Diane Richardson, a 55 years old woman presented with incidental finding of lung nodule on chest x ray. Take
a focused history and perform focused physical examination.
Vitals: BP - 120/80 mm Hg, HR - 86/min, RR - 18/min,Temp - 37.5C.

Clinical Info: Ms Diane Richardson has chronic cough for 2 months. It was gradual in onset. No fever or
recurrent pneumonia. A routine chest X ray revealed solitary nodule in the right middle lobe. She is a known
smoker for the past 30 years. She is a chronic alcoholic also.
Clinical Case : Solitary lung nodule

HOPI Past History


Why was the chest X ray done? Do you have diabetes or hypertension?
OCD PQRST UV + AAA Are you on any medications?
How did cough start? Sudden or gradual. Compliance with medications?
Is it getting worse/better or no changes in Are you allergic to any medications?
the symptoms? Any surgeries in the past?
Duration of cough? Past h/o recurrent infections?
Type of cough - dry/expectorant?
Amount /color of cough? Family and Social History
Any of shortness of breath? Do you smoke? Duration & frequency.
Present at rest or with exertion? Do you consume alcohol? Duration &
H/o orthopnea? frequency.
H/o paroxysmal nocturnal dyspnea? Any family history of cancers/ medical
Any chest pain/palpitations? illnesses?
Does the pain radiate or shoot anywhere? Occupation?
Severity of pain on a scale of 1-10. Exposure to carcinogenic substances?
Do your symptoms change with time?
H/o fever/chills/night sweats?
Any change in weight/appetite?
Any recent travel?
Any contact with sick person?
Any pets at home?

Differential Diagnosis Management


Lung Carcinoma. Symptomatic treatment.
Solitary granuloma. Comparison with older chest X rays.
Pulmonary tuberculosis. Urgent respirology consult.
TB skin test.
Investigations CT guided biopsy.
CBC, electrolytes, glucose. Bronchoscopy.
LFT, RFT.
Chest X Ray.
CT Chest.
Sputum cytology.
Clinical Cases - Surgery 155

Elaine Jones, a 60 years old lady presented with swelling in the neck for the past 4 months. Take a focused
history and perform focused physical examination.
Vitals: BP - 120/80 mm Hg, HR - 88/min, RR - 12/min,Temp - 37.5C.

Clinical info: Ms Elaine Jones presented with a solitary swelling in the right lobe of the thyroid for the past 4
months. The swelling has increased in size. She has no fever, cough or sore throat. She has decreased appetite
and 5 kg weight loss in the past 3 months. She has hoarseness of voice.
Clinical Case : Thyroid mass

HOPI Past History


OCD PQRST UV + AAA Do you have diabetes or hypertension?
How did it start? Sudden or gradual. Are you on any medications?
Is it getting worse/better or no changes in Are you allergic to any medications?
the symptoms? Any surgeries in the past?
Since how long have you noticed the Any history of goitre or thyroid problems?
swelling? Any radiation exposure?
Where is the swelling located?
How does the swelling feel like? Family and Social History
Is there any pain associated with the Do you smoke? Duration & frequency.
swelling? Do you consume alcohol? Duration &
Type of pain - sharp or dull pain? frequency.
Does the pain radiate or shoot anywhere? Any family history of thyroid disease?
Severity of pain on a scale of 1-10. Any family history of cancers/ medical
Do your symptoms change with time? illnesses?
H/o fever/cough/sore throat?
Any weakness or myalgia?
Any change in voice?
Any change in vision?
Any change in appetite?
Any change in weight?
Any changes in bowel & urinary habits?
Any temperature intolerance?
Any palpitations or tremors?
Any swelling of face or feet?

Differential Diagnosis Management


Thyroid adenoma. Urgent surgical opinion.
Multinodular goitre. Radioiodine in case of hyperthyroidism
Thyroid cyst. Chemo/radiotherapy in case or anaplastic
Parathyroid adenoma. tumor.
Benign nodule. Surgical removal in cases other than
anaplastic tumor or lymphoma.
Investigations
TSH.
Free T3 and T4.
Radioactive thyroid uptake scan-
Hot nodule : Hypo/Hyperthyroidism.
Cold nodule: Thyroid malignancy.
Thyroid ultrasound.
Thyroid biopsy and cytology.
Neck CT.
156 NAC OSCE |A Comprehensive Review

Joseph Quinton, a 25 years old male was brought to the ER after a motor vehicle accident with the following
vitals: BP - 80/50 mm Hg, HR - 116/min, RK - 10/min,Temp - 37.0C, 0 2 sat - 80%.
Manage the patient with a nurse.

Clinical Info : Mr Joseph Quinton had a MVA 1 hour ago. He is conscious, alert and responding to verbal
commands. He is in excruciating pain & complains of difficulty breathing. He can move all limbs. On
auscultation, there are decreased breath sounds on right side oi chest with dullness on percussion.
Diagnosis: Trauma - Right sided hemothorax.

Introduce yourself.
Call out the patient's name and assess verbal response.
Follow universal precautions - mask,wash hands,wear gloves.
Ask for patient's vitals.
Ask the patient to be connected to monitors: cardiac monitor, BP cuff, pulse oximeter, temperature
probe.
rlace cervical collar with in-line traction.
AIRWAY - Open mouth & check airway for any loose body/dentures/bleeding. Mention any
specific smell.
BREATHING -
- LOOK - cyanosis/pallor/icterus/nasal flaring/chest movements/respiratory rate/neck venous
engorgement.
- FEEL - flow of air/tracheal shift/chest wall for crepitus/flail segments/sucking chest
wounds/subcutaneous emphysema.
- LISTEN - sounds of obstruction/breath sounds/symmetry of air entry/air escaping/noisy
breathing.
CIRCULATION - feel for peripheral pulses/ assess for shock-capillary refill,cool extremeities.
DISABILITY - GCS/pupillary reaction.
Order primary INVESTIGATIONS - CBC, differentials, electrolytes, RFTs, LFTs, ABG, INR,
PTT, 12 lead ECG, urinalysis, urine toxicology screen, portable chest X ray, C-spine X ray, Blood
group, type & cross match, blood glucose.
Place large bore IV cannulas both arms & IVF 11 normal saline bolus stat.
Attach to 100 % oxygen through mask/nasal cannulas.
Ask for vitals again.
Ask for orientation to time/place/person, mechanism of injury/ any eye witnesses/ any loss of
consciousness/ vomiting/ pain anywhere in the body/ last meal/ any arug allergies/ TAMPLE or
SAMPLE.
EXPOSURE/ SECONDARY SURVEY - Assess for:
- Skull/cranium fractures.
- Injuries to the face.
- Hemotympanum/ otorrhea/ rhinorrhea/ epistaxis/ batde's sign/ racoon eyes.
- Check upper extremities for fractures/ bruises/ lacerations/ tattoos/ needle track marks/ medic
alert bracelet/ scars/ wounds.
- Check abdomen for movements/ scars/ wounds/ bruises/ rigidity/ masses, bowel sounds.
- Check lower extremities for fractures/ bruises/ wounds/ tattoos/needle track marks.
- Pelvic compression to rule out pelvic fracture.
- Deep tendon reflexes of upper & lower extremities.
- Sensory examination of upper & lower extremities.
- Motor examination of upper & lower extremities.
- Genital examination.
- Spinal examination - log roll with help to look for fracture/ step deformity.
- Digital rectal examination.
- Change rigid board to semi rigid board.
Clinical Cases - Surgery 157

Give SECONDARY ORDERS:


- Ask for vitals again.
- Results of the investigation ordered earlier.
- Request for needle thoracostomy with 16 G needle in 2nd intercostal space.
- Ask what do you see - air or blood, (nurse or physician examiner will respond)
- Urgent cardio-thoracic consult for chest tube.
- Order 2L blood transfusion.
- Foley's catheter.
- Naso-gastric tube.
- Measure intake/output.

Management for specific trauma case scenarios

Management for tension pneumothorax


Large bore IV needle in 2nd intercostal space in the mid-clavicular line.
Cardio-thoracic consult.
Chest tube to be inserted in the 5th intercostal space in the anterior axillary line.

Management of open pneumothorax


3 sided sealea dressing.
Cardio-thoracic consult.
Chest tube to be inserted in the 5th intercostal space in the anterior axillary line.
Management of flail chest
Nasal oxygen.
IV fluids.
Pain control with Inj Morphine 2-4 mg IV.
Positive pressure ventilation.
Cardio-thoracic consult.

Management of pericardial tamponade


Nasal oxygen.
IV fluids.
Cardio-thoracic consult.
Pericardiocentesis.

Management of increased intracranial pressure


Raise head end of the bed to 30-45 degrees.
Maintain neck in a neutrahsosition.
Hyperventilate to target pC02 30-35 mmHg.
Osmolar diuresis - Inj Mannitol 20% IV 1-1.5 g/kg, then 0.25 g/kg q6h to achieve serum osmolarity
of 315-320.
Sedation.
Paralysis with vecuronium.
Oxygen to maintain p02 >60 mmHg.
Urgent neurosurgery consult.
Order CT scan of tne head.
158 NAC OSCE |A Comprehensive Review

Management of open fracture


Remove debris and wound irrigation with normal saline.
Sterile dressing and splint the fracture.
Check the neurovascular status of the limb.
Order X ray of the fracture with 1 joint above and 1 joint below.
Start IV analgesics.
IV antibiotics.
Tetanus shot, if not immunized in the last 5 years.
NPO.
Urgent orthopedic consult.

Management of anterior shoulder dislocation


Posture of the arm - Abducted & externally rotated.
Order X rays: AP, trans-scapular, axillary views.
Urgent orthopedic consult.
Closed reduction with IV sedation & muscle relaxation.
Obtain post reduction x rays.
Check post-reduction neurovascular status.
Sling for 3 weeks, followed by shoulder rehabilitation.

Management of ankle sprain


Follow Ottawa ankle rules to order X rays.
Ankle X rays - AP, lateral, mortise views.
Rest to the joint.
Ice to be used for 5-20 minutes every 2 hours.
Compression with a tensor bandage.
Elevate the limb.
Analgesics.
Urgent orthopedic consult.
Reduce weight bearing with help of crutches.

Glasgow Coma Scale

t 2 3 4 5 6
Eyes Does not open Opens eyes in Opens eyes in Opens eyes N/A N/A
eyes response to response to spontaneously
painful stimuli voice

Verbal Makes no Incomprehensible Utters Confused, Oriented, N/A


sounds sounds inappropriate disoriented converses
words normally

Motor Makes no Extension to Abnormal Flexion / Localizes Obeys


movements painful stimuli flexion to Withdrawal to painful stimuli commands
(decerebrate painful stimuli painful stimuli
response) (decorticate
response)

Generally, brain Injury is classified as Severe : GCS < 8, Moderate : GCS 9-12 and Minor : GCS > 13.
Clinical Cases - Counseling 159

Allison George, a 28 years old primigravida came to your clinic for her antenatal visit. She wants info for
breast feeding.
Take a focused history and address her concerns.

Counseling Case : Breast Feeding

HOPI Past History


Current gestational age? Do you have any medical illnesses?
Any complications in the current pregnancy? Are you on any medications?
GTPAL : Gravidity, Term pregnancies, Are you allergic to any medications?
Prematurity, Abortion, Living children. Any surgeries in the past?
Last fetal ultrasound. Any h/o cancer in the past?
Any maternal screening till date for genetic
disease? Family and Social History
Any genetic disorder in family ? Do you smoke? Amount/frequency.
Any Breast feeding issues in previous Do you consume alcohol?
pregnancies? Amount/frequency.
Any recurrent infections of the breast? Do you use recreational drugs? TRAPPED.
Any h/o HIV/HCV/HBsAg/active HSV? Any family history of cancers (esp breast
cancer)?

Counseling for Breast Feeding


Breast feeding has to be initiated immediately after birth.
Initial clear breast milk called COLOSTRUM is full of nutrients and immunoglobulins.
It is beneficial for developing immunity in the newborn.
Full milk production starts by 3-7 days.
Exclusive breast feeding is recommended during the first 4 months.
Breast milk is easily digested with minimal renal load.
Breast milk has low allergic potential than cow's milk protein.
Lower pH promotes growth of lactobacillus in the Gl tract.
Creates parent - child bonding.
Breast fed babies require following supplements: Vitamin K, Vitamin D, Iron (from 4months to 12
months), Fluoride(after 6 months).
Contraindications to breast feeding-
- Mother receiving chemotherapy.
- Mother with HlV/AIDS, active TB, herpes in the breast region.
- Mother consuming alcohol/illicit drugs.
- Mother on drugs contraindicated for breast feeding like antimetabolites, bromocriptine,
chloramphenicol, metronidazole, tetracycline, lithium, cyclophosphamide.
Complications of breast feeding - sore/cracked nipples, breast engorgement, mastitis, breast feeding
jaundice, breast milk jaundice, oral thrush in baby.
Breast feeding helps in losing pregnancy weight.
Lactational amenorrhea protects against future pregnancy.
Give educational info for breast feeding.
160 NAC OSCE |A Comprehensive Review

Rachel Marshall is a 20 months old girl brought to the ER with excessive crying. She has signs of fracture of
right humerus. You also observe some old healed bruises elsewhere on her body. She is now stable. Take
history from the mother and address her concerns.

Counseling Case : Child abuse

HOPI Are there other children in the house?


How did the injury occur? Have they had broken bones or other
When did the injury occur? injuries?
Location of injury? Was this child a planned pregnancy?
What was the child doing at the moment of Problems with pregnancy, birth history?
injury? detailed history.
Any loss of consciousness? Developmental milestones. Detailed history.
Any abnormal position or posture of the What is the typical response of caregivers
body? when the child cries or misbehaves?
Any seizures after the trauma?
Any vomiting? Family and Social History
Any excessive crying? Do you smoke? Amount/frequency.
Any bleeding or discharge from Do you consume alcohol?
nose/ears/mouth? Amount/frequency.
Who are the child's care givers? Do you use recreational drues? TRAPPED.
Who lives in the house or comes in contact Alcoholism/smoking/drug abuse by other
with the child? caregivers?
How did the child get the bruises? Economic condition of the family?
What happened with the other fractures? Any problems with the law?
Any other injuries in the past? Were the caregivers abused as children?
Is the child accident prone or difficult to Is there spousal abuse, sexual abuse or incest?
handle? Has the Children's Aid Society been
What is the child's personality: open vs. involved with this child or other children?
withdrawn?

Counseling for child abuse


Do a complete physical examination of the baby.
Document and/or photograph all injuries: type, location, size, shape, color, pattern.
Inform parents or care takers about the suspicion of child abuse.
Order blood tests to rule out medical causes of presenting symptoms.
Sexually transmitted infection work up.
Skeletal survey/bone scan.
CT/MRI.
Fundoscopy.
Report all suspicious cases to the CHILDREN'S AID SOCIETY.
Admit for serious injuries.
Involve social worker and other community resources.
Inform that you are legally obliged to inform the Children's Aid Society.
Your duty to report overrides patient's confidentiality.
Evaluate the risk factors for chiild* abuse:
- Environmental factors- Social isolation, poverty, domestic violence.
- Caregiver factors- Parents were abused as children, psychiatric illnesses, substance abuse,
single parent family, poor social & vocational skills, below average intelligence.
- Child factors- difficult temperament, disability, special needs(eg developmental delay),
premature.
Clinical Cases - Counseling 161

Nancy Alfredo, a 30 years old woman presented to your clinic with a black eye and multiple bruises on her
arms. Take history and address her concerns.

Counseling Case : Domestic violence

HOPI Who are the biological parents of the


How did the injury occur? children?
When did the injury occur? Do the children witness physical abuse?
Location of injury/injuries? Ask about violence to the children, sexual
Circumstances in which the injury occurred? abuse?
Describe violent episode, what triggered it? Is the boyfriend willing to seek help?
Were objects used as weapons? Any stressors at home/work?
Was the boyfriend remorseful afterward?
History of previous episodes of violence or Family and Social History
loss of temper by boyfriend? Do you smoke? Amount/frequency.
What was the patient's response? Do you consume alcohol?
Has patient been in an abusive relationship Amount/frequency.
before? Do you use recreational drugs? TRAPPED.
Is the boyfriend controlling? Does the partner abuse alcohol or other
Does he restrict her activities? drugs?
Question her excessively after she has been Economic situation?
out? Any family history of physical abuse?
Engages in verbal abuse or threats?
Is the violence increasing in severity?
Are there children in the house?

Counseling for domestic violence


Explain that the boyfriend hitting the patient is a criminal assault and an example of domestic
violence.
Domestic violence tends to increase over time unless the victim leaves, or the abuser and couple seek
therapy.
Very often, women don't leave their abusive partner until they are seriously hurt.
Domestic violence between adult partners tends to be reflected in future behavior of children who
are exposed to it and there is a risk of violence to the children.
Child abuse is a criminal act and if suspected, is reportable to police by law.
Spousal abuse is also a criminal act but is not reportable by law.
Recommend that the patient not return to the abuser if there is risk to her safety (e.g. not the first
assault, abuser not remorseful).
If the patient does return, an exit plan should be developed to ensure patient safety.
Document all evidence of abuse (pictures, sketches) ana related visits; quote patient direcdy in chart.
Alternatively, the patient can contact the police to obtain a restraining order on the abuser.
Develop a plan with the patient to seek alternate living arrangements (women's shelter).
Enlist die help of patient's support structure (friends, other family members).
Contact the police (patient should be informed that, if contacted, the police will lay charges whether
the patient wants to or not).
Counsel patient on how to enter into a controlled, safe environment & contact with the abuser to
discuss possible therapy for anger management and controlling behaviors.
Social worker referral and provide info about community resources.
Arrange follow up.
162 NAC OSCE |A Comprehensive Review

Sara Chang, a 55 years old lady came to your clinic to get info about Hormone Replacement Therapy. She is
menopausal for the past 2 years. She is having significant hot flushes, mood fluctuations and vaginal dryness.
It is significantly affecting her quality of life.
Take a focused nistory and address her concerns.

Counseling Case : Hormone replacement therapy

HOPI Any h/o fibroids?


Menopausal since when? Any headaches/migraines?
Any post menopausal bleeding/spotting? Any liver/gallbladder disease?
Any hot flushes? Any blood clotting disorders?
Any vaginal discharge/itchiness/dryness?
Any dysparuenia? Past History
Any night sweats/sleep disturbances? Do you have medical illnesses?
Any chest pain/palpitations? Are you on any medications?
Any h/o breast lump/mass? Are you allergic to any medications?
Any nipple discharge? Any surgeries in the past?
Any weight loss/gain? Any h/o cancer in the past?
Any bony pains?
Any bowel/urinary changes? Family and Social History
Any mood changes/irritability/decreased Do you smoke? Amount/frequency.
libido? Do you consume alcohol?
GTPAL - Gravidity, Term pregnancies, Amount/frequency.
Prematurity, Abortions, Living children. Do you use recreational drugs? TRAPPED.
When was last PAP test done? Any family history of cancers (esp
When was the last breast exam and breast/encfometrial cancer)?
mammogram done?

Counseling for hormone replacement therapy


HRT is recommended for significant vasomotor symptoms of menopause.
Low dose of hormones is given for a short duration of < 5 years.
Types of HRT - Oral, Transdermal (patch, gel), Combined estrogen & progestin.
Give information brochures about HRT.
Compliance to medication dose is very important.
Encourage annual physical examination with a family physician.
Side effects:
Abnormal uterine bleeding, mastodynia.
edema, bloating, heartburn, nausea, mood changes.
Contraindications to HRT:
Pre-existing uncontrolled hypertension.
Uterine fibroids, endometriosis, migraine.
Family h/o estrogen dependent cancers.
Chronic thrombophlebitis, diabetes mellitus.
Impaired liver function/ gall bladder disease.
Hypertriglyceridemia, fibrocystic disease of the breasts.
HRT is protective against osteoporotic fractures and colon cancers.
HRT increases risk for:
Invasive breast cancer.
Coronary heart disease.
Deep vein thrombosis or pulmonary embolism.
Stroke.
Dementia and mild cognitive impairment.
Clinical Cases - Counseling 163

Nadia Solanski, a 45 years old lady came to your clinic to get info about mammogram.
Take a focused history and address her concerns.

Counseling Case : Mammogram

HOPI Past History


Any h/o breast lump/mass? Do you have medical illnesses?
Any breast discharge? Are you on any medications?
Any nipple discharge? Are you allergic to any medications?
Any trauma to the Breast? Any surgeries in the past?
Any abnormal skin discoloration of breast? Any h/o cancer in the past?
Any breast surgeries/biopsies?
Any fever/night sweats/weight loss? Family and Social History
Any bony pains? Do you smoke? Amount/frequency.
Any bowel/urinary changes? Do you consume alcohol?
When was your first menstrual period? Amount/frequency.
When was the last menstrual period? Do you use recreational drugs? TRAPPED.
Any irregularity in the menses? Any family history of cancers (esp breast
Any dysmenorrhea/ menorrhagia/ cancer)?
oligomenorrhea?
Any passage of clots?
GTPAL - Gravidity, Term pregnancies,
Prematurity, Abortions, Living children.
Age at which 1st child delivered?
H/o breast feeding?
When was the last PAP test done?
When was the last breast exam done?

Counseling for Mammogram

Mammogram is an annual screening test for the early detection of breast cancer.
Breast cancer is the second leading cause of cancer mortality in women.
Every 1 in 9 women in Canada are diagnosed with breast cancer.
It is recommended after the age of 40 years or more for every women.
It is done annually or every 2 years as a routine preventive test.
If there is a strong family nistory of breast cancer or genetic pre-disposition to breast cancer, then
mammogram is done 5-10 years prior to the age of the relative detected with cancer.
There are two types of mammogram - Screening and Diagnostic.
Diagnostic mammogram is done in cases of breast mass/lumps/suspicion of breast cancer.
Mammogram is a special X ray of the breast done in a diagnostic radiology clinic.
The procedure mignt cause slight discomfort or pain which lasts only few seconds.
It can take upto 20 minutes to do a mammogram.
Images are interpreted by an experienced radiologists.
In case of any abnormality, further tests will be arranged.
Ideally mammogram should be done after your period has stopped, to avoid discomfort.
Give information brochures for mammogram.
Encourage annual physical examination with a family physician.
164 NAC OSCE |A Comprehensive Review

Jenna Martin is a 28 year old mother who has concerns about the immunization for her 2 months old son.
Take history and address her concerns.

Counseling Case : Immunization

HOPI
Take detailed prenatal/antenatal history?
Any complications during pregnancy?
Any complications during delivery time?

Any h/o egg allergies?


Any allergies to medications?
Feeding pattern?
Developmental history?
Any fever/vomiting/irritability?
Any bowel complaints?
Any urinary complaints?
Any neonatal jaundice?
Hearing & vision tests for the newborn?
Any issues during newborn examination?
Any h/o complications after immunization in the family?

Counseling for immunization


Ask about the patients concerns?
What does she want to know and why?
Immunization (vaccination) is a way of creating immunity to certain diseases by using small amounts
of a killed or weakened microorganism that causes the particular disease.
Explain that vaccines protect children from diphtheria, tetanus, pertussis, polio (DPTP), mumps,
measles, rubella (MMR), influenza (Hib) and nepatitis B (Hep B).
All of which were once common and caused serious, sometimes fatal illness in Canada, and now
these diseases are hardly ever seen because of vaccines.
Explain that the vaccines stimulate the immune system.
Infants are fully capable of generating protective numoral Sc cellular immune responses to multiple
vaccines simultaneously.
The timing of vaccines is important; some vaccines are most effective when given to children at a
particular age or in combination with other vaccines.
Every batch of vaccine is tested for safety and quality in Canada before it is released for public use.
Some children have a temporary sore arm (with induration and tenderness) at the injection site,
malaise, mild fever or rash.
Allergic reactions to vaccines also occur including urticaria, rhinitis, anaphylaxis.
It is very rare to have a more serious reaction (seizures, encephalopathy have been reported).
Standard modern vaccines are not known to cause disease or to have long-term deleterious effects.
The risks of vaccinations are small compared with the health risks associated with the diseases
they're intended to prevent.
Explain the recommended immunization schedule :
Give the patient some information pamphlets, invite further questions and ask her to return
in two weeks for the child's first immunization.
Clinical Cases - Counseling 165

James Hendrik is a 40 years old man who came to your office today to discuss his weight issues. His current
weight is 250 lbs, Height 5 feet 7 inches, BMI 39.2.
Take history and address his concerns.

Counseling Case : Obesity

HOPI Crisis in patient's life, stress, anxiety?


Duration of weight gain? Assess patient's self-image: does the patient
Amount of weight gain? feel underweight, overweight or normal?
Dietary habits: frequency of meals, snacking, Does the patient feel that weight interferes
eating at night, foods eaten, binge eating, with health, activities?
guilt about food, hoarding, concealing eating Screen for eating disorders.
from others.
Estimated daily caloric intake. Past History
Exercise history. Do you have medical illnesses?
Diseases associated with weight gain: Are you on any medications?
hypothyroidism, DM type II, Cushing's, Are you allergic to any medications?
major depression, anxiety disorder, some Any surgeries in the past?
medications (TCA, steroids, OCP).
Problems associated with overweight: gout, Family and Social History
sleep apnea, cholecystitis, back pain, Do you smoke? Amount/frequency.
cardiovascular disease, hemorrhoids, lower Do you consume alcohol?
limb joint pain and osteoarthritis. Amount/frequency.
Why is patient seeking medical help for this Do you use recreational drugs? TRAPPED.
now? Any family history of cancers?
Past attempts to lose weight, successes, Any overweight relatives?
obstacles, goals.

Counseling for Obesity

Motivation: how would being at ideal body weight improve the patient's life?
Emphasize health, lifestyle, self esteem, relationship benefits.
Discuss nutrition-related problems: heart disease, obesity, hypertension, osteoporosis, anemia, dental
decay, cancer, gastrointestinal disorders, respiratory compromise, high lipids, diabetes, sleep apnea,
osteoarthritis.
Discuss diets tried and why these failed.
Fad diets involve unusual or extreme eating patterns and are not designed to be maintained for a
lifetime therefore these should be discouraged.
Weight loss agent Pondral no longer available.
SSRIs such as Paxil may assist with weight loss, unfortunately, when the drug is discontinued, most
people regain weight.
Explain mat the brain has a satiety set point which can be reset over time with reduction in caloric
intake.
Warn that the body's ability to determine caloric content is very good, and will not be fooled by so-
called diet products.
Recommend a balanced diet consisting of ordinary foods, with three distinct meals per day of small
size.
No eating at night and be careful of snacks.
Inform patient that he will be hungry for at least the first two weeks of reduced intake.
Suggest visualization techniques, redirection of interests, and to think of hunger as a sign of positive
progress on weight loss.
Group support can be beneficial too: Weight watchers, overeaters anonymous etc.
166 NAC OSCE |A Comprehensive Review

Behavior modification and positive outlook is vital for weight loss regime.
Dietary recommendations: reduce fat to 20% of caloric intake. 1200-1600 kcal/day for males.
Ideal caloric intake can be estimated at 10-12 Cal/lbs (ideal weight) in males.
Emphasize that caloric intake is more important for weight loss than food composition (i.e. excessive
calories lead to weight gain even if they are non-fat).
Exercise recommendations: 30 minutes of moderate intensity exercise, 4-5 days/week.
Sudden intense exercise in sedentary patient unwise.
More vigorous exercise can be initiated when weight is lost.
Behavioral modifications, self control, rewards on achieving goals.
Arrange regular follow-up for body mass monitoring and counseling.
Clinical Cases - Counseling 167

Peter Harper is a 35 years old man, who is a chronic smoker for the past 10 years. He came to your office
today because he wants to quit smoking.
Take history and address his concerns.

Counseling Case : Smoking

HOPI How many times have you tried to quit


Duration of smoking? smoking?
Frequency of smoking in a day? If you tried to quit before, what methods you
At what age did you start smoking? used?
What type of tobacco do you smoke- What challenges you faced while quitting?
cigarette/ cigars/ pipe/filter/non filter? What methods helped you in the plan?
When do you have your first cigarette of the What is your motivation to quit on the scale
day? of 1-10?
If you do not smoke for a while, do you
experience cravings or withdrawal Past History
symptoms? Do you have medical illnesses?
Which symptoms do you experience Are you on any medications?
- weak & tired? Are you allergic to any medications?
- sad or blue? Any surgeries in the past?
- irritable or cranky? Any h/o cancer in the past?
- difficulty concentrating?
- restlessness? Family and Social History
- anxious or jittery? Do you consume alcohol?
What people,places or events make you crave Amount/frequency.
a cigarette? Do you use recreational drugs? TRAPPED.
How does smoking help you? Any family history of cancers?
Does smoking help or harm your Occupation?
relationships?

Counseling for smoking cessation


Do not judge the patient.
Recognize the readiness of the patient to quit smoking.
Be sympathetic to the patient.
Respect the patients decision.
Be familiar with the relevant information and resources for smoking cessation.
Encourage all efforts taken.
Stress that many smokers have successfully stopped smoking.
Emphasize that smoking is a single most preventable cause of illness & death.
Talk about 5 R's for quitting:
Relevance to health.
Risks of smoking.
Rewards of quitting.
Road blocks in quitting.
Repetition of motivational intervention at each visit.
Quit plan - STAR
Set a quit date.
Tell family &c friends.
Anticipate challenge.
Remove tobacco products.
Methods of smoking cessation:
Nicotine replacement therapy - gum/patch/inhaler/nasal spray.
Tab Bupropion SR.
Tab Varenicline.
NOTES
mms
Alphabetical Index
Abortion - clinical case................................... 118 Collateral Ligament Stability......................... , 82
Acetaminophen.............................................. 30 Community Acquired Pneumonia................ , 25
Acne................................................................. 11 Congestive Cardiac Failure............................ ........... 8
Acute Bronchiolitis.......................................... 45 Congestive Heart Failure - clinical case........ ....104
Acute Cholecystitis......................................... 19 COPD............................................................ 24
acute confusional state..................................... 52 Crohns Disease.............................................. ......... 17
Acute Gastroenteritis...................................... ........16 48
Acute Myocardial Infarction.......................... 7 Deep Vein Thrombosis - clinical case........... ,,146
Acute Otitis Media........................................ 45 52
Acute Pancreatitis............................................ 17 Delirium - clinical case.................................. ,135
Acute Pharyngitis............................................ 23 Delirium Tremens.......................................... 36
Acute Pyelonephritis....................................... , . .30 Dementia........................................................ , 55
Acute Sinusitis................................................ 23 Dementia - clinical case................................. .,136
Alcohol............................................................ ,, .35 Depression................................................................56
Alcohol withdrawal......................................... 30,59 Depression - clinical case............................... 137
Allen Test........................................................ ,68 Diabetes Mellitus........................................... ......... 14
Allergic reaction.............................................. 50 Diabetic Foot - clinical case........................... ,147
Allergic Reaction............................................. ,30 Diabetic ketoacidosis..................................... , 31
Anaphylaxis..................................................... ........30 Diabetic Ketoacidosis..................................... 14
Anemia............................................................ 19,50 32
Ankle Anterior Drawer Test......................... ........ 84 Digoxin Toxicity - clinical case...................... ,106
Anorexia - clinical case................................... 133 Diverticulitis.................................................. . 17
Antenatal Visit - clinical case......................... 119 Domestic violence - counseling..................... ...... 161
Anterior Drawer Test...................................... ........82 Dysfunctional Uterine Bleeding.................... ......... 40
anti-hypertensive drugs................................... ........10 Dyslipidemia.................................................. ........... 9
Apley s Scratch Test........................................ ........ 86 Dysmenorrhea................................................ 40
Appendicitis.................................................... 16 Ectopic Pregnancy......................................... 42
Apprehension Sign......................................... ,86 Ectopic Pregnancy - clinical case.................. ....... 120
Arrhythmias.................................................... ........ 31 Emergency contraception.............................. .........41
ASA.............................................................. . , ,31 Empty Can Test............................................ ,, 86
Asthma............................................................ 24,45 Endometriosis................................................ ...... 40
Asthma - clinical case..................................... 103 Epiglottitis..................................................... 48
Athletes foot................................................... 13 Ethylene glycol............................................. . , 32
Atrial Fibrillation............................................ ...8,102 Examination - Abdominal............................. 63
Atrial Fibrillation - clinical case..................... 102 Examination - Back/Spine............................ 77
Atrophic vaginitis............................................ , .39 Examination - Breast..................................... ......... 90
Back Pain - clinical case.................................. ...... 142 Examination - Cardiovascular....................... 65
Bacterial Meningitis........................................ 49 Examination - Central Nervous System...... .......71
Bacterial Pneumonia...................................... 46 Examination - Elbow.................................... , 87
Bacterial Tracheitis.......................................... 45 Examination - Foot and Ankle.................... ..........83
Bacterial vaginosis........................................... 39 Examination - Hand and Wrist................... ......... 88
Basal Cell Carcinoma - clinical case.............. ...... 143 Examination - Hip........................................ 79
Benien Prostatic Hyperplasia - clinical case... 144 Examination - Knee...................................... , 81
Bipolar disorder............................................... 57 Examination - Lower Limb Neurological.... 75
Bowstring test................................................. 78 Examination - Mini Mental State............... 93
Breast Feeding - counseling............................ , , 159 Examination - Peripheral Vascular.............. ,6 7
Buergers Test.................................................. ........ 68 Examination - Respiratory........................... ..........69
Bulimia - clinical case..................................... 134 Examination - Shoulder................................ 85
Burns............................................................... 11 Examination - Thyroid................................. ......... 91
Ca Oesophagus - clinical case........................ 148 Examination - Upper Limb Neurological.... .......... 73
Candidiasis...................................................... 39 External Rotation Lag Sien.......................... ......... 86
Carpal Tunnel Syndrome - clinical case......... .......145 FABER.......................................................... 78,80
Cellulitis.......................................................... , , 13 Failure to thrive - clinical case....................... , .,126
Cerebrovascular Attack - clinical case............ 105 Febrile Seizures.............................................. , . 4 9
Chest Pain - clinical case................................ 111 Febrile seizures - clinical case........................ ,127
Child abuse - counseling................................ 160 Femoral stretch test.......................................,, , . 78
Chlamydia....................................................... 38 Fibromyalgia.................................................. ,, 29
Cluster headache............................................. 22 Finkelstein's test............................................ 89
Cocaine Overdose........................................... 59 General Anxiety Disorder............................. 54
Alphabetical Index
Pain Abdomen - clinical case.................................151
Genital herpes..........................................................38 Panic attack.............................................................. 53
Genital warts...........................................................38 Panic Attack - clinical case.................................... 139
Gerber Lift-OfFTest................................................86 Panic disorder...........................................................53
Gl Bleed...................................................................34 Parkinsons disease....................................................23
Golfer's Elbow......................................................... 87 Patellar Apprehension.............................................. 82
Gonorrhea................................................................38 Patellar Grind...........................................................82
Gout.........................................................................27 Pediculosis................................................................ 13
Group A Streptococcus........................................... 48 Pelvic Inflammatory Disease....................................40
Group B Streptococcus............................................41 Pelvic inflammatory disease - clinical case............ 123
Hallucinogens.......................................................... 59 Pemberton's sign...................................................... 91
Hawkins Impingement Sign....................................86 Peptic ulcer disease...................................................19
Headache - clinical case.........................................110 Peripheral Vascular Disease - clinical case.............152
Heart Failure..............................................................8 Pertussis....................................................................48
Helicobacter Pylori.................................................. 17 Phalen's test.............................................................89
Hematemesis - clinical case...................................149 Phencyclidine........................................................... 59
Hemothorax - clinical case.................................... 156 Placenta Previa - clinical case................................ 124
Hepatitis B...............................................................18 Plantar Fasciitis Test................................................ 84
HIV..........................................................................19 Pneumonia - clinical case.......................................112
Hormone Replacement Therapy............................. 41 Polymyalgia Rheumatica..........................................28
HRT - counseling..................................................162 Post exposure - clinical case................................... 113
Horner's sydrome.....................................................69 Post Operative fever - clinical case........................ 153
Hyperemesis Gravidarum........................................42 Post-traumatic stress disorder................................. 55
Hyperprolactinemia.............................................. I5p. Posterior Drawer Test.............................................. 82
Hypertension........................................................9 ,52 Pre Eclampsia - clinical case..................................125
Hyperthyroidism......................................................15 Pregnancy Induced Hypertension............................41
Hypoglycemia.......................................................... 32 Primary nocturnal enuresis - clinical case..............130
Hypothyroidism.......................................................15 Psoriasis....................................................................12
Immunization - counseling................................... 164 Psychosis...................................................................56
Immunization Schedule...........................................50 Pulmonary Embolism.............................................. 25
Impotence................................................................ 16 Pulmonary Embolism - clinical case..................... 114
Impotence - clinical case........................................108 Pulmonary tuberculosis............................................ 20
Infectious Mononucleosis - clinical case............... 107 Pyelonephritis...........................................................39
Infective Endocarditis..............................................10 Pyloric stenosis - clinical case................................ 131
Infertility - clinical case......................................... 121 Rabies....................................................................... 20
Lachman Test.......................................................... 82 Rheumatic Heart Disease........................................ 10
Laryngotracheobronchitis........................................48 Rheumatoid Arthritis...............................................26
Lasegues sign.......................................................... 78 Romberg's test.......................................................... 72
Malaria.....................................................................20 Scabies......................................................................13
Mammogram - counseling.................................... 163 Schizophrenia - clinical case..................................140
Mania.......................................................................53 Schober'sTest...........................................................78
Mania - clinical case.............................................. 138 Seizure disorder - clinical case............................... 115
McMurray's Test......................................................82 Seizures.................................................................... 21
Measles - clinical case............................................128 Septic Arthritis.........................................................29
Meningitis................................................................21 sexual dysfunction.................................................... 58
Meningitis - clinical case....................................... 109 Sexually Transmitted Infection................................ 38
Migraine.................................................................. 22 Shock........................................................................33
Mood stabilizers...................................................... 57 Smoking................................................................... 35
Myasthenia Gravis...................................................23 Smoking - counseling............................................ 167
Neck Swelling - clinical case................................. 150 Social Phobia............................................................54
Neer Impingement Sign.......................................... 86 Solitary lung nodule - clinical case........................ 154
Neonatal Jaundice - clinical case........................... 129 Speech delay - clinical case.................................... 132
Obesity - counseling..............................................165 Speed's Maneuver.................................................... 86
Obsessive-compulsive disorder...............................55 Sprain....................................................................... 33
OCP Counseling - clinical case.............................122 Straight leg raising test............................................ 78
Opioid......................................................................33 Streptococcal Pharyngitis........................................ 48
Opioid Intoxication................................................. 59 Stroke....................................................................... 33
Osteoarthritis...........................................................26 Substance abuse........................................................59
Osteoporosis............................................................ 26 Suicide - clinical case............................................. 141
Ottawa Ankle rules..................................................84
Alphabetical Index
Syphilis.....................................................................38 Trendelenburg Maneuver.........................................68
Talar Tilt Test...........................................................84 Trichomonas vaginalis..............................................39
TCA......................................................................... 34 Troisier's Sign...........................................................69
Temporal arteritis.....................................................28 Tylenol......................................................................50
Temporal Arteritis - clinical case...........................116 Ulcerative Colitis......................................................19
Tennis Elbow.......................................................... 87 Urinary tract infection..............................................29
Tension headache..................................................... 22 Urinary Tract Infection.......................................39,49
Tetanus..................................................................... 21 Viral Hepatitis - clinical case.................................117
Thomas test..............................................................80 Virchow s Node........................................................69
Thompson's Test...................................................... 84 Vocal fremitus...........................................................70
Thyroid mass - clinical case................................... 155 Vulvovaginitis........................................................... 39
Tinea Cruris.............................................................13 Warfarin................................................................... 34
Tinel's sign............................................................... 89 Whooping Cough................................................... 48
Trendelenberg test....................................................79 Yergason test.............................................................86

You might also like